Unit 1 (1).docx

  • Uploaded by: Shekhar
  • 0
  • 0
  • February 2021
  • PDF

This document was uploaded by user and they confirmed that they have the permission to share it. If you are author or own the copyright of this book, please report to us by using this DMCA report form. Report DMCA


Overview

Download & View Unit 1 (1).docx as PDF for free.

More details

  • Words: 41,153
  • Pages: 216
Loading documents preview...
2.3.1

Unit 1:

Today I will be writing to give you some suggestions of how to prepare 1b and 1h of the CSIR-NET syllabus that deals with the structure and function of the biomolecules. First thing you should keep in mind is that you have to be very focused because only around 60 DAYS are left now, so how you should prepare in a concise organized manner within this short time? Let’s find out…

Is this topic important? 

From exam point of view, this topic is a very basic portion of biochemistry and deals with the biomolecules and their properties. You can expect a lot number of questions from this part which are easy to answer but tricky ones. It is a favorite portion from which questions are set every year so don't ignore this topic. You should have your concepts cleared in these. Each time the paper consist of at least 2-3 questions on amino acids and proteins, especially application based questions are asked- like for example separation of proteins is a favorite topic, then simple problems on nucleic acids are frequently asked. So Students you should not skip this chapter, rather revise the important things that you need to concentrate on. Now you must be thinking how to start, how to organize and make the best use of time? In the following section we are going to help you out and ease out your difficulties.

How shall I start? 

Open the syllabus booklet, read it calmly. You have to be focused. You can write down the topics under this module that you need to revise again thoroughly. You all have studied these portions previously, so the most important thing that is to be done right now is recollect what all things you remember, write down these topics which you are confident enough to answer from. Then write down the portions you need to revise and these are the topics you should emphasize and prioritize these topics.

Keep in mind that these topics are from biochemistry portions and you need to get the concepts clear and the structures at your finger tips. If you are not so confident with the structures, practice it every day, at least 1 hr before you start with any other topic; that will help you to retain the structures for a longer time. Follow any basic standardized book of biochemistry that will help.

Important books: Follow any biochemistry book which you find easy to read and understand. These are some of the best books which you can refer to: 

Lehninger's Principles of Biochemistry: This book contains all the information in a readerfriendly manner. The structures and the key concepts are highlighted. You can refer to this book for amino acids and proteins, for carbohydrates, nucleic acids and for lipids.



Organic chemistry by I.L Finar: The chapter on carbohydrate contains lot of additional information which you might not find in Lehninger's Biochemistry book especially carbohydrate chemical reactions and the properties of carbohydrates.



Biochemistry by D. Das: This book is very handy for studying vitamins. It is organized and most of the key things on vitamins are provided in a very schematic way.



Biochemistry by Lubert Stryer: This book has an entire chart for A, B, Z forms of DNA and the structure and properties of t-RNA is well illustrated.



Karp Cell and Molecular Biology: The synthesis pathway and the applications of mi-RNA are provided in this book in a very concise manner.



Instant Notes in Biochemistry: A very handy book if you want to study in a concise manner



Additional books if you want to study these topics in more details:



Genes by Benjamin Lewin: Especially for nucleic acids.

Note: You should keep in mind that these books are the ones commonly used by most candidates appearing for CSIR-NET. In case you come across some other book/s which you find easier and more suitable to your needs, you are always welcome to go through them.

Prioritize the subtopics: 

This is just to help you out that among these topics which are the ones you will give more stress on Amino acids & proteins> carbohydrates> nucleic acids> lipids> vitamins

 But remember one thing the chapter with least priority shouldn't be skipped. This is just to give you an idea from which topic you can expect more no. of questions.

Suggestions 

Write and practice because writing down after going through the text helps you in retaining the structures for a longer time.



Practice problem sums and go through the previous year’s question papers.



Facts are not so important from these topics.



Remember the key concepts.

Topic wise suggestion: I have provided below the important topics which you should revise regularly because these are the basic things and questions come based on these concepts.

Carbohydrates: 

Structures of mono, di, polysaccharides- practice these



You should be well versed with the key terms like mutarotation, epimers, anomers, D &L, d & l, reducing and non-reducing sugars.

Proteins & amino acids: 

Structures of 20 a. a. (remember their R groups) - just break up the structure of a.a into their backbone and you will see that it's the R group that varies, so in this way it will help you retain all of the 20 a.a.



The pKr values of charged a.a; the pK1 and pK2 are 2 & 9 approx for all a.a, so you don't need to remember these values.



You should get your concepts cleared regarding titration curves, pI and zwitter ion.



Revise the primary, secondary, tertiary & quaternary structures of proteins



Ramachandran plot (what this plot suggest and the quadrants where each conformations are found)



Hemoglobin & myoglobin (binding curve of each and their overall structural difference)

Lipids: 

Structures of fatty acids if you can remember then it is always useful.



Basic types of lipids (glycolipids, sphingolipids)



Cholesterol (structure not required)



Iodine no; saponification number (brief idea will help)

Nucleic acids: 

Practice numerical problems



Remember the structure of the 4 bases



Properties of A, B, Z DNA



General properties of nucleic acids



Micro-RNA(how they are synthesized & their uses)



The structure of tRNA is important. You should be well versed with the structures.

Vitamins: 

Structures not required from this topic



Functions and sources of all vitamins



Biosynthesis of Vit D, A

Note: Follow any basic book for the functions and sources of all vitamins.

Nucleic acids and DNA topology 1. If in evolutionary time scale Nucleosomes had not been evolved, what would have been the entire length of human DNA molecule (2n) present in one cell? ( human cell has 3 x 10^9 bp and rise in helix is 34nm) a) 1 meter b) 2 meter c) 3meter d) 0.5 meter

2. What is the reason for the reduced gene density among the Eukaryotes than prokaryotes? a. Presence of Intergenic sequences b. Larger cell volume c. Larger body size d. Less no.of genes

3. What is the Lk of a 5,000 bp circular duplex DNA molecule, bound by a nucleosome? a. 476 b. 475 c. 0 d. 464

4. What is the axial ratio (length: diameter) of a viral DNA molecule 20μm long? a. 1 x 105 b. 1 x 104 c. 2 x 104 d. 2 x 105

5. Why RNA is hydrolyzed by alkali, whereas DNA is not? a. RNA has Uracil, unlike DNA b. The 2’deoxy sugar of RNA is more susceptible than 2’ oxy ribose of DNA c. The 2’deoxy sugar of DNA is less susceptible than 2’ oxy ribose of RNA d. The 2’ Deoxy ribose of DNA is not affected by alkali as DNA is present inside the Nucleus and wrapped by nucleosomes, so that no DNA is free for alkali action.

6. Once nerve cells become mature, they don’t usually undergo cell division. Based on your knowledge of the cell cycle, you would predict that mature nerve cells become arrested in the ____________ of the cell cycle.

a. b. c. d. e.

G0 phase S phase prophase G1 phase G2 phase

7. All of the following events occur during prometaphase EXCEPT: a. the nuclear envelope breaks down. b. the nucleoli disappear. c. the mitotic spindle is completely assembled. d. the spindle fibers “capture” chromosomes. e. the duplicated chromosomes become visible with the light microscope.

8. Which of the following statements concerning the cell cycle is FALSE? a. The activity of Cdks increases and decreases during the cell cycle. b. Cyclins fluctuate during the cell cycle. c. Cdks are active only when they bind to cyclins. d. The anaphase-promoting complex stimulates the separation of sister chromatids e. M-Cdk inhibits mitosis.

9. Thirty-six colonies grew in nutrient agar from 1.0 ml of sample withdrawn from a solution diluted to 10-5 in a standard plate count procedure. How many cells were in the original sample? a. 360 b. 3,600 c. 360,000. d. 3,600,00.

10. Consider the following statement. 1) Most human cells are diploid, except Megakaryones. 2) Megakaryones have 128 copies of each chromosome. 3) All human cells are diploid, except germ cells 4) Every human cell has a neuron, a lymph and blood source. Choose the correct statement. a. Only 1) b. Both 1) and 2) c. Only 3) d. 1), 2), 3) and 4)

11. If in evolutionary time scale Nucleosomes had not been evolved, what would have been the entire length of human DNA molecule (2n) present in one cell? ( human cell has 3 x 10^9 bp and rise in helix is 34nm) a. 1 meter b. 2 meter c. 3 meter d. 0.5 meter e. 1,800,000

12. Chromosome can have multiple replication sites but not multiple centromeres. What would be the effect on chromosome if you engineer it to have multiple centromeres? a. Results in aneuploid cells. b. Results in non-disjunction of chromosomes c. Results in breakage of chromosomes during strand separation. d. Results in duplication of e. Chromosome

13. The circular chromosomes after replication will still be interlocked which has to be a. DNA topoisomerase b. DNA gyrase c. Nucleases d. Restriction enzymes

14. Compaction ratio achieved by nucleosome formation is a. 1/3 b. ¼ c. 1/8 d. 1

15. Which of the following statement is true? A. Chromosomes are membrane bound and are stable in entire life span of a cell. B. If barr body is formed in female, no genes form father will be expressed in the child. a. Only A b. Only B c. Both A and B d. None of these.

16. Some of the heterochromatin region can convert into euchromatic regions, in different cell cycle stages. They are called as

a. b. c. d.

Constitutive Heterochromatin Facultative Heterochromatin Euchromatin None of the above

17. Telomerases are the a. Ends of the chromosomes b. Enzymes involved in replicating ends of chromosomes c. Central part of chromosomes d. Proteins which are necessary for termination of replication

18. Telomerases are over expressed in a. Actively dividing cells b. Gamete cells c. Bone marrow cells d. Cancer cells

19. Every chromatid contains, how many DNA strands a. 1 b. 2 c. 3 d. Different chromosomes have different DNA number

20. Topoisomers are a. Different structural forms of DNA b. Different 3D topology of cccDNA c. Enzymes which neutralize Supercoiling d. DNA supercoils

21. Topoisomerases are a. a.Different structural forms of DNA b. b.Different 3D topology of cccDNA c. c.Enzymes which neutralize Supercoiling d. d.DNA supercoils

22. Topoisomerase I a. Requires ATP b. Cleaves only one strand at a time

c. Called as Gyrase in E.coli d. Decreases linking number

23. Active site of Topoisomerase I contains a. Tyrosine b. Tryptophan c. Glycine d. Fe+3 ions

24. No. of chromosomes present in Yeast, S. cereviseae is a. 14 b. 16 c. 18 d. 25

25. Writhe corresponds to a. Coiling of ssDNA to form dsDNA b. Coiling of dsDNA to form triplex DNA c. Decoiling of cccDNA to form supercoiled DNA d. No. of times the dsDNA crosses another dsDNA strand in supercoiled cccDNA

26. Types of writhe a. 1 b. 2 c. 3 d. 4

27. If Twist is +ve , helix coiling is a. Right handed b. Left handed c. Both d. Cannot be determines

28. If interwound writhe is –ve then, the supercoiling is said to be a. Right handed b. Left handed c. East handed d. West handed

29. If Spiral writhe is +ve , the DNA Supercoiling is a. Right handed b. Left handed c. East handed d. West handed

30. Barr body is an example of a. Constitutive heterochromatin b. Facultative heterochromatin c. X chromosome expression d. Y chromosome dominance

31. A cell nucleus was found to have 10 nucleosomes. Considering the DNA wound around Histones is 146 bp and linker DNA is 40 bp. What would be the length of DNA strand if all Histones are removed. a. 0.1 nm b. 200 A c. 0.5 nm d. 0.589 micrometer

32. The DNA in sperm, is coiled by modified Histones called a. Vitrous DNA proteins b. Amino Histones c. Protamins d. Acetyles

33. Heterochromatins are rich in a. LINE elements b. Sine elements c. Tandem repeats and satellite DNA d. Visual band

34. If DNA concentration is kept constant, which of the following will show long renaturation time? a. DNA with many inverted and mirror repeats b. DNA with no repeats c. DNA with many direct and tandem repeats d. DNA with Transposon

35. Which positions in a purine ring of a purine nucleotide in DNA have the potential to form hydrogen bonds but are not involved in Watson-Crick base pairing? a. N-3 and N-7 b. N-2 and N-9 c. Only N-3 d. Only N-9 36. If a strand of a helix has 30% adenine, which of the following inferences is true? a. The strand has 30% thymine b. The opposite strand has 60% adenine c. The strand has 40% guanine d. The opposite strand has 30% thymine

37. Would you agree with the statement that Tm is the temperature at which 50% of the DNA molecule in a solution is single stranded? a. Yes b. No c. Statement is correct, but its not for DNA, but for RNA d. Cannot be determined

38. Hairpins may form at palindrome sequences in single strands of either RNA or DNA. How is the helical structure of a long and fully base-paired (except at the end) hairpin in RNA different from that of a similar hairpin in DNA? a. RNA helix is in the A conformation: the DNA helix is generally in the B conformation. b. RNA helix is in the B conformation: the DNA helix is generally in the A conformation c. RNA helix is single stranded: the DNA helix is generally double stranded d. RNA helix is having Z conformation; DNA helix has B conformation.

39. Why do you think DNA does not contain Uridine, instead of Thymidine? a. It will de stabilize the DNA b. It will convert DNA into RNA and hence breakdown of central dogma c. Uridine can be converted into cytosine, causing nucleotide transversion d. Uridine is unstable at nucleus pH.

40. Peudo uridine is modified form of uridine and is present in tRNA. It plays major role in ensuring the structure of tRNA. The modified bases give it different hydrogen bonding patterns, in turn different secondary structures. Which of the following Bond is present between Ribose sugar and the peudo uridine? a. C-5 to ribose glycosidic bond

b. N-1 to ribose glycosidic bond c. N-9 to ribose glycosidic bond d. C-1 to ribose glycosidic bond

41. DNA and RNA are the nucleic acids differing in only one OH- group. The DNA forms major genetic material and RNA , the messenger role and catalytic roles. The trans-esterification reaction involves formation of esters in the ribose sugar. What may be the reason, that this reaction is specific for only RNA , but not DNA. a. The esterase enzymes are present in the nucleus, where as DNA is present in the nucleus. b. The DNA has 3 –Phosphate groups which will not allow trans esterification c. The DNA has no 2’OH- group which is necessary for transesterification d. The RNA has no 2’H- which is acts as inhibitor of esterace enzyme.

42. DNA of the Human Body Calculate the weight in grams of a double-helical DNA molecule stretching from the earth to the moon (~320,000 km). The DNA double helix weighs about 1 x 1018g per 1,000 nucleotide pairs; eachbase pair extends 3.4 Å. For an interesting comparison, your body contains about 0.5 g of DNA! a. 9.4 x 10-4g b. 2.5 x 10-4g c. 9.4 x 10-8g d. 10-4g

43. The cells of many eukaryotic organisms have highly specialized systems that specifically repair G– T mismatches in DNA. The mismatch is repaired to form a GqC (not AUT) base pair. This G–T mismatch repair mechanism occurs in addition to a more general system that repairs virtually all mismatches. Can you suggest why cells might require a specialized system to repair G–T mismatches? a. G-T is the most common mismatches in eukaryotic cell b. G-T is the most rare mismatches in eukaryotic cell c. G-T mismatch requires special enzymes as G forms 2 hydrogen bonds d. G-T mismatch will lead to base transversion, hence we need special enzymes.

44. Explain why the absorption of UV light by double-stranded DNA increases (hyperchromic effect) when the DNA is denatured. a. Base stacking effect b. Base complementation effect c. B-conformation of DNA which will not allow absorption d. UV light causes Thymine dimer and is removed as helix opens up

45. Base Pairing in DNA In samples of DNA isolated from two unidentified species of bacteria, X and Y, adenine makes up 32% and 17%, respectively, of the total bases. One of these species was isolated from a hot spring (64 C). Suggest which species is the thermophilic bacterium. a. X b. Y c. Both are from hot springs d. Cannot be determined without further information about its nature.

46. Solubility of the Components of DNA: Which of the following chemical is highly soluble in water (most soluble to least soluble): deoxyribose, guanine, phosphate? a. Deoxyribose b. Guanine c. Phosphate d. Both phosphate and guanine are equally soluble.

47. Why the DNA is selected by nature but not RNA? a. The DNA is more stable as it can stay in helical structure, but not RNA b. RNA evolved later in evolutionary time scale. c. DNA has deoxyribose which makes it stable towards hydrolysis d. RNA contains uridine in place of thymine, which makes it stable.

48. If cell has codons made of 4 bases, instead off triplet, how many codons would possibly be made up? a. 8 b. 16 c. 64 d. 256

49. Ribozymes are the types of RNA which have catalytic property. RNAse P is the first discovered Ribozyme, which is involved in generation of tRNA molecules form larger precursors. It is made up of RNA as well as Protein. Which of them have catalytic property? a. RNA b. Protein c. Both d. Cannot be determined

50. Which of the following diagram indicated the correct form of U:A:U bonding?

51. Substrate for DNA synthesis is a. Nucleotide triphosphate b. Nucleoside triphosphate c. Nucleotide pyrophosphate d. Ribonucleotide triphosphate

52. An experiment was designed to find out the relation between DNA uptake and transformation efficiency. 32P-labelled genomic DNA from Bacillus subtilis (A) and cold genomic DNA from clostridium jejeuni (B) mixed in various proportions was used to transform Bacillus subtilis. The results obtained are tabulated below.

Set no

1 2 3 4

Concentration of DNA used(µg/ml) A 1 1 1 1

Uptake of 32PTransforming labelled DNA (%) efficiency (CFU) B 1 10 100 1000

90 82 15 5

30 10 5 1

The following interpretations could be made(AtoD) A. The transformation is dependent on recombination between homologous sequences B. Cells did not distinguish between homologous or heterologous sequences for uptake of DNA C. DNA uptake is based on specific receptors

D. DNA degradation dictates transformation efficiency.

53. siRNAs and miRNAs are used for achieving gene silencing. Although, major steps are similar there are distinct differences in the key players of the two processing pathways. Following statements relate to some characteristic features of gene silencing. A. Both siRNAs and miRNAs are processed by cytoplasmic endonuclease Dicer. B. 'Drosha' s needed for processing miRNAs and precursor siRNAs. C. Both siRNAs and miRNAs show association with Argonaute protein. D. Both the processing pathways involve RISe complex. Which of the following combinations is NOT correct? a. A and C b. C and D c. A and B d. D and A

54. It has been observed that in 5-10% of the eukaryotic mRNAs with multiple AUGs, the first AUG is not the initiation site. In such cases, the ribosome skips over one or more AUGs before encountering the favorable one and initiating translation. This is postulated to be due to the presence of the following consensus sequence (s): A. CCA CC AUG G B. CCG CC AUG G C. CCG CC AUG C D. AAC GG AUG A Which of the following sequence sets related to the above postulations is correct? a. b. c. d.

A and B A and C C and D B and D

55. The DNA duplex absorbs 40% less UV rays than individual bases. Which of the following statement can be a reasonable explanation for the above statement? a. The duplex formation diminishes the capacity of bass to absorb due to base stacking. b. The duplex has major and minor grooves c. The individual bases show different chemical property than that of Duplex form d. The duplex is present inside the nucleus, where as individual bases in nucleotide pool of cytoplasm.

56. Topoisomerases can cut and re knot the DNA strands. They break the phosphodiester bond between 2 adjacent bases. Surprisingly they do not use ATP hydrolysis energy to cleave the high energy Phosphate bond. What may be a possible explanation for this peculiar behavior? a. The total ATP used to cleave the bond, is regenerated at the end of the reaction, hence net ATP is used is Zero. b. The phospho diester bonds are not high energy bonds c. They pull the adjacent bases apart, thus breaking the bond d. Topoisomerases form phospho-tyrisine bond, which conserves the energy that is used to form the ester bond to rejoin the DNA.

57. You are separating a mixture of DNA molecules by Electrophoresis. Which of the following is the correct order of their separation? a. Well --> Supercoiled DNA --> linear relaxed DNA --> highly supercoiled cccDNA --> end point b. Well --> Supercoiled DNA --> highly supercoiled ccd DNA --> linear relaxed DNA --> end point c. Well --> linear relaxed DNA --> supercoiled DNA --> highly supercoiled cccDNA--> end point d. All will be at same location

58. Ethidium Ions cause DNA to unwind. They are florescent under UV radiation and used extensively in DNA mobility studies. They intercalate the DNA chaging the normal rotation of base pairs from 36° to 10°. If you happen to add one ethidium ion between every 2 bases of DNA, how many bases you will find in per turn of the helix? a. 36 b. 72 c. 10 d. 5.5

59. Which of the following statement about the RNA is false? a. RNA cannot form B-DNA structure because of 2’-hydroxyles. b. RNA can form local complementary regions , but not long range regular helicity of DNA c. RNA double helix has G:U base pairing d. None of the above

60. Why Uracil is present only in RNA and not in DNA? a. It is bulky then Thymine, affecting DNA stability b. Uracil can easily convert to Cytosine, causing frame shift in DNA c. Uracil forms 3 hydrogen bonds, hence cannot bind to Adenine in DNA d. Uracil can easily convert to Cytosine, causing Purine transition.

61. Deoxyribonse is attached to Adenine in which N-position and using what bond? a. N9 of adenine and by glycosidic bond b. N9 of adenine and by ester linkege c. N9 of adenine and by hydrogen bond d. N1 of adenine and by glycosidic bond

62. Hydrogen bond is a major intermolecular attraction force in biomolecules. Which of the following statement is false about hydrogen bonds? a. Hydrogen bonds are weak interactions but strong enough in close encounters. b. Hydrogen atom, accepts a hydrogen bond c. Hydrogen atom, donates a hydrogen bond d. Hydrogen bond is responsible for Water boiling temperature to be 100°C

63. The DNA duplex absorbs 40% less UV rays than individual bases. Which of the following statement can be a reasonable explanation for the above statement? a. The duplex formation diminishes the capacity of bass to absorb due to base stacking. b. The duplex has major and minor grooves c. The individual bases show different chemical property than that of Duplex form d. The duplex is present inside the nucleus, where as individual bases in nucleotide pool of cytoplasm.

64. DNA is eukaryotes is always a. Circular b. Double stranded and Negatively supercoiled c. Double stranded and Positively supercoiled d. Linear and coiling occurs only during cell division

65. Which of the following is responsible for introduction of negative Supercoiling in Eukaryotic DNA? a. DNA Topoisomerases b. DNA gyrases c. Nucleosomes d. Histones

66. Topoisomerases can cut and re knot the DNA strands. They break the phosphodiester bond between 2 adjacent bases. Surprisingly they do not use ATP hydrolysis energy to cleave the high energy Phosphate bond. What may be a possible explanation for this peculiar behavior? a. The total ATP used to cleave the bond, is regenerated at the end of the reaction, hence net ATP is used is Zero. b. The phospho diester bonds are not high energy bonds

c. They pull the adjacent bases apart, thus breaking the bond d. Topoisomerases form phospho-tyrisine bond, which conserves the energy , that is used to form the ester bond to rejoin the DNA.

67. Ethidium Ions cause DNA to unwind. They are florescent under UV radiation and used extensively in DNA mobility studies. They intercalate the DNA chaging the normal rotation of base pairs from 36° to 10°. If you happen to add one ethidium ion between every 2 bases of DNA, how many bases you will find in per turn of the helix? a. 36 b. 72 c. 10 d. 5.5

68. Which of the following statement about the RNA is false? a. RNA cannot form B-DNA structure because of 2’-hydroxyles. b. RNA can form local complementary regions , but not long range regular helicity of DNA c. RNA double helix has G:U base pairing d. None of the above.

69. Which positions in a purine ring of a purine nucleotide in DNA have the potential to form hydrogen bonds but are not involved in Watson-Crick base pairing? a. N-3 and N-7 b. N-2 and N-9 c. Only N-3 d. Only N-9

70. If a strand of a helix has 30% adenine, which of the following inferences is true? a. The strand has 30% thymine b. The opposite strand has 60% adenine c. The strand has 40% guanine d. The opposite strand has 30% adenine

71. Would you agree with the statement that Tm is the temperature at which 50% of the DNA molecule in a solution is single stranded? a. Yes b. No c. Statement is correct, but its not for DNA, but for RNA d. Cannot be determined

72. Hairpins may form at palindrome sequences in single strands of either RNA or DNA. How is the helical structure of a long and fully base-paired (except at the end) hairpin in RNA different from that of a similar hairpin in DNA? a. RNA helix is in the A conformation: the DNA helix is generally in the B conformation. b. RNA helix is in the B conformation: the DNA helix is generally in the A conformation c. RNA helix is single stranded: the DNA helix is generally double stranded d. RNA helix is having Z conformation; DNA helix has B conformation.

73. Why do you think DNA does not contain Uridine, instead of Thymidine? a. It will de stabilize the DNA b. It will convert DNA into RNA and hence breakdown of central dogma c. Uridine can be converted into cytosine, causing nucleotide transversion d. Uridine is unstable at nucleus pH.

74. Peudo uridine is modified form of uridine and is present in tRNA. It plays major role in ensuring the structure of tRNA. The modified bases give it different hydrogen bonding patterns, in turn different secondary structures. Which of the following Bond is present between Ribose sugar and the peudo uridine? a. C-5 to ribose glycosidic bond b. N-1 to ribose glycosidic bond c. N-9 to ribose glycosidic bond d. C-1 to ribose glycosidic bond

75. DNA and RNA are the nucleic acids differing in only one OH- group. The DNA forms major genetic material and RNA , the messenger role and catalytic roles. The trans-esterification reaction involves formation of esters in the ribose sugar. What may be the reason, that this reaction is specific for only RNA , but not DNA. a. The esterase enzymes are present in the nucleus, where as DNA is present in the nucleus. b. The DNA has 3 –Phosphate groups which will not allow trans esterification c. The DNA has no 2’OH- group which is necessary for transesterification d. The RNA has no 2’H- which is acts as inhibitor of esterace enzyme.

76. Base Pairing in DNA In samples of DNA isolated from two unidentified species of bacteria, X and Y, adenine makes up 32% and 17%, respectively, of the total bases. One of these species was isolated from a hot spring (64 C). Suggest which species is the thermophilic bacterium. a. X b. Y c. Both are from hot springs

d. Cannot be determined without further information about its nature.

77. Solubility of the Components of DNA: Which of the following chemical is highly soluble in water (most soluble to least soluble): deoxyribose, guanine, phosphate? a. Deoxyribose b. Guanine c. Phosphate d. Both phosphate and guanine are equally soluble.

78. Trans-acylation reaction is specific for a. DNA, as it lacks 2’OH group b. RNA, as it has 2’OH group c. DNA, as it has 2’H group d. RNA , as it lacks 2’H group

79. The conformation of a nucleotide in DNA is affected by rotation about how many bonds? a. 3 b. 4 c. 6 d. 7

80. The bond present in 5’cap of mRNA is a. 3’ – 5’ diphosphate bond b. 3’ – 5’ phosphodiester bond c. 5’- 5’ triphosphate bond d. 5’ -3’ phosphodiester bond

81. In a given nucleic acid G+A is not equal to C+T content. This indicates that the sample is a. b. c. d.

AT rich GC rich ssDNA dsDNA

82. The melting temperature Tm of a duplex is defined as the temperature at which half the molecules have dissociated into single strands. Tm will be maximal at a. a. b. c.

Low ionic strength and high DNA conc. b.High ionic strength and high DNA conc. High ionic strength and low DNA conc. Low ionic strength and low DNA concentration

83. If a solution of double stranded DNA is heated above its melting temperature, its absorbance will a. b. c. d.

Decrease Increase Remain unchanged Initially increase and then decrease

84. The molecular weight of an mRNA that codes for a protein of molecular weight 75000 is closest to ( Avg. weight of Nucleic acid = 330amu ; amino acid = 110amu) a. b. c. d.

6000 60000 600000 600

85. A non supercoiled B-DNA molecule is composed of 4800bps. How many helical turns are present? a. b. c. d.

10 380 460 Impossible to determine

86. The G+C content of bacteriophage 13 ds DNA is 68%.What would you expect The G+C content of its mRNA?? a. b. c. d.

about 68% about 34% about 32% about 86%

87.What is the correct statement of following pertaining to the mass of bases present in a ds DNA with 50% GC content??

a. b. c. d.

a.A=T b.C>G c.A>T d.T>A

88. The conformation of a nucleotide in DNA is affected by rotation about how many bonds? a. b. c. d.

1 3 7 6

89. In humans, the Barr body is an a. b. c. d. e.

active X chromosome in females active X chromosome in males inactive Y chromosome in males inactive Y chromosome in females inactive X chromosome in females

90.The nucleoside adenosine exists in a protonated form with a pKa of 3.8. The percentage of the protonated form at pH 4.8 is closest to a. b. c. d. e.

1 9 50 91 99

91. The distance between two base pairs in Watson-Crick B-DNA is a. b. c. d.

34nm 3.4nm 0.34nm 0.034nm

92. An rRNA sample was degraded into its constituent nucleotides. The absorbance of this sample at 260 nm would a. decrease if the degradation was effected by alkaline digestion but increase if the same was effected by a ribonuclease

a. increase if the degradation was effected by alkaline digestion but decrease if the same was effected by a ribonuclease b. remain the same irrespective of the use of chemicals or enzymes c. increase irrespective of the use of chemicals or enzymes

93. In a stretch of D N A , 10 % of guanines form non-Watson and Crick base pairing with adenine. If the segment contains 29% guanine, the amount of thymine would be a. b. c. d.

21% 20% 29% 18%

94. Telomerase is composed of a. b. c. d.

only protein only RNA Protein and RNA Protein and DNA

95.Under a given renaturing condition E. coli DNA reassociates with a Cotin of 4.0 and an unknown DNA sample with a Cotln of 283. T h e complexity of this unknown DNA will be a. b. c. d.

4.2X 106bp 6 X 1 0 bp 3.0X 108 bp 340 bp

96. Which mutation cannot occur in a tRNA encoding gene? a. b. c. d.

Deletion Transition Transversion Nonsense mutation

Answer key

1. b. 2. a 3. a 4. b 5. c 6. a 7. c 8. a 9. c 10.d 11.b 12.c 13.a 14.b 15.d 16b 17.d 18.d 19.a 20.b 21.c 22.b 23.a

24.b 25.d 26.b 27.a. 28.a 29.a 30.b 31.d 32.c 33.c 34.c 35.a 36.d 37.a 38.a 39.c 40.b 41.c 42.a 43.a 44.a 45.b 46.c 47.c 48.d

49.a 50.a 51.b. 52.a 53.b 54.a 55.a 56.d 57.c 58.a 59.d 60.d 61.a 62.c 63.a 64.b 65.c 66.d 67.a 68.d 69.a 70.d 71.a 72.a 73.c

74.b 75.c 76.b 77.c 78.b 79.d 80.c 81.c 82.b 83.b 84.c 85.C 86.a 87.a 88.c 89.e 90.b 91.c 92.d 93.a. 94.c 95.a 96.d

1. When an actively photosynthesizing plant is exposed to 14CO2 , which 2 carbon atoms in the resulting radioactive glucose will be labeled first? a. C3 and C4 b. C3 and C5 c. C1 and C2 d. C1 and C6 2.Hydrogen bond is a major intermolecular attraction force in biomolecules. Which of the following statement is false about hydrogen bonds? a. Hydrogen bonds are weak interactions but strong enough in close encounters. b. Hydrogen atom, accepts a hydrogen bond c. Hydrogen atom, donates a hydrogen bond d. Hydrogen bond is responsible for Water boiling temperature to be 100°C 3. Difference in pH of cell cytoplasm and Nucleus is a. 0 b. 1 c. 7.2 d. 5.5 4. Which is the cholesterol esters that enter cells through the receptor-mediated endocytosis of lipoproteins hydrolyzed? a. Endoplasmin reticulum b. Lysosomes. c. Plasma membrane receptor d. Mitochondria 5. Oxidation of which substance in the body yields the most calories a. Glucose b. Glycogen c. Protein d. Lipids 6. Which of the following phospholipids is localized to a greater extent in the outer leaflet of the membrane lipid bilayer? a. Choline phosphoglycerides b. Ethanolamine phosphoglycerides c. Inositol phosphoglycerides d. Serine phosphoglycerides 7. Which of the following is true about person suffering from Galactosemia? 1. He cannot synthesize Galactose 2. He cannot metabolize the Galactose 3. Suffers from Hepatomegaly

4. a. b. c. d.

It is Autosomal recessive disorder 1,2,3,4 2,3,4 1,2 Only 2

8. Orlistat is a drug widely used to control Obesity. Its mode action would be a. By inhibiting gastric and pancreatic lipases b. By inhibiting fatty acids synthesis c. By increasing allergy for fatty acids d. By inhibiting pepsin and pancreatic insulin. 9. The pI of a protein is 8.3. To carry out ion-exchange chromatography of this protein at pH = 7.0, the following should be used to ensure binding and fractionation: a. Cation exchanger resin b. Anion exchanger resin c. Reactive ion exchanger resin d. Polyanion exchanger resin 10. Jitesh carried out a steady-state kinetics experiment for an enzyme degradase with and without molecule X as part of the reaction mixture. The reactions were carried out for 5 minutes and he got the following results: Substrate Concentration (milliMolar) 0.1 0.5 1.0 1.5 2.0 2.5 3.0 3.5

Amount of Amount of product product formed with X without X (milliMoles) (milliMoles) 0.4 0.2 2.1 1.3 4.3 2.7 6.2 4.1 8.1 5.4 8.3 6.7 8.4 8.1 8.5 8.3

These results suggest that: (a) Molecule X is a competitive inhibitor (b) Molecule X is an allosteric inhibitor (c) Molecule X is a cofactor (d) Molecule X has no effect on catalysis

11. Irshath is recording the electrical activity of neurons in a cat when the cat is shown light bars at various angles. Two neurons respond to these light bars in the following manner:

What can you infer the above data?

a. b. c. d.

Bar angle does not affect neural response in Neuron 1 or 2 A bar at 50 degrees elicits no response in either of the neurons A bar at 20 degrees elicits 50% of the maximal response in Neuron 2 For any given angle the difference in response levels of Neuron 1 and 2 is a constant.

12. A student conducted an experiment where CO2 and N2 were bubbled through water in beakers A and B respectively. He recorded the pH in each of the solutions every 5 min. What is the most valid conclusion the student could draw from these results:

Time (min) 5 10 15 20

a. b. c. d.

pH reading A 7.5 7.2 7.0 6.8

B 7.5 7.3 7.5 7.4

The change in pH was too small to be significant Bubbling CO2 through water makes it more acidic Bubbling N2 through water makes it more acidic Both CO2 and N2 make water more acidic.

13. Alzheimers’ protein-A multimerizes at pH 2 form aggregates which leads to Alzheimers’ disease. In the brain the pH is always neutral ~7. So what leads to aggregation of A that induces Alzheimers’ in people greater than 60 years of age?

a. Older brains are more acidic. b. At pH 7, there will be very fast kinetic equilibrium between oligomers and monomers which can change with age to induce aggregation c. At pH 7, the reaction does proceed to oligomerization but the rate is so low that it takes 60 years to form aggregates d. Aggregates form during intermittent drops pH throughout your life. 14. A gene is transcribed at the maximum rate of per second, and codes for a transcription factor (TF). This transcription factor inhibits the transcription of its own gene, and this is the only way this gene and TF are regulated. Which of the following graphs is most likely to describe how the steady-state concentration of the TF depends on the value of k? a. A and C b. C c. B d. D

15. What volume of 20% sucrose would you use to make 2mL of 5% sucrose solution a. 0.0005 mL b. 0.05 mL c. 0.5 mL d. 0.005 mL 16. The equilibrium constant (K) for a reaction A + B = AB is 1 x 10-6 M, where K is defined as K = [A][B]/[AB]. If the rate constant for association of A and B is 2 x 107 M-1 S-1, what is the rate constant (RC) for the dissociation of AB and the average life-time (T) of the complex? a. RC = 2 x 10-13 S-1 and T = 1 microsecond b. RC = 20 S-1 and T = 50 milliseconds

c. RC = 5 S-1 and T = 200 milliseconds d. RC = 2 x 10-7 S-1 and T = 0.05 microseconds 17. An unknown, containing some combination of alanine, lysine or aspartic acid, is subjected to paper electrophoresis at pH = 7.0. Ninhydrin treatment (to stain them) shows some amino acid at the negative electrode and some amino acids have not moved from the center. No amino acid is found at the positive electrode. Which amino acid(s) is (are) in the unknown? a. Alanine and lysine but not aspartic acid b. Alanine and aspartic acid but not lysine c. Only alanine d. All the three amino acids 18. Srikanth wants to characterize the enzymatic properties of a novel enzyme (nE1) involved in NAD salvage pathway in human cells. It turns out that this pathway and the enzyme are evolutionarily conserved. But previous studies indicate that this pathway is critical in human cells. Manohar aims to establish that nE1 is the most important enzyme in this pathway. During his studies he happened to compare nE1 activities from mice (m-nE1) and humans (h-nE1). Interestingly, his experiments showed that the Km values for m-nE1 and h-nE1 were 10mM and 20mM, respectively. Based on these results what is the best interpretation? e. nE1 catalyzes the rate limiting step in mice but not in humans f. m-nE1 has more affinity to the substrate than h-nE1 g. nE1 catalyzes the rate limiting step in humans but not in mice h. h-nE1 has more affinity to the substrate than m-nE1 19. The conversion of Pyruvate to oxaloacetate is likely to require which of the following coenzymes?

a. b. c. d. e.

Biotin Vitamin B12 TPP Pyridoxal phosphate FAD

20. In an intact cell, free energy change associated with an enzyme catalyzed reaction is frequently different from the standard free energy change of the same reaction because in the intact cell the a. activation energy is different b. reaction is always near equilibrium c. enzyme may be regulated allosterically d. reactants are not at 1M concentration e. reaction may be catalyzed by more than one enzyme 21. Plants and some bacteria differ from animals in that plants and some bacteria can a. form polymers from glucose b. use carbon dioxide to increase their biomass c. produce NADH via reductive reactions d. synthesize glutamate and aspartate e. use glucose by the glycolytic pathway 22.Which of the following can act as nucleophile in metabolic reactions? I.Nitrogen of an amino group II.Oxygen of hydroxyl group III.Carbon of carbonyl group a. b. c. d. e.

a.I only b.II only c.III only d.I and II only e.I, II and III

23. Which of the following pairs of compounds are interconvertible in the liver by a single polypeptide chain containing two different catalytic sites? a. glucose and glucose-6-phosphate b. 3-phosphoglycerate and PEP c. PEP and Pyruvate d. Fructose-6-phosphate and fructose-1,6-bisphosphate e. Fructose-6-phosphate and fructose-2,6- bisphosphate 24. Which of the following compounds is a positive allosteric regulator of the enzyme pyruvate craboxylase? a. ATP b. acetyl CoA

c. Biotin d. PEP 25. Phosphofructo kinase is characterized by which of the following statements? a. it is a major regulatory enzyme in glycolysis b. ATP is a substrate of the enzyme c. ATP is a negative modulator of enzyme d. Citrate is a posiive modulator of enzyme 26. Conversion of glucose to glucose-6-phosphate requires ATP yet. Critically ill patients are given glucose solutions intravenously instead of G6P. The reason for not giving G6P directly is a. G6P is degraded fast in blood before it enters the cells b. commercial preparations of G6P are always contaminated with toxic chemicals c. high cost of G6P d. cells cannot take up G6P

27. In anaerobic glycolysis, 2 mol of inorganic phosphate are used per mole of glucose consumed. Which of the following enzymes catalyzes the uptake of inorganic phosphate? a. hexokinase b. PFK c. Pyruvate kinase d. Glyceraldehyde-3-phosphate dehydrogenase 28. GAPDH, an enzyme of glycolytic pathway, was modified with iodoacetic acid that resulted in the loss of enzyme activity. This is due to modification of active site residue a. Lysine b. Glutamic acd c. Cysteine d. Cystine 29.Match the following P. Isocitrate Lyase 1. Conversion of aminoacid into glucose Q. PEP Carboxykinase 2. Biotin R. Pyruvate dehydrogenase complex 3. Synthesis of glucose from acetate S. PFK

4. Lipoic acid T. Pyruvate carboxylase 5. an allosteric enzyme a. b. c. d.

P-1, Q-2, R-4, S-5, T-3 P-3, Q-1, R-4, S-5, T-2 P-3, Q-1, R-4, S-2, T-5 P-2, Q-5, R-1, S-4, T-3

Muscle cells were incubated in the presence of oxygen and then quickly made anoxic. The concentrations of various metabolites were measured mmediately following the removal of oxygen. The results are shown in figure below

30. The change in glucose-6-phosphate concentration can be explained best by which of the following? a. Increased synthesis of glycogen b. increased conversion to free glucose c. increased rate of glycolysis d. decreased synthesis of glucose-6-phosphate e. acceleration of citric acid cycle 31. The initial increase in concentration of fructose-1,6-bisphosphate is most likely due to a. activation of gluconeogenesis b. activation of PFK c. inhibition of citric acid cycle d. inhibition of aldolase e. increase in concentration of ATP 32. Which of the following is most likely to happen to the concentration of lactate in the cell? a. it will remain the same b. It will increase nitially and then decrease to control values as equilibrium is achieved c. it will increase to a new steady state level

d. it will decrease because the cell secretes the lactate e. It will decrease because cell uses lactate to synthesize glucose 33. Which GLUT gene helps in transport of glucose into liver cells? a. GLUT 1 b. GLUT 2 c. GLUT 4 d. GLUT 5 34.In submerged roots of mangrove plants the recycling of NAD + is carried out by a. Cellular oxidation b. Glycolysis c. ETC d. Fermentative metabolism 35. Incubation of cell extract containing all enzymes of glycolysis with gamma P-32 ATP and unlabelled inorganic phosphate results in the formation of which of the following labelled compounds? I. Glucose -6(32)-phosphate II. 3-(32) P - glycerate III. (1-phospho-3(32) Phospho) - bisphosphoglycerate IV. (1-phospho-6 (32) Phospho) fructobisphosphate a. I and III b. I, II and III c. II and IV d. Only IV 36. Routinely used glucose biosensor esimates blood glucose level by sensing the concentration of a. Glucose b. Oxygen c. delta-gluconolactone d. hydrogen peroxide 37. The Bohr Effect in hemoglobin refers to a. Reduced affinity for oxygen at low pH b. Higher pH in actively metabolizing tissues c. Increased affinity for oxygen at lower pH d. Low pH in actively metabolizing tissues

38. which of the following proteins is not required for the growth of yeast cells in a medium containing glucose as the sole carbon source? a. Phosphoglycerate kinase b. GAPDH c. Hexokinase d. Cytochrome C 39. How many molecules of ATP are consumed and produced from 2 molecules of glucose in glycolysis? a. 2 and 2 b. 2 and 4 c. 2 and eight d. 4 and eight 40. Blood fibrinogen is converted into fibrin during a. Carbon dioxide b. Oxygen transport c. An immune response d. Glucose regulation e. Clot formation 41. The enzyme responsible for conversion of O-2 to H2O2 is a. catalase b. ascorbate c. super oxide dismutase d. peroxidase 42. Formation of glucose from acetyl coA is called a. Glycogenesis b. Gluconeogenesis c. TCA cycle d. Glycolysis 43. Total energy available for work at equilibrium is termed as a. Free energy b. Entropy c. Enthalpy d. Activation energy 44. The conversion of glucose from fats in plants takes place in

a. b. c. d.

Lysosome Plastids Glyoxisomes Peroxisomes

45. Consider the following redox reaction of Glycolysis

The correct statement is a) The electron are readily picked up by NAD from Pyruvate b) NADH2 provide electrons to Pyruvate for oxidation c) Both reaction are independent d) NADH2 is not formed during Glycolysis

46. Glycoconjugates on proteins in intra cellular membranes are oriented toward a. Cytoplasmic face b. Lumen side c. Embedded in membrane d. .On both sides 47. Which of the following bonds will be most difficult to break? a. C-O b. C-C c. C-N d. C-S 48.In muscle tissue, the rate of conversion of glycogen to glucose 6-phosphate is determined by the ratio of phosphorylase a (active) to phosphorylase b (less active). Determine what happens to the rate of glycogen breakdown if a muscle preparation containing glycogen phosphorylase is treated with PP1? a. a.PP1will convert less active glycogen phosphorylase b to active glycogen phosphorylase a b. b.PP1will convert active glycogen phosphorylase a to less active glycogen phosphorylase b c. c.PP! will increase ATP content in cell d. d.PP1 gets inhibited by cAMP 49. The patient complains of painful muscle cramps when performing strenuous physical exercise but has no other symptoms. A muscle biopsy indicates a muscle glycogen concentration much higher than normal. Why does glycogen accumulate? a. Deficiency of glycogen phosphorylase b enzyme b. Deficiency of glycogen phosphorylase a enzyme c. Deficiency of branching enzyme

d. Deficiency of glycogen synthase 50. Which of these cofactors participates directly in most of the oxidation-reduction reactions in the fermentation of glucose to lactate? a. ADP b. ATP c. FAD/FADH2 d. Glyceraldehyde 3-phosphate e. NAD+/NADH 51. The metabolic function of the pentose phosphate pathway is: a. act as a source of ADP biosynthesis. b. generate NADPH and pentoses for the biosynthesis of fatty acids and nucleic acids. c. participate in oxidation-reduction reactions during the formation of H2O. d. provide intermediates for the citric acid cycle. e. synthesize phosphorus pentoxide. 52. In a tissue that metabolizes glucose via the pentose phosphate pathway, C-1 of glucose would be expected to end up principally in: a. carbon dioxide. b. glycogen. c. phosphoglycerate. d. pyruvate. e. ribulose 5-phosphate. 53. Which of the following enzymes acts in the pentose phosphate pathway? a. 6-phosphogluconate dehydrogenase b. Aldolase c. Glycogen phosphorylase d. Phosphofructokinase-1 e. Pyruvate kinase 54. Galactosemia is a genetic error of metabolism associated with: a. deficiency of galactokinase. b. deficiency of UDP-glucose. c. deficiency of UDP-glucose: galactose 1-phosphate uridylyltransferase. d. excessive ingestion of galactose. e. inability to digest lactose. 55. The ultimate electron acceptor in the fermentation of glucose to ethanol is:

a. b. c. d. e.

acetaldehyde. acetate. ethanol. NAD+. pyruvate.

56.Match the following P) Chitin Q) Hemicellulose R) Glycogen 4) D-galactose a. P – 1, Q- 3, R-4 b. P – 2, Q- 4, R – 1 c. P – 4, Q – 2, R – 3 d. P – 2, Q – 3, R – 1

1) D-Glucose 2) N-Acetyl glucosamine 3) D-Xylose

57. Which of the following is a heteroglycan? a. dextrins b. agar c. inulin d. chitin 58. The glycosminoglycan which does not contain uronic acid is a. dermatan sulphate b. chondroitin sulphate c. keratin sulphate d. heparan sulphate 59. The monosaccharides units are linked by 1 --- 4 glycosidic linkage in a. maltose b. sucrose c. cellulose d. cellobiose 60. alpha – D – glucuronic acid is present in a. hyaluronic acid b. chondroitin sulphate c. heparin

d. all of these 61. The predominant form of glucose in solution is a. acyclic form b. hydrated acyclic form c. glucofuranose d. glucopyranose 62. Honey contains the hydrolytic product of a. lactose b. maltose c. inulin d. starch 63. Glycogen is present in all body tissues except a. liver b. brain c. kidney d. stomach 64. Starch and glycogen are polymers of a. fructose b. mannose c. alpha – D – glucose d. galactose 65. A dissacharide formed by 1,1 – glycoside linkage between their monosaccharide units is a. lactose b. maltose c. sucrose d. trehalose 66. Keratan sulphate is found in abundance in a. Heart muscle b. Liver c. Adrenal cortex d. Cornea

67. The conversion of 1 mol of fructose-1,6-bisphosphate to 2 mol of pyruvate by the glycolytic pathway results in a net formation of a. 1 mol of NAD+ and 2 mol of ATP. b. 1 mol of NADH and 1 mol of ATP. c. 2 mol of NAD+ and 4 mol of ATP. d. 2 mol of NADH and 2 mol of ATP. e. 2 mol of NADH and 4 mol of ATP. 68. Which of the following is a cofactor in the reaction catalyzed by glyceraldehyde 3-phosphate dehydrogenase? a. ATP b. Cu2+ c. heme d. NAD+ e. NADP+ 69. Inorganic fluoride inhibits enolase. In an anaerobic system that is metabolizing glucose as a substrate, which of the following compounds would you expect to increase in concentration following the addition of fluoride? a. 2-phosphoglycerate b. Glucose c. Glyoxylate d. Phosphoenolpyruvate e. Pyruvate 70. Plasma is the liquid part of human blood that is like a soup containing several micro and macromolecules suspended in a fluid. What is the most abundant protein in human blood plasma? a. Albumin b. Prostacyclin c. Hemoglobin d. Insulin 71. Insulin is a peptide hormone produced in the pancreas by beta cells. It aims to lower the glucose level in blood and is normally secreted after every meal. Their action is to activate transporters of glucose found on the surface of the cells , so that the cells can take up glucose from blood and into cells. What glucose transporter is regulated by the action of insulin? a. Sodium-Glucose transporter (SGLT) b. Glucose transporter2 (GLUT 2)

c. Glucose transporter 5 (GLUT 5) d. Glucose transporter 4 (GLUT 4) 72.Fiber-rich foods have been an essential part of everyday meals. Research have shown that regular consumption of fiber or fiber-rich food can lower the risk of having colon cancer. Question: What is this fiber made of? a.Chitin b.Cellulose c.Starch d.Dextrin 73.In phase 1 of glycolysis, the appropriate sequence of enzymes is (choose one of the alternative combinations shown below): A phosphofructokinase-1 (PFK-1). B hexokinase / glucokinase. C fructose bisphosphate aldolase. D Phosphoglucoisomerase. E riose phosphate isomerase (TIM). a.A, C, B, E, D b.B, D, A, C, E c.B, D, C, A, E d.B, C, D, E, A e.B, D, E, C, A 74.For phosphofructokinase-1: a.Low ATP stimulates the enzyme, but fructose-2,6-bisphosphate inhibits. b.High ATP stimulates the enzyme, and fructose-2,6-bisphosphate activates. c.High ATP stimulates the enzyme, but fructose-2,6-bisphosphate inhibits. d.The enzyme is more active at low ATP than at high, and fructose-2,6-bisphosphate activates the enzyme. e.ATP and fructose-2,6-bisphosphate both inhibit the enzyme.

75. A typical animal cell membrane contains glycolipids and glycoproteins in the plasma membrane. To determine its topological distribution, lectin is used as a probe. The following interactions may be the basis of the probing method : P. Protein-protein interaction Q. Protein-sugar interaction

R. Protein-lipid interaction S. Protein-sterol interaction The appropriate answer is a. Only P b. Only Q c. All of P,Q and R d. Only S 76. Lipids which ranges from 120-160 KD in size. Then also they are kept amongst the macromolecules becausea. They are main component of membrane b. They are main component of cell c. In water they form large complex structures d. They are present in large amount 77. Epinephrine does not cause increase concentrations of a. Glucose in blood b. Lactate in blood c. Free fatty acids in blood d. cAMP in heart muscle e. triglycerides in fat cells 78. Aspirin, used as a common analgesic, antipyretic and anti-inflammatory agent, inhibits the synthesis of which one of the following? a. Arachidonic acid b. Prostaglandins c. Glucocorticoids d. Histamine 79 The storage disease associated with excess spingomyelin is called a. Niemann Pick b. Sandhoff c. Tay Sachs d. Krabbe's 80. Which of the following is not true of the reaction catalyzed by the pyruvate dehydrogenase complex? a. Biotin participates in the decarboxylation.

b. c. d. e.

Both NAD+and a flavin nucleotide act as electron carriers. The reaction occurs in the mitochondrial matrix. The substrate is held by the lipoyl-lysine “swinging arm.” Two different cofactors containing —SH groups participate.

81, Which of the below is not required for the oxidative decarboxylation of pyruvate to form acetyl-CoA? a. ATP b. CoA-SH c. FAD d. Lipoic acid e. NAD+ 82. Which combination of cofactors is involved in the conversion of pyruvate to acetyl-CoA? a. Biotin, FAD, and TPP b. Biotin, NAD+, and FAD c. NAD+, biotin, and TPP d. Pyridoxal phosphate, FAD, and lipoic acid e. TPP, lipoic acid, and NAD+ 83. Which of the following statements about the oxidative decarboxylation of pyruvate in aerobicconditions in animal cells is correct? a. One of the products of the reactions of the pyruvate dehydrogenase complex is a thioester of acetate. b. The methyl (—CH3) group is eliminated as CO2. c. The process occurs in the cytosolic compartment of the cell. d. The pyruvate dehydrogenase complex uses all of the following as cofactors: NAD+, lipoic acid,pyridoxal phosphate (PLP), and FAD. e. The reaction is so important to energy production that pyruvate dehydrogenase operates at full speed under all conditions. 84.Glucose labeled with 14C in C-3 and C-4 is completely converted to acetyl-CoA via glycolysis and the pyruvate dehydrogenase complex. What percentage of the acetyl-CoA molecules formed will be labeled with 14 C, and in which position of the acetyl moiety will the 14 C label be found? a. 100% of the acetyl-CoA will be labeled at C-1 (carboxyl). b. 100% of the acetyl-CoA will be labeled at C-2. c. 50% of the acetyl-CoA will be labeled, all at C-2 (methyl). d. No label will be found in the acetyl-CoA molecules. e. Not enough information is given to answer this question.

85. Which of the following is not true of the citric acid cycle? a. All enzymes of the cycle are located in the cytoplasm, except succinate dehydrogenase, which is bound to the inner mitochondrial membrane. b. In the presence of malonate, one would expect succinate to accumulate. c. Oxaloacetate is used as a substrate but is not consumed in the cycle. d. Succinate dehydrogenase channels electrons directly into the electron transfer chain. e. The condensing enzyme is subject to allosteric regulation by ATP and NADH. f. Reactions of the citric acid cycle 86. Acetyl-CoA labeled with 14C in both of its acetate carbon atoms is incubated with unlabeled oxaloacetate and a crude tissue preparation capable of carrying out the reactions of the citric acid cycle. After one turn of the cycle, oxaloacetate would have 14C in: a. all four carbon atoms. b. no pattern that is predictable from the information provided. c. none of its carbon atoms. d. the keto carbon and one of the carboxyl carbons. e. the two carboxyl carbons.

87. Malonate is a competitive inhibitor of succinate dehydrogenase. If malonate is added to a mitochondrial preparation that is oxidizing pyruvate as a substrate, which of the following compounds would you expect to decrease in concentration? a. Citrate b. Fumarate c. Isocitrate d. Pyruvate 88. Which of the following is not an intermediate of the citric acid cycle? a. Acetyl-coA b. Citrate c. Oxaloacetate d. Succinyl-coA e. α-Ketoglutarate 89. In mammals, each of the following occurs during the citric acid cycle except: a. formation of α-ketoglutarate.

b. generation of NADH and FADH2. c. metabolism of acetate to carbon dioxide and water. d. net synthesis of oxaloacetate from acetyl-CoA. e. oxidation of acetyl-CoA. 90. Oxaloacetate uniformly labeled with 14C (i.e., with equal amounts of 14C in each of its carbon atoms)is condensed with unlabeled acetyl-CoA. After a single pass through the citric acid cycle back tooxaloacetate, what fraction of the original radioactivity will be found in the oxaloacetate? a. all b. 1/2 c. 1/3 d. 1/4 e. ¾

91. Conversion of 1 mol of acetyl-CoA to 2 mol of CO2 and CoA via the citric acid cycle results in the net production of: a. 1 mol of citrate. b. 1 mol of FADH2. c. 1 mol of NADH. d. 1 mol of oxaloacetate. 92. Which one of the following is not associated with the oxidation of substrates by the citric acid cycle? a. All of the below are involved. b. CO2 production c. Flavin reduction d. Lipoic acid present in some of the enzyme systems e. Pyridine nucleotide oxidation

93. The two moles of CO2 produced in the first turn of the citric acid cycle have their origin in the: a. carboxyl and methylene carbons of oxaloacetate b. carboxyl group of acetate and a carboxyl group of oxaloacetate. c. carboxyl group of acetate and the keto group of oxaloacetate. d. two carbon atoms of acetate. e. two carboxyl groups derived from oxaloacetate.

94. The oxidative decarboxylation of α-ketoglutarate proceeds by means of multistep reactions in which all but one of the following cofactors are required. Which one is not required? a. ATP b. Coenzyme A c. Lipoic acid d. NAD+ e. Thiamine pyrophosphate

95. The reaction of the citric acid cycle that is most similar to the pyruvate dehydrogenase complexcatalyzed conversion of pyruvate to acetyl-CoA is the conversion of: a. citrate to isocitrate. b. fumarate to malate. c. malate to oxaloacetate. d. succinyl-CoA to succinate. e. α-ketoglutarate to succinyl-CoA. 96. Which one of the following enzymatic activities would be decreased by thiamine deficiency? a. Fumarase b. Isocitrate dehydrogenase c. Malate dehydrogenase d. Succinate dehydrogenase e. α-Ketoglutarate dehydrogenase complex 97. The reaction of the citric acid cycle that produces an ATP equivalent (in the form of GTP) by substrate level phosphorylation is the conversion of: a. citrate to isocitrate. b. fumarate to malate. c. malate to oxaloacetate. d. succinate to fumarate. e. succinyl-CoA to succinate. 98. The standard reduction potentials (E'°) for the following half reactions are given.Fumarate + 2H ++ 2e– → succinate E'° = +0.031 V FAD + 2H++ 2e– → FADH2 E'° = –0.219 V If succinate, fumarate, FAD, and FADH 2 , all at l M concentrations, were mixed together in the presence of succinate dehydrogenase, which of the following would happen initially? a. Fumarate and succinate would become oxidized; FAD and FADH2 would become reduced. b. Fumarate would become reduced; FADH2 would become oxidized.

c. No reaction would occur because all reactants and products are already at their standard concentrations. d. Succinate would become oxidized; FAD would become reduced. e. Succinate would become oxidized; FADH2 would be unchanged because it is a cofactor, not a substrate. 99. For the following reaction, ∆G'° = 29.7 kJ/mol. L-Malate + NAD+→ oxaloacetate + NADH + H+The reaction as written: a. can never occur in a cell. b. can only occur in a cell if it is coupled to another reaction for which ∆G'° is positive. c. can only occur in a cell in which NADH is converted to NAD+by electron transport. d. may occur in cells at certain concentrations of substrate and product. e. would always proceed at a very slow rate 100. All of the oxidative steps of the citric acid cycle are linked to the reduction of NAD + except the reaction catalyzed by: a. isocitrate dehydrogenase. b. malate dehydrogenase. c. pyruvate dehydrogenase d. succinate dehydrogenase. e. the α-ketoglutarate dehydrogenase complex. 101. Which of the following cofactors is required for the conversion of succinate to fumarate in the citric acid cycle? a. ATP b. Biotin c. FAD d. NAD + e. NADP + 102. In the citric acid cycle, a flavin coenzyme is required for: a. condensation of acetyl-CoA and oxaloacetate. b. oxidation of fumarate. c. oxidation of isocitrate. d. oxidation of malate. e. oxidation of succinate. 103. Which of the following intermediates of the citric acid cycle is prochiral?

a. b. c. d. e.

Citrate Isocitrate Malate Oxaloacetate Succinate

104. The conversion of 1 mol of pyruvate to 3 mol of CO2 via pyruvate dehydrogenase and the citric acid cycle also yields _____ mol of NADH, _____ mol of FADH2, and _____ mol of ATP (or GTP). a. 2; 2; 2 b. 3; 1; 1 c. 3; 2; 0 d. 4; 1; 1 e. 4; 2; 1 105. Entry of acetyl-CoA into the citric acid cycle is decreased when: [AMP] is high. a. NADH is rapidly oxidized through the respiratory chain. b. the ratio of [ATP]/[ADP is low c. the ratio of [ATP]/[ADP] is high. d. the ratio of [NAD+]/[NADH] is high 106. Citrate synthase and the NAD+-specific isocitrate dehydrogenase are two key regulatory enzymes ofthe citric acid cycle. These enzymes are inhibited by: a. acetyl-CoA and fructose 6-phosphate. b. AMP and/or NAD + c. AMP and/or NADH. d. ATP and/or NAD+ e. ATP and/or NADH. 107. During seed germination, the glyoxylate pathway is important to plants because it enables them to: a. carry out the net synthesis of glucose from acetyl-CoA. b. form acetyl-CoA from malate. c. get rid of isocitrate formed from the aconitase reaction. d. obtain glyoxylate for cholesterol biosynthesis. e. obtain glyoxylate for pyrimidine synthesis.

108. A function of the glyoxylate cycle, in conjunction with the citric acid cycle, is to accomplish the: a. complete oxidation of acetyl-CoA to CO2 plus reduced coenzymes. b. net conversion of lipid to carbohydrate. c. net synthesis of four-carbon dicarboxylic acids from acetyl-CoA. d. net synthesis of long-chain fatty acids from citric acid cycle intermediates. e. both B and C are correct 109. The glyoxylate cycle is: a. a means of using acetate for both energy and biosynthetic precursors. b. an alternative path of glucose metabolism in cells that do not have enough O2. c. defective in people with phenylketonuria. d. is not active in a mammalian liver. e. the most direct way of providing the precursors for synthesis of nucleic acids (e.g., ribose) 110. The citric acid cycle is also known as the a. Krebs cycle. b. Cori cycle. c. tricarboxylic acid cycle. d. a and c. e. a, b, and c. 111. What molecule initiates the citric acid cycle by reacting with oxaloacetate? a. All of the above. b. acetyl CoA c. None of the above. d. Oxaloacetate 112. What enzyme(s) is (are) responsible for the following reaction? Pyruvate + CoA + NAD+ acetyl CoA + NADH + H+ + CO2 a. Aacetyl CoA synthetase b. pyruvate decarboxylase c. pyruvate dehydrogenase complex d. a and b e. a, b, and c 113. What are the steps involved (in order) in the conversion of pyruvate to acetyl CoA? a. decarboxylation, oxidation, transfer to CoA b. decarboxylation, transfer to CoA, oxidation

c. oxidation, decarboxylation, transfer to CoA d. oxidation, transfer to CoA, decarboxylation e. None of the above. 114. Which of the following vitamins are precursors to coenzymes that are necessary for the formation of acetyl CoA from pyruvate? a. thiamine, riboflavin, niacin, lipoic acid, and pantothenic acid b. thiamine, riboflavin, niacin, lipoic acid, pantothenic acid, and biotin c. thiamine, riboflavin, niacin, and biotin d. thiamine, riboflavin, and lipoic acid e. none of the above 115. Which of the following functions as a “flexible swinging arm” when it transfers the reaction intermediate from one active site to the next? a. FAD b. NAD+ c. lipoamide d. thiamine pyrophosphate e. coenzyme A 116. Formation of citrate from acetyl CoA and oxaloacetate is a(n) _________ reaction. a. oxidation b. ligation c. reduction d. None of the above. e. Condensation 117. What is/are the chemical change(s) involved in the conversion of citrate into isocitrate? a. hydration followed by dehydration b. oxidation c. oxidation followed by reduction d. dehydration followed by hydration e. a and b 118. In which reaction is GTP (or ATP) directly formed in the citric acid cycle? a. conversion of succinyl CoA to succinate b. decarboxylation of -ketoglutarate c. conversion of isocitrate to -ketoglutarate

d. All of the above. e. None of the above.

119. In which step of the citric acid cycle is FADH2 formed? a. the conversion of succinate to malate b. the conversion of succinate to oxaloacetate c. the conversion of succinate to fumarate d. the conversion of malate to oxaloacetate e. none of the above 120. Which of the following conditions will activate pyruvate dehydrogenase kinase which catalyzes the phorphorylation and inactivation of E1 in the pyruvate dehydrogenase complex? a. elevated concentrations of NADH and ATP b. elevated concentrations of NAD+ and ADP c. Ca2+ d. insulin e. elevated concentrations of acetyl-CoA 121. In glycolysis, fructose 1,6-bisphosphate is converted to two products with a standard free energy change (∆G'°) of 23.8 kJ/mol. Under what conditions (encountered in a normal cell) will the free-energy change (∆G) be negative, enabling the reaction to proceed to the right? a. If the concentrations of the two products are high relative to that of fructose 1,6bisphosphate. b. The reaction will not go to the right spontaneously under any conditions because the ∆G'° is positive. c. Under standard conditions, enough energy is released to drive the reaction to the right. d. When there is a high concentration of fructose 1,6-bisphosphate relative to the concentration of products. e. When there is a high concentration of products relative to the concentration of fructose 1,6-bisphosphate.

1. A 2. C 3. A 4. B

5. D 6. A 7. B 8. A 9. A 10.

A

11.

C

12.

B

13.

C

14.

C

15.

C

16.

B

17.

B

18.

B

19.

A

20.

D

21.

B

22.

D

23.

C

24.

B

25.

D

26.

A

27.

B

28.

C

29.

B

30.

A

31.

C

32.

B

33.

B

34.

D

35.

A

36.

C

37.

A

38.

D

39.

B

40.

E

41.

A

42.

B

43.

A

44.

D

45.

C

46.

D

47.

C

48.

B

49.

C

50.

E

51.

B

52.

A

53.

A

54.

C

55.

A

56.

D

57.

B

58.

C

59.

A

60.

D

61.

D

62.

B

63.

B

64.

C

65.

B

66.

D

67.

E

68.

D

69.

A

70.

A

71.

D

72.

B

73.

B

74.

D

75.

B

76.

C

77.

E

78.

B

79.

D

80.

A

81.

A

82.

E

83.

A

84.

D

85.

A

86.

A

87.

B

88.

A

89.

D

90.

B

91.

B

92.

E

93.

E

94.

A

95.

E

96.

E

97.

E

98.

B

99.

D

100.

D

101.

C

102.

E

103.

A

104.

D

105.

D

106.

E

107.

A

108.

E

109.

A

110.

D

111.

B

112.

C

113.

A

114.

A

115.

C

116.

E

117.

D

118.

A

119.

C

120.

A

121.

D

Composition Structure and Function of Biomolecules

1. The direction of migration of the tripeptide Lys-Lys-Lys in an electric field at pH = 3 and at pH = 10 is a. Cathode and Cathode respectively b. Cathode and Anode respectively c. Anode and Cathode respectively d. Anode and Anode respectively e. When an actively photosynthesizing plant is exposed to 14CO2 , which 2 carbon atoms in the resulting radioactive glucose will be labeled first? a. b. c. d.

C3 and C4 C3 and C5 C1 and C2 C1 and C6

2. Hydrogen bond is a major intermolecular attraction force in biomolecules. Which of the following statement is false about hydrogen bonds? a. b. c. d.

Hydrogen bonds are weak interactions but strong enough in close encounters. Hydrogen atom, accepts a hydrogen bond Hydrogen atom, donates a hydrogen bond Hydrogen bond is responsible for Water boiling temperature to be 100°C

3. Difference in pH of cell cytoplasm and Nucleus is a. b. c. d.

0 1 7.2 5.5

4. Which is the cholesterol esters that enter cells through the receptor-mediated endocytosis of lipoproteins hydrolyzed? a. Endoplasmin reticulum

b. Lysosomes. c. Plasma membrane receptor d. Mitochondria

5. Oxidation of which substance in the body yields the most calories a. b. c. d.

Glucose Glycogen Protein Lipids

6. Which of the following phospholipids is localized to a greater extent in the outer leaflet of the membrane lipid bilayer? a. b. c. d.

Choline phosphoglycerides Ethanolamine phosphoglycerides Inositol phosphoglycerides Serine phosphoglycerides

7. Which of the following is true about person suffering from Galactosemia? 1. He cannot synthesize Galactose 2. He cannot metabolize the Galactose 3. Suffers from Hepatomegaly 4. It is Autosomal recessive disorder a. b. c. d.

1,2,3,4 2,3,4 1,2 Only 2

8. Orlistat is a drug widely used to control Obesity. Its mode action would be a. By inhibiting gastric and pancreatic lipases b. By inhibiting fatty acids synthesis

c. By increasing allergy for fatty acids d. By inhibiting pepsin and pancreatic insulin.

9. The pI of a protein is 8.3. To carry out ion-exchange chromatography of this protein at pH = 7.0, the following should be used to ensure binding and fractionation: a. b. c. d.

Cation exchanger resin Anion exchanger resin Reactive ion exchanger resin Polyanion exchanger resin

10. Alzheimers’ protein-A multimerizes at pH 2 form aggregates which leads to Alzheimers’ disease. In the brain the pH is always neutral ~7. So what leads to aggregation of A that induces Alzheimers’ in people greater than 60 years of age? a. Older brains are more acidic. b. At pH 7, there will be very fast kinetic equilibrium between oligomers and monomers which can change with age to induce aggregation c. At pH 7, the reaction does proceed to oligomerization but the rate is so low that it takes 60 years to form aggregates d. Aggregates form during intermittent drops pH throughout your life.

11. An SN2 reaction of an asymmetric carbon of a compound always gives a. b. c. d.

An enantiomer of the substrate A product of opposite optical rotation A mixture of diastereomers A single stereoisomer

12. The conversion of Pyruvate to oxaloacetate is likely to require which of the following coenzymes?

a. b. c. d.

Biotin Vitamin B12 TPP Pyridoxal phosphate

13. In an intact cell, free energy change associated with an enzyme catalyzed reaction is frequently different from the standard free energy change of the same reaction because in the intact cell reactants a. b. c. d. e.

are not at 1M concentration reaction activation energy is different reaction is always near equilibrium enzyme may be regulated allosterically reactants may be catalyzed by more than one enzyme

14. Plants and some bacteria differ from animals in that plants and some bacteria can a. b. c. d. e.

form polymers from glucose use carbon dioxide to increase their biomass produce NADH via reductive reactions synthesize glutamate and aspartate Use glucose by the glycolytic pathway.

15. In animals, an enzyme unique to gluconeogenesis is a. b. c. d. e.

enolase phosphoglyceromutase glyceraldehyde-3-phosphate Dehydrogenase aldolase fructose 1,6 – bisphosphatase

16. Which of the following compounds is a positive allosteric regulaor of the enzyme pyruvate craboxylase? a. b. c. d.

ATP acetyl CoA Biotin PEP

17. Suppose a glycogen molecule with 6000 glucose residues. If branches occur every eight residues, how many reducing end does the molecule have? a. b. c. d.

1 500 6000 499

18.Phospho fructo kinase is characterized by which of the following statements? P. it is a major regulatory enzyme in glycolysis Q. ATP is a substrate of the enzyme R. ATP is a negative modulator of enzyme s. Citrate is a posiive modulator of enzyme a. b. c. d.

P,Q P,R P,Q,R P,Q,R,S

19. Conversion of glucose to glucose-6-phosphate requires ATP. Yet, critically ill patients are given glucose solutions intravenously instead of G6P. The reason for not giving G6P directly is a. b. c. d.

G6P is degraded fast in blood before it enters the cells Commercial preparations of G6P are always contaminated with toxic chemicals high cost of G6P cells cannot take up G6P

20. In anaerobic glycolysis, 2 mol of inorganic phosphate are used per mole of glucose consumed. Which of the following enzymes catalyzes the uptake of inorganic phosphate? a. b. c. d.

hexokinase PFK Pyruvate kinase Glyceraldehyde-3-phosphate dehydrogenase

21. GAPDH, an enzyme of glycolytic pathway, was modified with iodoacetic acid that resulted in the loss of enzyme activity. This is due to modification of active site residue a. b. c. d.

Lysine Glutamic acd Cysteine Cystine

22. Which GLUT gene helps in transport of glucose into liver cells? a. b. c. d.

GLUT 1 GLUT 2 GLUT 4 GLUT 5

23. Routinely used glucose biosensor esimates blood glucose level by sensing the concentration of a. b. c. d.

Glucose Oxygen delta-gluconolactone hydrogen peroxide

24. The Bohr Effect in hemoglobin refers to a. Reduced affinity for oxygen at low pH b. Higher pH in actively metabolizing tissues c. Increased affinity for oxygen at lower pH

d. Low pH in actively metabolizing tissues

25. which of the following proteins is not required for the growth of yeast cells in a medium containing glucose as the sole carbon source? a. b. c. d.

Phosphoglycerate kinase GAPDH Hexokinase Cytochrome C

26. Which one of the following sequences in proteins corresponds to N-glycosylation site? a. b. c. d.

Asn-Ser/Thr Asn-X-Ser/Thr Asn-X-X-Ser/Thr Ser/Thr-Asn

27. Which one of the following replacements of an amino acid by another amino acid actually represents substitution of an atom by another atom? a. b. c. d.

Ser Gly Gly Asp

cys Phe ala Asn

28. Which pair of amino acid will have the highest absorbance at 280 nm? (assume equimolar concentrations) a. b. c. d.

Thr and His Phe and Pro Trp and Tyr Phe and His

29. The following amino acid has a nonpolar and aliphatic R group a. b. c. d.

Tyrosine Histidine Glutamate Leucine

30. How many molecules of ATP are consumed and produced from 2 molecules of glucose in glycolysis? a. b. c. d.

2 and 2 2 and 4 2 and eight 4 and eight

31. Blood fibrinogen is converted into fibrin during a. b. c. d. e.

Carbon dioxide Oxygen transport An immune response Glucose regulation Clot formation

32. The enzyme responsible for conversion of O-2 to H2O2 is a. b. c. d.

catalase ascorbate super oxide dismutase peroxidase

33. Formation of glucose from acetyl coA is called a. Glycogenesis b. Gluconeogenesis c. TCA cycle

d. Glycolysis

34. Total energy available for work at equilibrium is termed as a. b. c. d.

Free energy Entropy Enthalpy Activation energy

35. The conversion of glucose from fats in plants takes place in a. b. c. d.

Lysosome Plastids Glyoxisomes Peroxisomes

36. Consider the following redox reaction of Glycolysis The correct statement is a. b. c. d.

The electron are readily picked up by NAD from Pyruvate NADH2 provide electrons to Pyruvate for oxidation Both reaction are independent NADH2 is not formed during Glycolysis

37. Glycoconjugates on proteins in intra cellular membranes are oriented toward a. b. c. d.

Cytoplasmic face Lumen side Embedded in membrane On both sides

38. How many different disaccharides contaning D-galacto pyranose plus d-glucopyranose are possible?

a. b. c. d.

25 100 20 15

39. Which of the following bonds will be most difficult to break? a. b. c. d.

C-O C-C C-N C-S

40. In muscle tissue, the rate of conversion of glycogen to glucose 6-phosphate is determined by the ratio of phosphorylase a (active) to phosphorylase b (less active). Determine what happens to the rate of glycogen breakdown if a muscle preparation containing glycogen phosphorylase is treated with PP1? a. b. c. d.

PP1will convert less active glycogen phosphorylase b to active glycogen phosphorylase a PP1will convert active glycogen phosphorylase a to less active glycogen phosphorylase b PP1 will increase ATP content in cell PP1 gets inhibited by cAMP

41. The patient complains of painful muscle cramps when performing strenuous physical exercise but has no other symptoms. A muscle biopsy indicates a muscle glycogen concentration much higher than normal. Why does glycogen accumulate? a. b. c. d.

Deficiency of glycogen phosphorylase b enzyme Deficiency of glycogen phosphorylase a enzyme Deficiency of branching enzyme Deficiency of glycogen synthase

42. Which of these cofactors participates directly in most of the oxidation-reduction reactions in the fermentation of glucose to lactate? a. ADP

b. c. d. e.

ATP FAD/FADH2 Glyceraldehyde 3-phosphate NAD+/NADH

43. The metabolic function of the pentose phosphate pathway is: a. b. c. d. e.

act as a source of ADP biosynthesis. generate NADPH and pentoses for the biosynthesis of fatty acids and nucleic acids. participate in oxidation-reduction reactions during the formation of H2O. provide intermediates for the citric acid cycle. synthesize phosphorus pentoxide.

44. Yeast can metabolize D-mannose to ethanol and CO2. In addition to the glycolytic enzymes, the only other enzyme needed is phosphomannose isomerase, which converts mannose 6phosphate to fructose6-phosphate. If mannose is converted to ethanol and CO2 by the most direct pathway, which of the compounds and cofactors in this list are involved? 1) Lactate 2) Acetaldehyde 3) Acetyl-CoA 4) FAD 5) Glucose 6-phosphate 6) Fructose 1-phosphate 7) Pyruvate 8) Lipoic acid 9) Thiamine pyrophosphate 10) Dihydroxyacetone phosphate a. 1) & 7) b. 2)& 7) c. 6) &10)

d. 4) & 8)

45. In a tissue that metabolizes glucose via the pentose phosphate pathway, C-1 of glucose would be expected to end up principally in: a. b. c. d. e.

carbon dioxide. glycogen. phosphoglycerate. pyruvate. ribulose 5-phosphate.

46. Which of the following enzymes acts in the pentose phosphate pathway? a. b. c. d. e.

6-phosphogluconate dehydrogenase Aldolase Glycogen phosphorylase Phosphofructokinase-1 Pyruvate kinase

47. Galactosemia is a genetic error of metabolism associated with: a. b. c. d. e.

deficiency of galactokinase. deficiency of UDP-glucose. deficiency of UDP-glucose: galactose 1-phosphate uridylyltransferase. excessive ingestion of galactose. inability to digest lactose.

48. The ultimate electron acceptor in the fermentation of glucose to ethanol is: a. b. c. d. e.

acetaldehyde. acetate. ethanol. NAD+. Pyruvate

49. Match the following

P) Chitin

1) D-Glucose

Q) Hemicellulose

2) N-Acetyl glucosamine

R) Glycogen

3) D-Xylose 4) D-galactose

a. b. c. d.

P – 1, Q- 3, R-4 P – 2, Q- 4, R – 1 P – 4, Q – 2, R – 3 P – 2, Q – 3, R – 1

50. Which of the following is a heteroglycan? a. b. c. d.

dextrins agar inulin chitin

51. The glycosminoglycan which does not contain uronic acid is a. b. c. d.

dermatan sulphate chondroitin sulphate keratin sulphate heparan sulphate

52. The monosaccharides units are linked by 1 --- 4 glycosidic linkage in a. maltose b. sucrose

c. cellulose d. cellobiose

53. alpha – D – glucuronic acid is present in a. b. c. d.

hyaluronic acid chondroitin sulphate heparin none of these

54. The predominant form of glucose in solution is a. b. c. d.

acyclic form hydrated acyclic form glucofuranose glucopyranose

55. Honey contains the hydrolytic product of a. b. c. d.

lactose maltose inulin starch

56. Glycogen is present in all body tissues except a. b. c. d.

liver brain kidney stomach

57. Starch and glycogen are polymers of a. fructose

b. mannose c. alpha – D – glucose d. galactose

58. A dissacharide formed by 1,1 – glycoside linkage between their monosaccharide units is a. b. c. d.

lactose maltose sucrose trehalose

59. Keratan sulphate is found in abundance in a. Heart muscle b. Liver c. Adrenal cortex d. Cornea

60. The conversion of 1 mol of fructose-1,6-bisphosphate to 2 mol of pyruvate by the glycolytic pathway results in a net formation of a. 1 mol of NAD+ and 2 mol of ATP. b. 1 mol of NADH and 1 mol of ATP. c. 2 mol of NAD+ and 4 mol of ATP. d. 2 mol of NADH and 2 mol of ATP. e. 2 mol of NADH and 4 mol of ATP.

61. Which of the following is a cofactor in the reaction catalyzed by glyceraldehyde 3-phosphate dehydrogenase?

a. b. c. d. e.

ATP Cu2+ heme NAD+ NADP+

62. Inorganic fluoride inhibits enolase. In an anaerobic system that is metabolizing glucose as a substrate, which of the following compounds would you expect to increase in concentration following the addition of fluoride? a. 2-phosphoglycerate b. Glucose c. Glyoxylate d. Phosphoenolpyruvate e. Pyruvate

63. Plasma is the liquid part of human blood that is like a soup containing several micro and macromolecules suspended in a fluid. What is the most abundant protein in human blood plasma? a. Albumin b. Prostacyclin c. Hemoglobin d. Insulin 64. Insulin is a peptide hormone produced in the pancreas by beta cells. It aims to lower the glucose level in blood and is normally secreted after every meal. Their action is to activate transporters of glucose found on the surface of the cells , so that the cells can take up glucose from blood and into cells. What glucose transporter is regulated by the action of insulin? a. b. c. d.

Sodium-Glucose transporter (SGLT) Glucose transporter2 (GLUT 2) Glucose transporter 5 (GLUT 5) Glucose transporter 4 (GLUT 4)

65. Fiber-rich foods have been an essential part of everyday meals. Research have shown that regular consumption of fiber or fiber-rich food can lower the risk of having colon cancer. Question: What is this fiber made of? a. b. c. d.

Chitin Cellulose Starch Dextrin

66. Why can you digest maltose but not cellobiose? They are both two glucoses held together by a (1à4) glycosidic bond. a. b. c. d.

Enzyme catalyzing β (1-4) linkage found in cellobiose is there. Cellobiose is tetrasaccharide. Cellobiose has α (1-4) linkage. Cellobiose is a polysaccharide

67. In phase 1 of glycolysis, the appropriate sequence of enzymes is (choose one of the alternative combinations shown below): A. phosphofructokinase-1 (PFK-1). B. hexokinase / glucokinase. C. fructose bisphosphate aldolase. D. Phosphoglucoisomerase. E. ribose phosphate isomerase (TIM). a. b. c. d. e.

A, C, B, E, D B, D, A, C, E B, D, C, A, E B, C, D, E, A B, D, E, C, A

68. For phosphofructokinase-1: a. Low ATP stimulates the enzyme, but fructose-2,6-bisphosphate inhibits.

b. High ATP stimulates the enzyme, and fructose-2,6-bisphosphate activates. c. High ATP stimulates the enzyme, but fructose-2,6-bisphosphate inhibits. d. The enzyme is more active at low ATP than at high, and fructose-2,6-bisphosphate activates the enzyme. e. ATP and fructose-2,6-bisphosphate both inhibit the enzyme.

69. A typical animal cell membrane contains glycolipids and glycoproteins in the plasma membrane. To determine its topological distribution, lectin is used as a probe. The following interactions may be the basis of the probing method : p. Protein-protein interaction q. Protein-sugar interaction r. Protein-lipid interaction s. Protein-sterol interaction The appropriate answer is a. Only P b. Only Q c. All of P,Q and R d. Only S

70. In most dicot seeds, which of the following structures is responsible for the storage of carbohydrates? a. b. c. d. e.

Seed coat Cotyledon Radicle Plumule Embryo

71. Among the following amino acids which can act as buffer against addition of acid a. b. c. d.

Glycine Histidine Arginine Phenylalanine

72. Pepsin which digest protein do not digest the cells of intestine because a. b. c. d.

Intestine cells do not have proteins Half life of pepsin is very low Pepsin acts only in acidic pH Pepsin do not digest intestine proteins

73. If all the 8 cysteine residues in a protein take part in making disulfide bonds, then the possible number of ways the four S-S bonds can be formed is a. b. c. d.

8C2 8P2 7x5~3~1 4~3x2

74. Which pair of amino acids represents the weakest interaction between their side chains? a. b. c. d.

Ser . . . Leu Asn . . . Thr Tyr . . . Trp Lys ... Phe

75. Identify the amino acid which can exist as a diastereoisomer a. b. c. d.

Val Pro Gly Ile

76. In a peptide unit Cα refers to the central carbon atom in an amino acid and C refers to the carbon in the carbonyl group. Which covalent bond is the most difficult to rotate in the peptide unit? a. Cα-C b. Cα-N

c. N-H d. N-C

77. A protein segment consists of a stretch of 10 residues that adopt an α - helical secondary structure conformation. What is the maximum number of hydrogen bonds that can stabilize this α - helix? a. b. c. d.

4 5 6 7

78. Salt bridges formed in proteins involve a. b. c. d.

SuIfur atoms in cysteine side chains Metal ions such as Na and K Opposite charged side chains ProximaI aromatic and aliphatic side chains

79. The quaternary structure of human hemoglobin is best described as a a. b. c. d.

Dimer of two myoglobin dimers Tetramer of identical subunits Tetramer of four different subunits Tetramer of two different subunits

80. A favourable charge interaction between R groups in an alpha helix is expected to occur when the interacting side chains are separated by a. b. c. d.

One two residues Three four residues Five six residues Seven eight residues

81. Pernicious anemia is due to a. b. c. d.

Blockage of vitamin B12 absorption Blockage of vitamin A absorption Deficiency of vitamin C Deficiency of vitamin B2

82. Lipids which ranges from 120-160 KD in size. Then also they are kept amongst the macromolecules becausea. b. c. d.

They are main component of membrane They are main component of cell In water they form large complex structures They are present in large amount

83. Epinephrine does not cause increase concentrations of a. b. c. d.

Glucose in blood Lactate in blood Free fatty acids in blood cAMP in heart muscle

84. Aspirin, used as a common analgesic, antipyretic and anti-inflammatory agent, inhibits the synthesis of which one of the following? a. b. c. d.

Arachidonic acid Prostaglandins Glucocorticoids Histamine

85. The storage disease associated with excess sphingomyelin is called a. Niemann Pick b. Sandhoff

c. Tay Sachs d. Krabbe's

86. A 15-year-old black male patient complains that lately, after the ingestion of dairy products, he experiences bloating, cramps and flatulence, and sometimes diarrhea. With this information, it is reasonable to think that the patient is intolerant to: a. b. c. d.

a.Cellobiose b.Sucrose c.Lactose d.Isomaltose

87. Human beings don’t have the enzymes necessary for the hydrolysis of the beta 1, 4-Oglycosidic linkages between molecules of glucose. That is why we can not digest this compound and it is part of some laxatives. a. b. c. d.

Glycogen Amylose Cellulose Amylopectin

88. This heteropolysaccharide has multiple uses in Medicine that include its use in blood transfusions to prevent the blood from coagulating before administration, as anticoagulant therapy in prophylaxis and treatment of venous thrombosis and its extension, in pulmonary embolisms and in other similar situations: a. b. c. d.

Hyaluronic acid Chondroitin-4-sulfate Chondroitin-6-sulfate Heparin

89. Which of the following sugars are reducing sugars? a.

b.

c.

a. b. c. d.

B&C ONLY B A& C ONLYC

90. Which of these compounds is unable to undergo mutarotation? a. b. c. d.

Glucose Maltose Lactose Sucrose

91. With respect to galactose, glucose isa. b. c. d.

A stereoisomer A structural isomer Not an isomer Unrelated except that they are both sugars

92. Which of the following is not a disaccharide? a. b. c. d.

Sucrose Lactose Amylose All of the above are disaccharides

93.Milk is deficient in which vitamins? a. b. c. d.

a.Vitamin C b.Vitamin A c.Vitamin B2 d.Vitamin K

94. Milk is deficient of which mineral? a. b. c. d.

Phosphorus Sodium Iron Potassium

95. Synthesis of prostaglandins is inhibited by a. b. c. d.

Aspirin Arsenic Fluoride Cyanide

96. Inulin is a polymer of a. b. c. d.

Glucose Fructose Mannose Both a and b.

97. The enzyme diastase acts on a. b. c. d.

Glycogen Starch Cellulose Chitin

98. The number of stereo isomers exhibited by fructose are a. b. c. d.

4 8 16 32

99. Reduction of glucose leads to the formation of a. b. c. d.

5- hydroxyl methyl furfural Gluconic acid Glucitol Glucoside

100. Among the following which is a wrong statement a. b. c. d.

Sucrose is a disaccharide Sucrose is a reducing sugar Sucrose can be digested by invertase Fructose is a component of sucrose

101. Chitin is a polymer of a. b. c. d.

N-acetyl D-glucosamine with α-(1→4) linkage N-acetyl D-glucosamine with β-(1→4) linkage N-acetyl D-galactosamine with α-(1→4) linkage N-acetyl D-galactosamine with β-(1→4) linkage

102. The carbon atoms involved in osazone formation are a. b. c. d.

C1 and C2 C1 and C3 C2 and C4 C2 and C3

103. The number of asymmetric carbon atoms present in the erythrulose a. b. c. d.

1 2 3 4

104. Which of the following is an optically inactive carbohydrate a. b. c. d.

Glycine Dihydroxy acetone Glyceraldehydes All of these

105. Cellulose is a polymer of a. b. c. d.

α-D-Glucose β-D-Glucose α-D-Fructose β-D-Fructose

106. The fundamental cause of optical activity is due to a. b. c. d.

Presence of four different groups attached to a central carbon atom Presence of helical symmetry Non super imposability of the molecule None of the above

107. α-D-Glucose and β-D-glucose are related to a. b. c. d.

Epimers Anomers Enantiomers Cis-trans isomers

108. The stable ring formation of glucose involves a. b. c. d.

C1 and C4 C2 and C5 C1 and C5 C2 and C1

109. Invert sugar is a. b. c. d.

Maltose Inulin Glucose Hydrolytic product of sucrose

110. The repeating unit of hyaluronic acid is a. b. c. d.

Glucuronic acid and N-acetyl D-galactosamine Galacturonic acid and N-acetyl D-galactosamine Galacturonic acid and N-acetyl D-glucosamine Glucuronic acid and N-acetyl D-glucosamine

111. Ninhydrin does not form Ruhemann's purple with which of the following? a. b. c. d.

Histidine Serine Glutamine Proline

112. The peptide bond is planar a. b. c. d.

due to restriction caused by rotation around Cα –N due to restriction around Cα-C' due to delocalization of the lone pair of electrons of the nitrogen onto carbonyl oxygen because amide proton and carbonyl oxygen are involved in hydrogen bonding.

113. A Ramachandran plot shows a. The amino acid residues that have the greatest degrees of rotational freedom. b. The stearically allowed rotational angles between the side chain groups in a peptide and the peptide backbone. c. The sterically limited rotational angles (domains) where phi and psi are allowed in the protein backbone. d. The angles that are allowed about the bonds connecting the amide nitrogen in a peptide bond 114. The net charge on the peptide N-D-H-E-A-R at pH- 11 is ------and at pH-1 is -----a. b. c. d.

-2 and +3 -2 and -3 +2 and -3 +2 and +3

115. A strong peptide bond is present in arginine and aspartic acid. This bond will be weak if arginine is being replaced by a. b. c. d.

Glutamic acid Lysine Histidine Proline

116. Calculate the pK1 value of a negatively charged amino acid which becomes immobile in an electric field at pH-5 and the carboxylic gp in the side chain gets deprotonated at pH 7 a. b. c. d.

1 2 3 4

117. Fluoride is added during collection of blood sample for glucose estimation because it inhibits a. b. c. d.

glucose-6-phosphate Dehydrogenase enolase hexokinase glucokinase

118. The mean molecular weight of an amino acid in a typical globular protein is a. b. c. d.

70 110 150 90

119. If the molecular mass of an amino acid is 150 Daltons, the molecular mass of a tripeptide will be a. b. c. d.

450 486 504 414

120. The closest estimate for the number of amino acid residues in a protein with a molecular weight of 85 kDa is a. b. c. d.

710 7100 8500 85

121. If one arginine has a molecular weight of 174 Daltons, then what would be the molecular weight (in Daltons) of a circular polymer of 38 arginines? a. b. c. d.

6612 5928 5946 7278

122. The molecular formula for glycine is C2H5O2N. What would be the molecular formula for an linear oligomer made by linking ten glycine molecules together by condensation synthesis? a. b. c. d.

C20H50O20N10 C20H32O11N10 C20H40O10N10 C20H68O29N10

123. A protein X was fused with DFP in a vector and expressed in E.coli. The length of the protein X is 1000 amino acids and the molecular weight of DFP is 27 kilodaltons. What is the total approximate weight of the fusion proteins in Daltons? a. b. c. d.

137000 138000 83000 27000

124. An amino acid has 3 ionizable groups with pKa’s of 2.0,10.5 and 3.8, what is the pI of this amino acid? a. b. c. d.

6.25 9.05 5.43 2.9

125. At isoelectric point, net mobility of an amino acid is a. b. c. d.

Towards cathode Towards anode Zero None

126. Which of the following statement is not true? a. b. c. d.

The pI is the pH value at which a protein has overall charge of +1 At a pH value equal to pI, a protein will not move in the electric field An acidic protein will have a pI less than 7 A basic protein will have a pI greater than 7

127. Which of the following ionic species of glutamic acid would be prevalent at pH 10? a. b. c. d.

HOOC – CH – (CH2)2 – COOH OOC – CH – (CH2)2 – COOH OOC – CH – (CH2)2 – COO HOOC – CH – (CH2)2 – COO

128. Choose the incorrect statement? a. b. c. d.

Aspartate has the smallest pI value Argenine has the largest pI value pKa value of side chain is maximum for aspartate side chain of isoleucine has maximum hydropathy value

129. which of the following is not true about alanine a. b. c. d.

at pH 1, the overall charge is +1 at pH 1, it will move towards cathode an equimolar mixture of D and L alanine does not rotate the plane polarized light contains branched side chain

130. At the isoelectric point of an enzyme, which is 5, it was observed that there are 10 positively charged groups and 10 negatively charged groups. When the enzymes were titrated with alkali starting from pH 5 to give pH 7, 3 positively charged groups were deprotonated. The net charge on the protein at pH 7 would be a. b. c. d.

+2 -3 -7 -10

131. Imagine you are generating a site-directed mutant of a protein in which a given residue is replaced by another residue. Which one of the following substitutions will result in the highest isoelectric point of the mutant compared to the wild type? (X Y means residue X in the wild type is replaced by residue Y in the mutant) a. b. c. d.

Asp Arg Arg -Asp Tyr Lys Gly Arg

132. Which of the following amino acids can exist as diasteriomers? a. b. c. d.

Isoleucine and leucine Isoleucine and valine Threonine and serine Isoleucine and threonine

133. Which of the following amino acid pairs have two chiral centers? a. b. c. d.

Proline and argenine Leucine and isoleucine Isoleucine and threonine Methionine and cysteine

134. Which amino acid residue is most likely to be found in the interior of a water soluble globular protein? a. b. c. d.

Ser Arg Val Asp

135. Non-polar amino acid residues are found mostly a. b. c. d.

In the core of proteins On the surface of proteins On alpha helices In non specific region

136. From the following sets, pick the one that contains exclusively hydrophobic amino acids a. b. c. d.

Asp, Glu, Lys, Arg, Ser Arg, Phe, Tyr, Trp, Asn Ala, Ile, Leu, Phe, Val Ala, Arg, Phe, Leu, Tyr

137. Which of the sequence below would be most likely termed homologous to the following : Leu-Arg-Cys-Lys-Gln-Gly-Ser a. b. c. d.

Ser-Gly-Gln-Lys-Sys-Arg-Leu Ile-Arg-Cys-Arg-Gln-Gly-Thr Val-Lys-Ala-Arg-Asp-His-Thr Leu-Cys-Arg-Gln-Lys-Ser-Gly

138. Which of the following peptides can be easily detected by absorbance at 280 nm? 1. Leu – Tyr – Met – Ala – Glu 2. Ser – Thr – Trp – Val – Ile – Leu

3. Asp – Ala – Glu – Gln – Ser – Asp – Lys 4. Thr – Tyr – Trp – Val - Ile a. b. c. d.

1,2 and 4 1 and 4 2 and 3 2 and 4

139. Of the groups of amino acids indicated below, which one group of amino acids can be phosphorylated? a. b. c. d.

Thr, Tyr, Asn Ser, Thr, Tyr Ser, Phe, Asn Ser, Thr, Phe

140. In cell-free lysates of E.coli, radioactive isotope, [ -32] ATP can be used to label the following a. b. c. d. e.

Closed circular duplex DNA Protein containing serine, threonine or tyrosine Protein devoid of serine, threonine and tyrosine Peptidoglycan Choose the mismatch pair

141. Match the following Amino acids A. Tyrosine B. Arginine C. Histidine D. Cysteine 1. A-1; B-3; C-2; D-4 2. A-2; B-3; C-2; D-1 3. A-1; B-4; C-2; D-3 4. A-1; B-3; C-4; D-2

Group present on side chain 1. Phenolic group 2. Guanidine group 3. Imidazole group 4. thiol group

142. Presence of specific functional group in an amino acid gives a specific color reaction and used as a test for that amino acid. Choose the incorrect combinations Name of test A. Hopkins cole

B. Pauly C. Nitroprusside D. Millon a. A-2;B-1; C-4; D-3 b. A-4;B-1; C-2; D-3 c. A-2;B-3; C-4; D-1 d. A-2;B-4; C-1; D-

Amino acid 1. 2. 3. 4.

Histidine Tryptophan Arginine Cysteine

143. The high solubility of amino acids in water is due to 1. 2. 3. 4.

Presence of side chain Dipolar ion structure Unipolarity Hydrophilic nature of amino group

144.Amino acids found in proteins that are formed by post-translational modification of one of the common amino acids (derived amino acids) include which of the following? 1. 2. 3. 4.

a.Isoleucine b.- carboxyglutamate c.Threonine d.4-hydroxyproline

145. Choose the incorrect statement a. b. c. d.

At isoelectric point, protein has minimum solubility Among the standard amino acids, tryptophan has highest molecular weight c. Selenocysteine is derived from cysteine Histidine contains three ionizable groups.

146. Three properties of amino acid are given. Study the structure and mark one amino acid which has all the three properties A. Contain an imino group B. With ninhydrin formed a yellow derivative C. Is a non essential amino acid a. b. c. d.

Glutamic acid Proline Lysine Histidine

147. Which of the following substitutions, introduced by site-directed mutagenesis, in a protein, is expected to bring about the maximum change in the isoelectric point in the mutant protein compared to the wild-type protein? a. b. c. d.

Asparagines to serine Lysine to argenine Arginine to glutamate Glutamate to aspartate

148. Which statement about amino acid is correct? a. They always have at least one amino group and at least one carboxyl group b. In the formation of proteins, it is a condensation reaction that links the amino group of one amino acid to the variable side chain of the adjacent amino acid c. The variable side chains of all of the amino acids are highly reactive and carry a charge at neutral pH d. The peptide bond that links amino acids together in a protein is a type on ionic bond, which explains why proteins are unstable at high temperatures

149. The peptide bond has a backbone of atoms in which of the following sequences? a. C – N – N – C b. C – C – C – N

c. C – C – N – C d. N – C – C – C

150. The peptide bond is not a. b. c. d.

Partially double bonded Polar Planar Trans in proteins

151. In peptide bond, resonance energy stabilization contributes to a. b. c. d.

Coplanarity Trans alignment Polarity Repulsion of electron pair

152. The property of resonance as applied to protein structure is responsible for a. b. c. d.

The prevention of rotation about the alpha carbon The partial double bond character of the side chain-alpha carbon bond The planar nature of the peptide bond The ability of cochlear cells to detect sound waves by mechanotransduction

153. The peptide bond in protein is a. b. c. d.

Planar, but rotates to three preferred dihedral angles Non planar, but rotates to three preferred dihedral angles Nonpolar, and fixed in a trans conformation Planar, and usually found in a trans configuration

154. Which one of the following bonds in proteins has a partial double bond character? a. Cα - C

b. Cα - N c. C - N d. C – O

155. What should be the net charge at pH= 1 on the tetrapeptide : Glu – Gly – Ala – Lys a. b. c. d.

+1 +2 -1 Zero

156. Aspartame, the synthetic sweetner is a. b. c. d.

A dipeptide Also secreted by the posterior pituitary gland in response to thirst A glucose derivative Not digested (i.e. hydrolyzed) by humans

157. For the pentapertides, Glu-Met-Arg-Thr-Gly a. b. c. d.

Number of charged groups at pH7 is 4 The net charge at pH 1 is +1 The sequence using one letter symbol is E-T-R-M-G The net charge at pH 1 is -2

158. Which of the following statements concerning fatty acids is correct? a. b. c. d. e.

One is the precursor of prostaglandins. Phosphatidic acid is a common one. They all contain one or more double bonds. They are a constituent of sterols. They are strongly hydrophilic.

159. Which of the following molecules or substances contain, or are derived from, fatty acids? a. b. c. d. e.

Beeswax Prostaglandins Sphingolipids Triacylglycerols All of the above contain or are derived from fatty acids.

160. Which of the following statements is true of lipids? a. b. c. d. e.

Many contain fatty acids in ester or amide linkage Most are simply polymers of isoprene. Testosterone is an important sphingolipid found in myelin. They are more soluble in water than in chloroform. They play only passive roles as energy-storage molecules.

161. Which of the following contains an ether-linked alkyl group? a. b. c. d. e.

Cerebrosides Gangliosides Phosphatidyl serine Platelet-activating factor Sphingomyelin

162. Sphingosine is not a component of: a. b. c. d. e.

cardiolipin. ceramide. cerebrosides. gangliosides. sphingomyelin.

163. Which of the following statements about membrane lipids is true? a. Glycerophospholipids are found only in the membranes of plant cells.

b. Glycerophospholipids contain fatty acids linked to glycerol through amide bonds. c. Lecithin (phosphatidylcholine), which is used as an emulsifier in margarine and chocolate, is a sphingolipid. d. Some sphingolipids include oligosaccharides in their structure. e. Triacylglycerols are the principal components of erythrocyte membranes.

164. Which of the following is true of sphingolipids? a. b. c. d. e.

Cerebrosides and gangliosides are sphingolipids. Phosphatidylcholine is a typical sphingolipid. They always contain glycerol and fatty acids. They contain two esterified fatty acids. They may be charged, but are never amphipathic.

165. Compound containing N-acetylneuraminic acid (sialic acid) is: a. b. c. d. e.

cardiolipin. ganglioside GM2. phosphatidylcholine. platelet-activating factor. sphingomyelin.

166. Fatty acids are component of: a. b. c. d. e.

carotenes. cerebrosides. sterols. vitamin D vitamin K.

167. Which of the following statements about sterols is true? a. b. c. d.

All sterols share a fused-ring structure with four rings. Sterols are found in the membranes of all living cells. Sterols are soluble in water, but less so in organic solvents such as chloroform. Stigmasterol is the principal sterol in fungi.

e. The principal sterol of animal cells is ergosterol.

168. Which of the following is not true of sterols? a. Cholesterol is a sterol that is commonly found in mammals. b. They are commonly found in bacterial membranes. c. They are more common in plasma membranes than in intracellular membranes (mitochondria, lysosomes, etc.). d. They are precursors of steroid hormones. e. They have a structure that includes four fused rings.

169. Which of the following best describes the cholesterol molecule? a. b. c. d. e.

Amphipathic Nonpolar, charged Nonpolar, uncharged Polar, charged Polar, uncharged

170. Tay-Sachs disease is the result of a genetic defect in the metabolism of: a. b. c. d.

gangliosides. phosphatidyl ethanolamine. sterols.triacylglycerols vitamin D.

171. An example of a glycerophospholipid that is involved in cell signaling is: a. b. c. d. e.

arachidonic acid. ceramide. phosphatidylinositol. testosterone. vitamin A (retinol).

172. Which of the following is not a fat soluble vitamin? a. b. c. d. e.

A C D E K

173. Which vitamin is derived from cholesterol? a. A b. B12 c. Vitamin D

174. Which of the following statement is false? a. b. c. d.

Amino acids tend to be least soluble in water at their isoelectric point Sickle cell Hb and normal Hb have same value of negative charge Amino acids are made visible on the chromatogram by treatment with ninhydrin The net charge on an amino acid is a function of the pH of the solution

175. Which one of the following peptides lacks a chiral center at the first position, is devoid of hydrogen in the backbone amide at the second position and the side- chain of the third residue has a pK close to 5the physiological pH?

a. b. c. d.

Gly-Pro-Tyr Pro-Gly-Tyr Pro-Gly-His Gly-Pro-His

176. If the genetic code consisted of four bases per codon rather than three, the maximum number of unique amino acids that could be encoded would be a. 16

b. c. d. e.

64 128 256 512

177. The nucleoside adenosine exists in a protonated form with a pKa of 3.8. The percentage of the protonated form at pH 4.8 is closest to a. b. c. d. e.

1 9 50 91 99

178. The distance between two base pairs in Watson-Crick B-DNA is a. 34nm b. 3.4nm c. 0.34nm

179. In a stretch of D N A , 10 % of guanines form non-Watson and Crick base pairing with adenine. If the segment contains 29% guanine, the amount of thymine would be a. b. c. d.

21% 20% 29% 18%

180. In which of the secondary structures, two residues well separated (more than 10 residues) in the amino acid sequence can be proximal in the three- dimensional structure? a. b. c. d.

Alpha-helix beta-sheet 3D-helix Beta turn

181. Which compound can serve as a direct acceptor of an additional amino group derived from amino acid catabolism? a. b. c. d.

Glutamine Fumarate Alpha-ketoglutarate Glycerol

182. Which one of the following has least solubility in water? a. b. c. d.

Lysozyme Colagenase DNA polymerase Rhodopsin

183. The direction of migration of the tripeptide Lys-Lys-Lys in an electric field at pH = 3 and at pH = 10 is a. b. c. d.

Ca thode and Cathode respectively Caihode and Anode respectively Anode and Cathode respeciively Anode and Anode respectively

184. The quaternary structure of human hemoglobin is best described as a a. b. c. d.

Dimer of two myoglobin dimers Tetramer of identical subunits Tetramer of four different subunits Tetramer of two different subunits

185. A favourable charge charge interaction between R groups in an alpha helix is expected to occur when the interacting side chains are separated by a. One two residues

b. Three four residues c. Five six residues d. Seven eight residues

186. Difference between the R and T states of haemoglobin is a. b. c. d.

Dimeric and tetrameric structures Oxy and deoxy states Binding to ferrous and ferric ions Folded and unfolded states

187.Osazones are not formed with the a. b. c. d.

Glucose Fructose Sucrose Lactose

188.The purity of an enzyme at various stages of purification is best measured by: a. b. c. d. e.

Total protein Total enzyme activity Specific activity of the enzyme Percent recovery of protein Percent recovery of the enzyme

189. Which would be best to separate a protein that binds strongly to its substrate? a. b. c. d.

Affinity chromatography Cation exchange Anion exchange Cation or anion exchange

190. The most important buffering system for maintaining proper blood pH is:

a. b. c. d.

the charges on the amino acids the bicarbonate buffer system of CO2, carbonic acid, and bicarbonate phosphate groups of serum phosphoproteins none of the above

191. Myoglobin binding of oxygen depends on: a. the oxygen concentration (pO2) b. the hemoglobin concentration c. the affinity of myoglobin for the O2 a. and c) d. a), b) and c)

192. In the α helix the hydrogen bonds: a. b. c. d. e.

are roughly parallel to the axis of the helix are roughly perpendicular to the axis of the helix occur mainly between electronegative atoms of the R groups occur only between some of the amino acids of the helix occur only near the amino and carboxyl termini of the helix

193. In the binding of oxygen to myoglobin, the relationship between the concentration of oxygen and the fraction of binding sites occupied can best be described as: a. b. c. d. e.

hyperbolic linear with a negative slope linear with a positive slope random sigmoidal

194. Which of the following is true about the properties of aqueous solutions? a. A pH change from 5.0 to 6.0 reflects an increase in the hydroxide ion concentration b. ([OH-]) of 20%.

c. A pH change from 8.0 to 6.0 reflects a decrease in the proton concentration ([H+]) by a factor of 100. d. Charged molecules are generally insoluble in water. e. Hydrogen bonds form readily in aqueous solutions. f. The pH can be calculated by adding 7 to the value of the pOH.

195. The Henderson-Hasselbalch equation: a. b. c. d. e.

allows the graphic determination of the molecular weight of a weak acid from its pH alone. does not explain the behavior of di- or tri-basic weak acids employs the same value for pKa for all weak acids is equally useful with solutions of acetic acid and hydrochloric acid relates the pH of a solution to the pKa and the concentrations of acid and conjugate base

196. Two amino acids of the standard 20 contain sulfur atoms. They are: a. b. c. d. e.

cysteine and serine cysteine and threonine methionine and cysteine methionine and serine threonine and serine

197. The enzyme fumarase catalyzes the reversible hydration of fumaric acid to l-malate, but it will not catalyze the hydration of maleic acid, the cis isomer of fumaric acid. This is an example of: a. b. c. d. e.

biological activity chiral activity racemization stereoisomerization stereospecificity

198. The major reason that antiparallel β-stranded protein structures are more stable than parallel β-stranded structures is that the latter a. are in a slightly less extended configuration than antiparallel strands

b. c. d. e.

do not have as many disulfide crosslinks between adjacent strands do not stack in sheets as well as antiparallel strands have fewer lateral hydrogen bonds than antiparallel strands have weaker hydrogen bonds laterally between adjacent strands

199. Experiments on denaturation and renaturation after the reduction and reoxidation of the –SS- bonds in the enzyme ribonuclease (RNase) have shown that: a. folding of denatured RNase into the native, active conformation requires the input of energy in the form of heat b. native ribonuclease does not have a unique secondary and tertiary structure c. the completely unfolded enzyme, with all –S-S- bonds broken, is still enzymatically active d. the enzyme, dissolved in water, is thermodynamically stable relative to the mixture of amino acids whose residues are contained in RNase e. the primary sequence of RNase is sufficient to determine its specific secondary and tertiary structure

200. Myoglobin and the subunits of hemoglobin have: a. b. c. d.

no obvious structural relationship very similar primary and tertiary structures very similar primary structures, but different tertiary structures very similar tertiary structures, but different primary structures

201. When the linear form of glucose cyclizes, the product is a(n): a. b. c. d. e.

anhydride glycoside hemiacetal lactone oligosaccharide

202. You have isolated an unknown protein from thermophilic bacteria growing in one of the geothermal pools of Yellowstone National Park. Name three reagents you could use to cleave the protein to determine its primary structure.

a. b. c. d.

trypsin; chymotrypsin; CNBr trypsin, pepsin,rennin; protease, chymotrypsin; pepsin CNBr, protease, Pepsin

203. The dihedral angle indicated by an arrow in the tripeptide molecule corresponds to

a. b. c. d.

psi angle Phi angle Chi angle Omega angle

204. The amino acid sequence in the above tripeptide a. b. c. d.

Glutamine- valine- threonine Asparagine-valine-Serine Glutamine-Leucine- Threonine Asparagine-Valine-Threonine

205. In the diagram below, the plane drawn behind the peptide bond indicates the:

a. b. c. d. e.

absence of rotation around the C-N bond because of its partial double-bond character plane of rotation around the Cα-N bond region of steric hindrance determined by the large C=O group region of the peptide bond that contributes to a Ramachandran plot What kind of isomers are B and C?

ANSWER SHEET 1.C 2.A 3.C 4.A 5.B 6.D 7.A 8.B 9.A 10.A 11.C 12.B 13.A 14.A 15.B 16.E 17.B 18.A 19.A 20.D 21.D 22.C 23.B

24.D 25.A 26.D 27.B 28.A 29.C 30.D 31.B 32.E 33.C 34.B 35.A 36.C 37.D 38.D 39.C 40.C 41.A 42.A 43.E 44.B 45.B 46.A 47.A

48.C 49.A 50.D 51.B 52.C 53.A 54.D 55.D 56.B 57.D 58.C 59.D 60.D 61.E 62.D 63.A 64.A 65.D 66.B 67.A 68.C 69.E 70.B 71.B

72.B 73.C 74.A 75.A 76.D 77.D 78.D 79.C 80.D 81.B 82.A 83.C 84.E 85.B 86.D 87.C 88.C 89.D 90.C 91.D 92.A 93.C 94.A 95.C

96.A 97.D 98.B 99.C 100.B 101.B 102.A 103.A 104.B 105.B 106.B 107.A 108.B 109.C 110.D 111.A 112.D 113.C 114.C 115.A 116.D 117.C 118.C 119.B

120.D 121.A 122.B 123.B 124.A 125.B 126.C 127.B 128.A 129.C 130.D 131.B 132.A 133.D 134.C 135.C 136.A 137.C 138.B 139.A 140.B 141.B 142.A 143.A

144.B 145.D 146.C 147.B 148.C 149.A 150.C 151.B 152.A 153.B 154.D 155.C 156.B 157.A 158.A 159.A 160.E 161.B 162.D 163.A 164.D 165.A 166.B 167.B

168.A 169.B 170.A 171.A 172.C 173.B 174.C 175.B 176.D 177.D 178.B 179.B 180.A 181.D 182.B 183.B 184.C 185.D 186.B 187.B 188.C 189.C 190.A 191.B

192.C 193.B 194.A 195.D 196.E 197.A 198.E 199.E 200.E 201.D 202.C 203.A 204.B 205.A 206.a

Conformation of Proteins

1. a. b. c. d.

β-sheets found in a protein can be considered an example of: Primary structure Secondary structure Tertiary structure Quaternary structure

2. Which of the following statements best describes the tertiary structure of a protein? a. The 3-D conformation of a multisubunit protein, compose of a number of subunits joined by noncovalent interactions b. Local regions of polypeptide chains that have a regular conformation, which is stabilized by hydrogen bonds c. The total 3-D conformation of an entire polypeptide chain including a-helices, bsheets and any other loops or bends. d. The linear sequence of amino acids in the polypeptide chain e. Certain types of conformational domains, which are composed of an assortment of α and β structures

3. a. b. c. d.

All of the following are types of secondary structures EXCEPT α-helix β-sheet Bend or turn Domain

4. a. b. c. d.

R-groups in an α-helix: Extend outward from the central axis to avoid interfering sterically with each other Extend inward from the central axis Have to be close together to stabilize the helix Form the anti-parallel β-sheets

e. Are hydrogen bonded to peptide bonds four amino acids away in the chain

5. All of the following statements regarding the R-groups in an α-helix are true EXCEPT: a. Runs of negative R-groups will disrupt the helix b. Runs of positive R-groups will stabilize the helix c. The R groups all protrude out from the central axis d. Runs of bulky R-groups will disrupt the helix because of steric hindrance e. Proline will destabilize the helix

6. A β-sheet consisting of β-strands connected laterally. Which of the following is TRUE of anti-parallel β-sheets? a. The N-termini of successive strands are oriented in the same direction b. The N-termini of successive strands are adjacent to one another c. Successive β strands alternate directions so that the N-terminus of one strand is adjacent to the C-terminus of the next d. Hydrophobic residues are located on both sides of the sheet e. The strands form hydrogen bonds with with loops or turns

7. All of the following statements regarding β-sheets are true EXCEPT? a. β-sheets are formed between 2 linear regions of polypeptide chains b. It is possible for one side of a b-sheet to be hydrophobic while the other side is hydrophilic c. Anti-parallel b-sheets are formed when the amino terminal to carboxy terminal directions of the two strands match d. Anti-parallel chains are often the same chain doubled back upon its self e. The two strands of a β-sheet are stabilized by hydrogen bonds

8. Which of the following is NOT a non-repetitive secondary structure? a. Bend b. Turn c. Loop d. Domain

9. Nonregular, nonrepetitive secondary structures a. Include α-helices and β-sheets b. Are bends, loops, and turns that do not have a repeating element c. Are beta-strands d. Are Beta-sheets e. Include beta-sheets

10. Motifs are supersecondary structures. Any motif can be defined by describing: a. The first 50 to 100 amino acids in the primary structure b. Any secondary structure and the turn at its C-terminus c. Any secondary structure and the loop at its N-terminus d. Any two secondary structures in the protein e. A group of secondary structures and their relationship to one another

11. A motif is best described as? a. Specific combinations of secondary structures that occur in a number of types of proteins b. Combinations of secondary structures that occur in only one enzyme from different species c. A specific primary sequence that occur in a number of types of proteins d. A structural domain e. A section of protein structure sufficient to perform a particular physical or chemical task

12. Which of the following is NOT true of structural domains? a. They usually contain a number of loops, turns or bends in addition to other secondary structures b. They usually contain of one or more secondary structural motifs c. They are usually about 25 or 300 amino acids in length d. They can maintain their conformation without the rest of the protein e. All the amino acids in the protein must be a part of the structural domains in the protein

13. A structural domain is best described as? a. A monomeric subunit in a quaternary protein b. Combinations of unstable secondary structures that occur in a number of types of proteins c. Loops that contain residues beyond the minimum number necessary to connect adjacent regions of secondary structure d. A series of secondary structures whose only common feature is that they form a turn, bend or loop e. A section of protein structure sufficient to perform a particular physical or chemical task

14. Proteins in a fold family usually share all of the following EXCEPT: a. Usually have a similar function b. Contain the same secondary structures c. Share a probable common evolutionary origin d. Have one or more domains e. Must contain only beta-strands or α-helix

15. Folds are relatively large patterns of 3-dimentional structures that have been recognized in domain(s) of one or more proteins. All of the following statements about folds are true, EXCEPT? a. It is believed that there are only a few thousand folds for all the proteins that ever existed b. Protein folds form because of the thermodynamic stability to the secondary structure arrangements c. Members of a fold have a similar arrangement of secondary structure in the domain d. Members of a fold have the same type of secondary structures e. Proteins often belong to more than one fold because it is very easy for an alpha-helix to become a beta-sheet

16.Folds are relatively large patterns of 3-dimentional structures that have been recognized in domain(s) of one or more proteins. All of the following statements about folds are true, EXCEPT?

a. b. c. d. e.

Members of a fold may have only beta strands Members of a fold may have only alpha-helices Members of a fold my contain a mixture of alpha helices and beta strands Members of a fold may contain only 7 beta strands Members of a fold are clearly related to one another by evolution

17 If we say that two proteins are homologous, we are saying that they a. Probably derived from different ancestral genes b. Thei tertiary structures would look dissimilar c. They probably have the same number and types of secondary structures oriented in the same way in space d. Their primary sequences would be 99% similar e. They have the different functions

18.Actin, heat shock protein 70 and hexokinase are homologous because: a. Their primary structures are very similar b. There is a structural correspondence between the three fold families that implies a common ancestral gene c. They all have the same primary structure d. Their evolution depended upon different genes and was not related e. They are only found in

19. Which of the following amino acids is most likely to be found in the interior of a typical globular protein? a. Leucine b. Glutamate c. Threonine d. Lysine e. Serine

20. Which of the following statements is true with respect to the amino acids in a globular protein?

a. Most of the hydrophobic amino acids are found in the center of globular proteins b. Most of the hydrophobic amino acids are found on the surface of globular proteins c. Most of the hydrophilic amino acids are found in the center of globular proteins d. Most of the charged amino acids are found in the center of globular proteins e. Glycine, valine, leucine, and isoleucine would normally be found on the surface

21. An amino acid contains no ionizable group in its side chain (R). It is titrated from pH 0 to 14. Which of the following ionizable state is not observed during the entire titration in the pH range 0 - 14? ANS: 4

Answer from the following: a. (1) b. (2) & (4) c. (3) d. (4)

22. An α-helix in a peptide or protein is characterized by hydrogen bonds and characteristic dihedral angles. Choose the right combination. a. Hydrogen bonding between the amide CO of residue i and amide NH of residue i + 4. Dihedral angles in the region ϕ = -50°, ψ = -60°. b. Hydrogen bonding between the amide NH of residue i and amide CO of residue i + 4. Dihedral angles in the region of ϕ = -50°, ψ = -60.

c. Hydrogen bonding between the amide CO of residue i and amide NH of residue i + 4. Dihedral angles in the region of ϕ = - 50°, ψ = +60 d. Hydrogen bonding between the amide CO of residue i and amide NH of residue 1 + 3. Dihedral angles in the region of ϕ = -50°, ψ = -60

23. Protein molecules carry ionized groups, H-bonding group, and non-polar group on their surfaces. Which type of non-covalent interaction do you suppose is likely to be most important in causing protein molecules in dilute aqueous solution to associate with each other? a. Hydrophobic interactions b. Hydrogen bonding c. Covalent interactions d. Ionic bonds

24. How many different polypeptides of 61 residues can be made from the 20 naturally occurring amino acids? a. 61^3 b. Infinite c. 20^3 d. 2 0^61

25. In proteins under normal physiological condition (pH =7) which of the following amino acids will be partially ionized? a. Arginie b. Lysine c. Histidine d. Serine

26. The tertiary structure and function of a polypeptide is principally determined by the a. length of the polypeptide b. number of nucleotides present in the polypeptide c. repeated units of glycerol making up the polypeptide d. interactions between amino acids present in the polypeptide

e. number of introns within the polypeptide

27. In an polypeptide if alanine is replaced by proline then a. Its tendency to form α helix increases b. Its tendency to form β sheet increases c. The hydrophobicity of chain decreases d. There would be no effect

28. A stong peptide bond is present between arginine and aspartic acid. This bond will be weak if arginine will be replaced by a. Glutamic acid b. Lysine c. Histidine d. Proline

29. In which of the secondary structures, two residues well separated (more than 10 residues) in the amino acid sequence can be proximal in the three-dimensional structure? a. Alpha-helix b. beta-sheet c. 310-helix d. Beta turn

30. The end-to-end distance of a 10 amino acid alpha-he1ix is -1 5.2A. If these ten residues were in a fully extended conformation (beta-strand) the approximate end-to-end distance would be a. 15.2A b. 5A c. 32A d. 70A

31. If a valine residue is buried in a protein structure, which one of the substitutions is likely to be least disruptive to the three-dimensional structure? a. Val + Ile b. Val + Gly c. Val+ Thr d. Val + Phe

32. In a peptide unit Cα refers to the central carbon atom in an amino acid and C refers to the carbon in the carbonyl group. Which covalent bond is the most difficult to rotate in the peptide unit? a. Cα-C b. Cα-N c. N-H d. N-C

33. A protein segment consists of a stretch of 10 residues that adopt an α - helical secondary structure conformation. What is the maximum number of hydrogen bonds that can stabilize this α - helix? a. 4 b. 5 c. 6 d. 7

34. Salt bridges formed in proteins involve a. SuIfur atoms in cysteine side chains b. Metal ions such as Na and K c. Opposite charged side chains d. ProximaI aromatic and aliphatic side chains

35. The quaternary structure of human hemoglobin is best described as a a. Dimer of two myoglobin dimers

b. Tetramer of identical subunits c. Tetramer of four different subunits d. Tetramer of two different subunits

36. A favourable charge interaction between R groups in an alpha helix is expected to occur when the interacting side chains are separated by a. One two residues b. Three four residues c. Five six residues d. Seven eight residues

37. Difference between the R and T states of haemoglobin is a. Dimeric and tetrameric structures b. Oxy and deoxy states c. Binding to ferrous and ferric ions d. Folded and unfolded states

38. If there are 20 kinds of amino acids and they can be assembled in any order, then how many different proteins of 100 amino acids can theoretically be produced? a. 20 x 100 b. 20¹⁰⁰ c. 100²⁰ d. 20 x 20 x 100 39. How many different pentapeptides is there that contain one residue each of Gly, Asp, Tyr, Cys and Leu? a. 625 b. 120 c. 32 d. None

40. In five-residue long peptide each amino acid can adopt three conformations. The total number of conformations that the peptide can adopt is

a. 15 b. 243 c. 8 d. 125

41. If one needs to use the natural 20 amino acid residues, with no restrictions on the number of times a given residue type can be used, what is the total number of possible tripeptide sequences? a. 20 x 3 b. 20 x 19 x 18 c. 4000 d. 8000

42. Enormous diversity of protein molecules is due to a. Sequence of amino acids b. R-group of amino acids c. Peptide bonds d. Amino groups of amino acids

43. An alpha helix represents a. Primary structure of protein b. secondary structure of protein c. tertiary structure of protein d. aggregation of proteins

44. the shape of polypeptide backbone making α-helix in protein is a. like the English letter “a” b. right handed spiral c. left handed spiral d. like the Greek letter “α”

45. hydrogen bonds in α-helices are

a. more numerous than van der Waals interactions b. not present at phenylalanine residues c. analogous to the steps in a spiral staircase d. roughly parallel to the helix axis

46. A α-helix is made up of 78 amino acid residues. What will be the value of its axial lengthand the length when polypeptide chain is fully extended respectively a. 117Å ,117 Å b. 281 Å , 117 Å c. 117 Å, 281 Å d. None

47. Consider three polypeptides of 15 residues each. They adopt distinct conformations corresponding to right handed α-helical structure, left handed α-helical structure and single strand of β-sheet structure. Considering the shortest distance between the first and the last residues as the length of the structure, which one of the statement is true a. right handed α-helical structure is longest b. left handed α-helical structure is longest c. β-sheet structure is longest d. all the three have identical length

48. Which one of the following statements of the protein secondary structure is correct? a. An α-helix is primarily stabilized by ionic interactions between the side chains of amino acids b. β-sheet exist only in Antiparallel form c. β turns often contain proline d. an α-helix can be composed of more than one polypeptide chain

49. the amino acid in the protein super secondary structure are held together primarily by a. charge-charge interaction b. covalent cross links c. favorable R group interactions d. Main chain hydrogen bonding

50. The mirror image of right handed α-helix with all L- amino acids will appear as a. left handed α-helix with L- amino acids b. left handed α-helix with D- amino acids c. right handed α-helix with L- amino acids d. right handed α-helix with D- amino acids

51. Which of the following has a quaternary structure? a. Α- chymotrypsin b. Hemoglobin c. Insulin d. Myoglobin

52. For a protein to have a quaternary structure, it must a. Have 4 amino acids b. Consist of two or more polypeptide subunits c. Consist of four polypeptide subunits d. Have at least four disulfide bridges

53. The forces that maintain the three- dimensional structure of a protein is mainly a. Non-covalent b. Covalent c. Coordinate d. Covalent and Non-covalent

54. The following type of interaction is mainly responsible for aggregation of proteins in dilute solutions a. Hydrogen bonds b. Hydrophobic interaction c. Disulphide bonds d. Peptide bonds

55. Protein structures are confirmed by weak bonds, the example of which is a. Hydrophobic b. Disulphide c. Peptide d. All the above

56. A favorable charge-charge interaction between R group in an α-helix is expected to occur when the interacting side chains are separated by a. 1 – 2 residues b. 3 – 4 residues c. 5 – 6 residues d. 7 – 8 residues

57. Which one of the following is example of a non-covalent interaction in proteins? a. Salt bridge b. Disulphide bridge c. Peptide bond d. Phophodiester bond

58. What does a protein lose when it denatures? a. Its primary structure b. Its peptide bonds c. Its sequence of amino acids d. Its three dimensional shape

59. Which one of the following is the least important factor as far as protein folding is concerned? a. Hydrophobic effect b. Hydrogen bonding c. Electrostatic interactions d. Quaternary association

60. Disulphide bridge forms between two cystiene residues as a result of a. Oxidation of sulfhydral group b. Reduction of sulfhydral group c. Amide formation d. None

61. In how many ways can 8 cystiene residues form S-S bonds a. 8 b. 16 c. 24 d. 28

62. When you boil an egg, you convert the albumin into white solid mass. In chemical terms you would say that a. The protein was dehydrated by heat b. The protein was denatured by heat c. The protein was cross-linked by heat d. The protein was degraded by heat

63. Thermal denaturation of proteins involves a. Conformational change in the protein b. Random cleavage of the peptide bonds c. Covalent modification of certain amino acids d. Increase in its isoelectric

64. DNA polymerase contains a lysine residue that is important for binding to DNA. Mutations were found that converted this lysine into glutamate, glycine, valine or arginine. Which mutations would be predicted to be the most and least harmful to the ability of the enzyme to bind DNA? Most

Least

a. Glycine

Arginine

b. Arginine

Glycine

c. Glutamate

Valine

d. Glutamate

Arginine

65. Which statement about protein is correct? a. Protein quaternary structure is determined solely by the primary amino acid sequence b. Protein sequence that span a lipid bilayer membrane contain a number of charged amino acids c. Examples of protein secondary structure include an alpha helix or a beta pleated sheet d. Protein tertiary structure is the result of the interaction of two or more independent polypeptides.

66. Which of the following statement about protein structure is correct? a. The extended β-configuration is not found in globular proteins b. The stability of the α- helix is mainly due to hydrophobic interactions c. Globular proteins tend to fold into configurations that keep hydrophobic side chains in the interior d. The primary structure of a peptide does not influence the formation of the native-three dimensional configuration

67. Globular and Fibrous proteins Hemoglobin is a. Monomeric protein b. Oligomeric protein c. Chromo protein d. Both b and c

68. The quaternary structure of human hemoglobin is best described as a a. Dimer of two myoglobin dimmers b. Tetramer of identical subunits c. Tetramer of four different subunits d. Tetramer of two different subunits

69. When Glu at the sixth position in the β-chain of human hemoglobin is replaced by Val, a. Secondary structure are significantly altered b. Tertiary structure are significantly altered c. Quaternary structure are significantly altered d. No significant structural change takes place

70.Difference between the R- and T-states of hemoglobin is a. Dimeric and tetrameric structure b. Oxy and deoxy states c. Binding to ferrous and ferric ions d. Folded and unfolded states

71. Binding of oxygen to hemoglobin when Fe is in the following oxidation state a. +1 b. +2 c. +3 d. +4

72. A protein in 100 mM KCl solution was heated and the observed Tm (midpoint of unfolding) was 60°C. When the same protein solution in 500 mM KCI was heated, the observed Tm was 65°C. What is the most probable reason for this increase in Tm?

(1) Hydrophobic interaction is increased and electrostatic repulsion is decreased.

(2) Hydrophobic interaction is decreased and electrostatic repulsion is increased. (3) Hydrogen-bonding is increased. (4) vander Waals interaction is increased

Answer key: 1. b 2. e 3. d 4. a 5. b 6. c 7. c 8. d 9. b 10.e 11.a 12.e 13.e 14.e 15.e 16.e 17.c 18.b 19.a 20.a 21.d 22.a 23.a

24.d 25.c 26.d 27.c 28.d 29.a 30.c 31.a 32.d 33.b 34.c 35.d. 36.b 37.b 38.b 39.a 40.b. 41.d 42.a 43.b 44.b 45.d 46.c 47.c

48.C 49.a 50.a. 51.C 52.b 53.a 54.b 55.d 56.b 57.b 58.d 59.d 60.a 61.b 62.b. 63.a 64.c 65.C 66.C 67.b 68.d 69.c 70.b 71.b

ENZYMES 1. Which of the following statements concerning enzymes is FALSE? a. Each enzyme has an optimal temperature. b. Each enzyme has an optimal pH. c. Most enzymes are highly specific. d. Some enzymes require cofactors. e. Most enzymes are RNA molecules. 2. Parts of the enzyme molecule that interact with a substrate are called: a. cofactors. b. active sites. c. induced-fit models. d. orientation sites. e. reaction sites. 3. What refers to the situation in which the binding of a substrate to the enzyme causes a change in the enzyme’s shape, facilitating an enzyme’s function? a. active site b. cofactor c. activation energy d. induced fit e. allosteric inhibition 4. Hydrolases are one important class of enzyme that function to catalyze a. splitting a molecule using water. b. conversions between isomers. c. reactions in which double bonds are formed. d. oxidation-reduction reactions. e. None of these. 5. Which refers to an organic, nonpolypeptide compound that binds to the apoenzyme and serves as a cofactor? a. coenzyme b. catalyst c. substrate d. mineral e. allosteric regulator. 6. Which of the following does not represent a method by which cells regulate enzyme activity? a. controlling the intracellular concentration of the enzyme b. feedback inhibition of enzymes c. the binding of allosteric regulators to allosteric sites on the enzyme d. differential gene expression of the genes that produce enzymes e. heat denaturation of the enzyme 7. Select the enzyme that does not match the reaction: a. sucrase—splits sucrose into glucose and fructose b. lipase—breaks ester linkages c. phosphatase—removes phosphate groups

d. urease—converts urea to ammonia and CO2 e. kinase—breaks peptide bonds 8. If one continues to increase the temperature in an enzyme-catalyzed reaction, the rate of the reaction: a. Does not change. b. Increases and then levels off. c. Decreases and then levels off. d. Increases and then decreases rapidly. e. Decreases and then increases rapidly. 9. You conduct an experiment in which you add increasing amounts of substrate to an enzyme solution and then measure the resulting reaction rate. What do you conclude? a. The reaction rate is directly proportional to substrate concentration. b. The reaction rate is independent of substrate concentration. c. The reaction rate increases but then decreases, forming a bell-shaped curve. d. The reaction rate decreases with increasing substrate concentration. e. The reaction rate increases with increasing substrate concentration up to a point, above which the rate remains constant. 10. An allosteric enzyme: a. has an active site that binds to the substrate and an allosteric site that binds to the product. b. allows the movement of molecules and ions from one part of the cell to another. c. catalyzes both oxidation and reduction reactions. d. raises a reaction’s activation energy so that the rate of the reaction declines. e. allows a substance other than the substrate to bind to the enzyme, thereby activating or inactivating it. 11. Competitive inhibitors inhibit biochemical reactions in such a way as to seemingly: a. Increase the concentration of substrate. b. Reduce the concentration of enzyme. c. Increase the concentration of enzyme. d. Reduce the concentration of substrate. e. Denature the enzyme 12. Penicillin is a drug that acts by: a. Irreversibly inhibiting transpeptidase. b. Reversibly inhibiting transpeptidase. c. Competitively inhibiting transpeptidase. d. Noncompetitively inhibiting transpeptidase. e. None of these. 13. The relationship between an enzyme and a reactant molecule can best be described as: a. a temporary association. b. an association stabilized by a covalent bond. c. one in which the enzyme is changed permanently. d. a permanent mutual alteration of structure e. noncomplementary binding.

14. Which of the following statements about initial rate enzyme inhibition kinetics is TRUE? a. Competitive inhibition permanently lowers the maximum velocity of the enzyme. b. Non-competitive inhibitors resemble the substrate. c. You cannot perform kinetics in the presence of inhibitors since the enzyme is no longer active. d. Whenever Vmax decreases as a result of the presence of inhibitors, Km decreases also e. A competitive inhibitor is a structural analogue of the enzyme's substrate. 15. Which of the following is not an example of covalent regulation of enzyme activity? a. ADP-ribosylation of arginine residues. b. Uridylylation of tyrosine residues. c. Dephosphorylation of threonine residues d. Phosphorylation of serine residues. e. All of the above are examples of covalent regulation of enzyme activity. 16. What is the name given to a molecule that decreases the activity of an enzyme by binding to a site other than the catalytic site? a. Alternative inhibitor. b. Allosteric inhibitor. c. Stereospecific agent. d. Competitive inhibitor. e. Transition-state analog. 17. When performing an assay to accurately measure (quantitate) the enzyme activity of a biological sample (i.e. NOT a kinetic study), which of the following parameters must be rate-limiting? a. Buffer. b. Temperature. c. Cofactor concentration. d. Km for substrates. e. Enzyme concentration . 18. Which of the following statements is FALSE? a. A reaction may not occur at a detectable rate even though it has a favourable equilibrium. b. At the end of an enzyme-catalysed reaction, the functional enzyme becomes available to catalyse the reaction again. c. Substrate binds to an enzyme's active site. d. For conversion of A to B, a catalyst shifts the reaction equilibrium to the right. e. Lowering the temperature of a reaction will lower the reaction rate.

19. Which of the following statements about sigmoidal kinetics is TRUE? a. Accurate determination of a value for Km for the substrate is not possible. b. This type of kinetics occurs only for enzymes involved in feedback inhibition.

c. Sigmoidal kinetics has nothing to do with Koshland's Direct Sequential model for multiple subunit enzymes. d. The kinetics cannot be analysed using the Michaelis Menten e quation. e. Such kinetics occurs because the enzyme is saturated with low levels of substrate. 20. Which of the following statements about enzymes is TRUE? a. Catabolic enzymes require energy to degrade large macromolecules. b. Each enzyme must have a bound cofactor for a reaction to occur. c. Multienzyme complexes consist of many identical enzyme polypeptides linked to each other like Lego blocks. d. Anabolic enzymes degrade carbohydrates. e. A holoenzyme consists of an inactive apoenzyme plus a cofactor. 21. In a kinetic plot of enzyme activity versus substrate concentration, why does the curve asymptote (taper off) at high substrate concentrations? a. The reverse reaction starts to occur. b. The enzyme is saturated with substrate. c. The reaction reaches equilibrium. d. The reaction runs out of substrate. e. The reaction cannot exceed the Vmax. 22. In order for catalysis to occur, several requirements must be met. Which of the following is NOT a requirement? a. The enzyme must be in an active form. b. Hydrogen bonding between the substrate and water must be replaced with covalent bonding to the enzyme. c. The activation energy must be overcome. d. Functional groups must be correctly aligned. e. Entropy must be reduced. 23. Which best describes the active site of an enzyme? a. A highly polar region of the enzyme molecule. b. A few amino acid residues brought together to interact with the enzyme's substrate following correct folding of the enzyme molecule. c. A region of the enzyme where high energy molecules bind to provide energy for the catalytic process. d. An allosteric site which, when occupied, changes the enzyme's conformation. e. A small hole or cleft in the enzyme surface which is large enough for small substrate molecules to enter. 24. Which of the following is the role of an enzyme in an enzyme-catalysed reaction? a. To ensure that the product is more stable than the substrate. b. To make the free-energy change for the forward reaction more favourable. c. To increase the rate at which the equilibrium between substrate and product is reached. d. To ensure that all the substrate is converted to product. e. To do none of the above.

25. What is the single critical feature of initial rate kinetic measurements? a. You can re-plot your data using one of several linear transformations of the Michaelis Menten equation. b. Clinical applications of the data permit the detection of defective enzymes or organ malfunction. c. You must know the Km for the substrate before doing the assay so that enough substrate is added for the reaction to proceed. d. Activity is measured for the forward reaction only. e. The enzyme must NOT be a regulatory enzyme. 26. Enzymes are potent catalysts. Which of the following is TRUE? a. They quickly drive reactions to equilibrium. b. They are consumed in the reactions they catalyse. c. They are very specific and can prevent the conversion of products back to substrates. d. They increase the equilibrium constants for the reactions they catalyse. e. They lower the maximum velocity of the reactions they catalyse. 27. In competitive inhibition, what does the inhibitor do? a. It binds at several different sites on an enzyme. b. It binds reversibly at the active site. c. It binds only to the ES complex. d. It binds covalently to the enzyme. e. It lowers the characteristic Vmax of the enzyme. 28. Which of the following statements is unrelated to protein phosphorylation? a. Single displacement reactions may be random or ordered sequential. b. Large numbers of enzyme molecules may be activated in response to hormonal stimuli. c. Initial rate kinetics usually generate a sigmoidal plot of v versus [S]. d. Phosphorylase kinase is an inter-converible enzyme with multiple subunits. e. Serine, threonine or tyrosine amino acids are modified. 29. Which of the following statements about the Michaelis Constant is TRUE? a. The term refers to half of the maximum enzyme activity. b. The term is given the symbol KMAX. c. The value increases in the presence of a competitive inhibitor. d. The value is read directly from the "v" axis of an Eisenthal & Cornish Bowden kinetic plot. e. The value cannot be determined for regulatory enzymes. 30. One of the enzymes involved in glycolysis, aldolase, requires Zn2+ for catalysis. Under conditions of zinc deficiency, when the enzyme may lack zinc, it would be referred to as which of the following? a. Holoenzyme. b. Prosthetic group. c. Apoenzyme. d. Coenzyme. e. Substrate. 31. Which of the following statements about the Vmax for an enzyme-catalysed reaction is

true? The value for Vmax is independant of temperature. Vmax increases in the presence of a competitive inhibitor. Vmax is unchanged in the presence of uncompetitive inhibitor. Vmax is twice the rate observed when the concentration of substrate is equal to the Km. e. Vmax is limited only by the amount of substrate supplied. a. b. c. d.

32. Which of the following describes the concept of "induced fit"? a. When a substrate binds to an enzyme, the enzyme induces a loss of water (desolvation) from the substrate. b. Substrate binding may induce a conformational change in the enzyme, which then brings catalytic groups into proper orientation. c. Enzyme-substrate binding induces an increase in the reaction entropy, thereby catalysing the reaction. d. Enzyme specificity is induced by enzyme-substrate binding. e. Enzyme-substrate binding induces movement along the reaction coordinate to the transition state. 33. Which of the following terms does not relate to regulatory enzymes? a. Feed-forward activation. b. Homotropic. c. Heterotropic. d. Allosteric. e. Endergonic. 34. The Km (Michaelis constant) of an enzyme for a substrate is defined operationally as a. half the substrate concentration at which the reaction rate is maximal b. the substrate concentration at which the reaction rate is half maximal c. the dissociation constant of the enzyme- substrate complex d. the dissociation constant of the enzyme- product complex e. the rate constant of the reaction at saturation 35. a. b. c. d.

Enzymes accelerate reactions by ___ the activation energy needed for the reaction. stopping raising lowering binding

36. a. b. c. d.

The values for environmental factors at which an enzyme functions best is called the. paradigm of maximum effectiveness optimal conditions comfortable conditions efficient conditions

37. During the procedure using the potato extract demonstrating the affect of temperature, what was the enzyme studied? a. catalase b. catechol

c. catechol oxidase d. benzoquinone 38. During the procedure using the meat extract demonstrating the affect of pH, what was the substrate of the reaction? a. catalase b. catechol c. catechol oxidase d. hydrogen peroxide 39. What happens to an enzyme when it denatures? a. The activation energy of the reaction is doubled. b. The activation energy of the reaction is lowered. c. It's optimal conditions for temperature of the enzyme are doubled. d. The shape of the enzyme molecule is changed. 40. During the procedure using the turnip extract demonstrating the affect of inhibitors, what was the enzyme studied? a. peroxidase b. catalase c. catechol oxidase d. hydrogen peroxide 41. Exactly how would an extremely low pH (less than 2.0) denature an enzyme? a. The extra energy would cause violent movement and collisions until the bonds in the molecule broke, causing the shape to change. b. The high concentration of hydrogen ions (H+) would break the bonds in the molecule, causing the shape to change. c. The high concentration of hydroxide ions (OH-) would break the bonds in the molecule, causing the shape to change. d. Because pH molecules are structurally similar to the substrate molecules, they inhibit the chemical reaction. 42. Exactly how do inhibitors affect the reaction rate of enzymes? a. The extra energy would cause violent movement and collisions until the bonds in the molecule broke, causing the shape to change. b. The high concentration of hydrogen ions (H+) would break the bonds in the molecule, causing the shape to change. c. The high concentration of hydroxide ions (OH-) would break the bonds in the molecule, causing the shape to change. d. Because the inhibitor molecules are structurally similar to the substrate molecules, they slow down the chemical reaction.

43. For some of the procedures a spectrophotometer was used at an exact wavelength. Why was this wavelength used as opposed to any others? a. It was the wavelength of the highest absorbance. b. It was the wavelength of lowest absorbance. c. It was the lowest wavelength setting for the spectrophotometer. d. It was the wavelength at which the enzyme would react with the substrate.

44. The catalytic activity of an enzyme is restricted to its small portion called a. Active site b. Passive site c. Allosteric site d. All Choices are correct 45. An activated enzyme made of polypeptide chain and a co-factor is a. Coenzyme b. Substrate c. Apoenzyme d. Holoenzyme 46. Koshland in 1959 proposed a. Fluid mosaic model b. Induce fit model c. Lock and key model d. Reflective index model 47. Enzymes are largely _________________________ in their chemical nature. a. Lipids b. Steroids c. Proteinaceous d. All A, B and C 48. Who proposed “lock and key” model to study enzyme – substrate interaction? a. Koshland (1959) b. Wilhelm Kuhne (1878) c. Fischer (1898) d. None of these 49. In human body the optimum temperature for enzymatic activities is a. 37ºC b. 40ºC c. 25ºC d. 30ºC 50. Optimum pH value for pepsin is a. 5.5 b. 7.4 c. 4.1 d. 1.4

51. Competitive inhibitors stop an enzyme from working by a. Changing the shape of the enzyme b. merging with the substrate instead c. blocking the active site of the enzyme d. combining with the product of the reaction 52. The enzymes are sensitive to a. Changes in pH b. Changes in temperature c. Both A and B d. None of these 53. Enzyme B requires Zn2+ in order to catalyze the conversion of substrate X. The zinc is best identified as a(n): a. Coenzyme b. Activator c. Substrate d. Product 54. The enzyme minus its coenzyme is referred to as the a. Iso-enzyme b. Metalloenzyme c. Apoenzyme d. All of these 55. The “lock and key” model of enzyme action illustrates that a particular enzyme molecule a. forms a permanent enzyme-substrate complex b. may be destroyed and resynthesized several times c. interacts with a specific type of substrate molecule d. reacts at identical rates under all conditions 56. Consider this reaction. A + B --> C + D + energy. a. This reaction is exergonic b. An enzyme could still speed the reaction c. A and B are reactants; C and D are products d. All of these are correct 57. An inhibitor that changes the overall shape and chemistry of an enzyme is known as a(n) a. Auto-steric inhibitor b. Competitive inhibitor c. Steric inhibitor d. Noncompetitive inhibitor 58. Non-protein components of enzymes are known as a. Coenzymes b. Activators c. Cofactors d. All A, B, and C

59. The reaction below occurs within the cells to prevent the accumulation of hydrogen peroxide. In this reaction, catalase functions as an

a. b. c. d.

Enzyme in the breakdown of hydrogen peroxide Enzyme in the synthesis of hydrogen peroxide Emulsifier in the digestion of hydrogen peroxide Indicator in the detection of hydrogen peroxide

60. An enzyme is generally named by adding ________ to the end of the name of the ____________. a. "-ase". coenzyme b. "-ase". cell in which it is found c. "-ose". substrate . d. "-ase". substrate 61. The minimum amount of energy needed for a process to occur is called the a. Minimal energy theory b. Process energy c. Kinetic energy d. Activation energy 62. A student conducts an experiment to test the efficiency of a certain enzyme. Which would probably not result in a change in the enzyme's efficiency? a. b. c. d.

Adding an acidic solution to the setup Adding more substrate but not enzyme Increasing temperature of solution All a, b, & c change enzyme's efficiency

63. Enzymes function as a. Organic catalysts b. Inorganic catalysts c. Inhibitors d. All of these 64. A catalyst is a chemical involved in, but not ____________ by, a chemical reaction. a. Supported b. Changed c. Controlled d. All of these 65. Many enzymes function by __________________ the activation energy of reactions. a. Increasing b. Promoting c. Lowering d. Both A and B

66. An uncatalysed reaction requires a a. Higher activation energy b. Lower activation energy c. Balanced activation energy d. All of these 67. It suggests that the binding of the substrate to the enzyme alters the structure of the enzyme, placing some strain on the substrate and further facilitating the reaction. a. Lock and Key hypothesis b. Induced fit hypothesis c. Fischer’s hypothesis d. D.D. Wood’s hypothesis 68. They are non-protein organic molecules bound to enzymes near the active site. a. Activators b. Coenzymes c. Holoenzymes d. All of these 69. The first step in any reaction catalysed by an enzyme is the formation of a specific association between the molecules called an a. Enzyme-product complex b. Enzyme-intermediate complex c. Enzyme-substrate complex d. None of these 70. The function of competitive inhibitors is defined by their ability to interact or bind to a. The active site of an enzyme b. Regulatory sub-units of an enzyme c. Non-competitive inhibitor d. Enzyme cofactors 71. If an enzyme solution is saturated with substrate, the most effective way to obtain an even faster yield of products would be a. Add more of the enzymes b. Add more substrate c. Add an allosteric inhibitor d. Add a non-competitive inhibitor 72. During _____________ the final product of a metabolic pathway turn off the first step of metabolic pathway. a. Positive feed back b. Negative feed back c. Competitive feed back d. Both A and C 73. _____________ occurs when the inhibitory chemical, which does not have to resemble the substrate, binds to the enzyme other than at the active site. a. Noncompetitive Inhibition

b. Competitive Inhibition c. Uncatalysed reaction d. All A, B and C 74. Which one is not attribute of enzyme a. Specific in nature b. Protein in chemistry c. Consumed in reaction d. Increases rate of reaction 75. Which one inactivates an enzyme by indirectly changing the shape of the active site of an enzyme a. Non-competitive inhibitor b. Competitive inhibitor c. Coenzyme d. Activator 76. The enzymes are classified into a. Five groups b. Three groups c. Six groups d. Four groups 77. Non-proteinaceous part of holoenzyme is a. Prosthetic group b. Apoenzyme c. Tubulin d. None of these 78. Enymes are highly specific for a given substrate which is due to the shape of their a. Active site b. Allosteric site c. Non-competitive site d. None of these 79. The name enzyme was suggested in 1878 by the German physiologist a. Wilhelm Kuhne b. Koshland c. Fischer d. Paul Filder 80. Proteinaceous part of holoenzyme is a. Prosthetic group b. Apoenzyme c. Lecithin d. None of these 81. The "lock and key hypothesis" attempts to explain the mechanism of a. vacuole formation b. pinocytosis

c. sharing of electrons d. enzyme specificity 82. An enzyme that hydrolyzes protein will not act upon starch. This fact is an indication that enzymes are a. hydrolytic b. specific c. catalytic d. synthetic 83. The site where enzyme catalyzed reaction takes place is called? a. Active site b. Allosteric site c. Denatures site d. Dead Site 84. What is a cofactor? a. Inorganic ions b. Organic molecules c. Both a and b d. None of the above 85. Mg+2 is an inorganic activator for the enzyme a. Phosophatase b. Carbonic anhydrase c. Enterokinase d. Amylase 86. Zn+2 is an inorganic activator for enzyme. a. Carbonic anhydrase b. Phosophatase c. Chymotrypsin d. Maltase 87. Which antibiotic blocks the active site of an enzyme that many bacteria used to make cell-walls. a. Amphotericin b. Gentamicin c. Penicillin d. Cephalosporin 88. DDT and Parathion are inhibitors of key enzymes in a. Nervous system b. Respiratory system c. Digestive system d. Circulatory system 89. At high temperature the rate of enzyme action decreases because the increased heat a. Changes the pH of the system b. Alters the active site of the enzyme c. Neutralize acids and bases in the system

d. Increases the concentration of enzymes 90. Which of the following enzymes would digest a fat? a. sucrase b. protease c. Ligase d. lipase 91. In the Lock and Key model of enzyme action, the part of the enzyme that recognizes the substrate is known as the a. Enzyme-substrate complex b. Product c. Enzyme-product complex d. Active site 92. Which model of enzyme action is represented in this diagram.

a. b. c. d.

Fluid mosaic model Induce fit model Lock and key model Reflective index model

93. A certain enzyme will hydrolyze egg white but not starch. Which statement best explains this observation? a. Starch molecules are too large to be hydrolyzed b. Enzyme molecules are specific in their actions c. Egg white acts as a coenzyme for hydrolysis d. Starch is composed of amino acids. 94. At about 00 C, most enzymes are a. Inactive b. Active c. Destroyed d. Replicated 95. Vitamins are essential to the survival of organisms because vitamins usually function as a. Substrates b. Nucleic acids c. Co-enzymes d. Nucleosides

96. When a molecule binds to an area of an enzyme that is not the active site, and changes the shape of the enzyme so that it no longer can work, this is called a. denaturation b. competitive inhibition c. noncompetitive inhibition d. substrate delocation 97. What is a coenzyme? a. Inorganic ion b. Organic molecule c. Both A and B d. None of these 98. Which statement best expresses the information represented in the graph shown?

a. b. c. d.

The action of enzymes varies with pH A pH of 7 provides the optimum environment for digestive enzymes Gastric juice is active at a pH extending from 0 to 12 Acids have a pH greater than 7

99. Which type of inhibitor is shown in this diagram?

a. b. c. d.

Competitive Non-competitive Allosteric Both B and C

100.

Which enzyme represents an enzyme functioning in this reaction?

a. b. c. d.

E C B A

101.

Which of the statements regarding enzymes is false? a. Enzymes are proteins that function as catalysts. b. Enzymes are specific. c. Enzymes provide activation energy for reactions. d. Enzyme activity can be regulated. e. Enzymes may be used many times for a specific reaction.

102.

The relationship between an enzyme and a reactant molecule can best be described as: a. a temporary association. b. an association stabilized by a covalent bond. c. one in which the enzyme is changed permanently. d. a permanent mutual alteration of structure. e. Non-complementary binding.

103.

When [S] = KM, the velocity of an enzyme catalyzed reaction is about: a. 0.1*Vmax. b. 0.2*Vmax. c. 0.3*Vmax. d. 0.5*Vmax. e. 0.9*Vmax.

104.

The active site of an enzyme a. remains rigid and does not change shape. b. is found at the center of globular enzymes. c. is complementary to the rest of the molecule. d. contains amino acids without sidechains. e. None of the above choices are correct.

105. The active site of an enzyme differs from an antibody-antigen binding site in that the enzyme active site a. contains modified amino acids. b. catalyzes a chemical reaction.

c. is complementary to a specific ligand. d. contains amino acids without sidechains. e. None of the above are correct. 106.

Which statement about enzyme catalyzed reactions is NOT true? a. Enzymes form complexes with their substrates. b. Enzymes lower the activation energy for chemical reactions. c. Enzymes change the K eq for chemical reactions. d. Many enzymes change shapes slightly when substrate binds. e. Reactions occur at the "active site" of enzymes, where a precise 3D orientation of amino acids is an important feature of catalysis.

107. The equilibrium constant for the conversion of the disaccharide sucrose to the simple sugars glucose and fructose is 140,000. What can you conclude about the reaction: sucrose + H2O glucose + fructose? a. It is a closed system. b. It never reaches equilibrium. c. It is spontaneous, starting with sucrose. d. The equilibrium constant increases when the starting concentration of sucrose is increased. e. At equilibrium, the concentration of sucrose is much higher than the concentrations of glucose and fructose. 108. To overcome an energy barrier between reactants and products in a chemical reaction, energy must be provided to get the reaction started. This energy, which is recovered as the reaction proceeds, is called: a. activation energy b. initiation energy c. reaction energy d. kinetic energy e. potential energy 109. In the presence of alcohol dehydrogenase, the rate of reduction of acetaldehyde to ethanol increases as you increase the concentration of acetaldehyde. Eventually the rate of the reaction reaches a maximum, where further increases in the concentration of acetaldehyde have no effect. Why? a. All of the alcohol dehydrogenase molecules are bound to acetaldehyde molecules b. At high concentrations of acetaldehyde, the activation energy of the reaction decreases c. The enzyme is no longer specific for acetaldehyde d. At high concentrations of acetaldehyde, the change in free energy of the reaction decreases 110. The equilibrium constant for the ionization of acetic acid, is 0.00002. What can you conclude about this reaction? a. It is a closed system. b. At equilibrium, the concentration of CH3COOH is much higher than the concentrations of CH3COO- + H+. c. It never reaches equilibrium. d. It is spontaneous, starting with CH3COOH.

e. The equilibrium constant increases when the starting concentration of CH3COOH is increased. 111. In describing the reaction rate for a chemical reaction, which of the following statements about reaction rate is NOT true? a. Reaction rate is the speed at which the reaction proceeds toward equilibrium. b. Reaction rate is governed by the energy barrier between reactions and products c. Enzymes can accelerate the rate of a reaction. d. Reaction rates are not sensitive to temperature. e. None of these. 112.

When considering the features of an exergonic reaction, exergonic reactions: a. Release energy b. Are spontaneous reactions c. Have an equilibrium constant greater than 1 d. Can be coupled to energonic reactions. e. All statements are true.

113.

In describing enzyme feature, enzymes: a. are composed primarily of polypeptides, which are polymers of amino acids. b. can bind prosthetic groups such as metal ions that participate in enzyme reactions c. have defined structures. d. bind their substrates at active sites. e. all statements are true

114.

In understanding activation energy, activation energy is a. energy that must be added to get a reaction started, which is recovered as the reaction proceeds. b. difference in energy between reactants and products. c. Energy that is lost as heat. d. Free energy e. Equal to the entropy times the absolute temperature.

115. In thinking about energy that requires reactions in biological systems, energy-requiring reactions can occur in biological systems because enzymes allow their coupling to other reactions with: a. an increase in entropy b. a low activation energy c. no inhibitors d. products of lower free energy than the reactants e. oxidation-reduction 116. In reference to the equilibrium constant for hydrolysis of glucose-6-phosphate. The equilibrium constant for the reaction, glucose 6-phosphate + water glucose + phosphate, is 260. What can you conclude about this reaction: a. It is a closed system b. It never reached equilibrium. c. Starting with glucose 6-phosphate, it is not spontaneous.

d. At equilibrium, the concentration of glucose is much higher than the concentrations of glucose 6-phosphate. e. At equilibrium, the concentration of glucose is much higher than the concentrations 117. When interpreting a ‘S’shaped curve, the graph shows the results of reaction rate vs substrate concentration for a(n):

a. b. c. d. e. 118.

enzyme plus a competitive inhibitor enzyme plus a non-competitive inhibitor allosteric enzyme enzyme plus an un-competitive inhibitor two enzymes competing for the same substrate

Interpreting the plateau of an enzyme kinetics curve:

In the graph reaction rate vs substrate concentration, the reason that the curve reaches a plateau, and does not increase any further at high substrate concentration is that: a. the active site is saturated with substrate b. there is a competitive inhibitor present c. there is a non-competitive inhibitor present d. the allosteric enzyme is locked in an inactive conformation e. all substrate has been converted to product 119. When speaking about energy requiring and energy yielding reactions which answer makes the following statement correct: The result of a(n) __________ reaction is that energy is released. Energy must be added for a(n) __________ reaction to proceed. a. enzyme catalyzed, non-spontaneous b. exergonic, endergonic c. endergonic, spontaneous d. catalytic, non-catalytic e. oxidative, hydrolysis 120.

Which of the following best describes allosteric inhibition of an enzyme? a. The inhibitor binds to the mRNA to prevent translation of the enzyme. b. The inhibitor binds to the enzyme substrate.

c. The inhibitor binds to the enzyme but not at its active site. d. The inhibitor binds to the enzyme at its active site. e. The inhibitor binds to the gene that encodes for the enzyme. 121.

The a. b. c. d. e.

Km (Michaelis constant) of an enzyme for a substrate is defined operationally as half the substrate concentration at which the reaction rate is maximal the substrate concentration at which the reaction rate is half maximal the dissociation constant of the enzyme- substrate complex the dissociation constant of the enzyme-product complex the rate constant of the reaction at saturation

122. During an enzyme assay if the initial velocity Vo = Vmax, then substrate concentration will be a. a Km b. Km c. ½ Km d. 0.2 Km 123.

Phosphofructokinase is allosterically regulated by a. ATP b. ADP c. AMP d. Fructose -6-PO4

124. A double reciprocal plot for an enzymatic reaction with a ping pong Bi mechanism looks similar to the plot obtained for a. Competitive inhibition b. Uncompetitive inhibition c. Non competitive inhibition d. Mixed non competitive inhibition 125.

XH2 + NAD+ → X + NADH + H+. In the preceding equation, NAD+ is said to be: a. b. c. d. e.

126.

an enzyme. storing two hydrogen atoms. reduced. oxidized a catalyst.

Select the reduced molecule: a. b. c. d. e.

NAD+ FAD NADH HNADP+

127. Select the hydrogen acceptor molecule that stores electrons in the process of photosynthesis: a. nicotinamide adenine dinucleotide (NAD+)

b. nicotinamide adenine dinucleotide phosphate (NADP+) c. flavin adenine dinucleotide (FAD) d. adenine triphosphate (ATP) 128.

FAD and cytochromes are classified as: a. b. c. d. e.

hydrogen or electron acceptors. phosphate oxidizers. phosphate reducers. proteins that donate hydrogens or electrons. redox intermediate catalysts.

129. Because enzymes affect the speed of chemical reactions without being consumed, they are referred to as: a. b. c. d. e. 130.

Which of the following statements concerning enzymes is FALSE? a. b. c. d. e.

131.

Each enzyme has an optimal temperature. Each enzyme has an optimal pH. Most enzymes are highly specific. Some enzymes require cofactors. Most enzymes are RNA molecules.

Enzymes are important biological catalysts because they: a. b. c. d. e.

132.

hydrogen acceptors. activation energy. catalysts. cytochromes. transformation proteins.

supply the energy to initiate a biochemical reaction. increase the free energy of a biochemical reaction. lower the entropy of a biochemical reaction. decrease the enthalpy of a biochemical reaction. lower the activation energy of a biochemical reaction.

Which of the following statements concerning activation energy is FALSE? a. b. c. d. e.

133.

Exergonic reactions have an energy of activation. Endergonic reactions have an energy of activation. Enzymes lower a reaction’s activation energy. Catalysts raise a reaction’s activation energy. Activation energy is the energy required to break existing bonds. Use the figure to answer the corresponding questions,

a.

7-6

The Km (Michaelis constant) of an enzyme for a substrate is defined operationally as a. half the substrate concentration at which the reaction rate is maximal b. the substrate concentration at which the reaction rate is half maximal c. the dissociation constant of the enzyme- substrate complex d. the dissociation constant of the enzyme-product complex e. the rate constant of the reaction at saturation 134.

Which bond cannot be observed between enzyme and substate a. Ionic Covalent Bond b. Hydrogen Bond c. Peptide Bond d. Ionic Bond

135. The following data refer to the hydrolysis of various tripeptides into their N-terminal amino acids and C-terminal dipeptides catalysed by intestinal aminopeptidase at pH 7.0 and 37°C. Substrate Kcat (s-1) Km (mM) L-Pro-Gly-Gly 385 1.3 L-Leu-Gly-Gly 190 0.55 L-Ala-Gly-Gly 365 1.4 L-Ala-Ala-Ala 298 0.52

a. b. c. d. 136.

Which of the following substrates would be hydrolysed most rapidly in the early stages of reaction if the enzyme was added to equimolar concentration of the substrates individually? L-Pro-Gly-Gly L-Leu-Gly -Gly L-Ala-Gly-Gly L-Ala-Ala-Ala

Which enzyme causes the release of pyrophosphate from ATP? a. Glycogen phosphorylase b. RNA polymerase

c. ATPase d. Aspartate transcarbamylase 137.

Parts of the enzyme molecule that interact with a substrate are called: a. b. c. d. e.

138.

cofactors. active sites. induced-fit models. orientation sites. reaction sites.

The substance on which an enzyme acts is called the: a. b. c. d.

substrate product ATP. free energy. cofactor.

139. What refers to the situation in which the binding of a substrate to the enzyme causes a change in the enzyme’s shape, facilitating an enzyme’s function? a. b. c. d. e. 140.

active site cofactor activation energy induced fit allosteric inhibition

Hydrolases are one important class of enzyme that function to catalyze: a. b. c. d. e.

splitting a molecule using water. conversions between isomers. reactions in which double bonds are formed. oxidation-reduction reactions. None of these.

141. Which refers to an organic, nonpolypeptide compound that binds to the apoenzyme and serves as a cofactor? a. b. c. d. e.

coenzyme catalyst substrate mineral allosteric regulator

142. Which of the following does not represent a method by which cells regulate enzyme activity? a. b. c. d. e. 143.

controlling the intracellular concentration of the enzyme feedback inhibition of enzymes the binding of allosteric regulators to allosteric sites on the enzyme differential gene expression of the genes that produce enzymes heat denaturation of the enzyme

Select the enzyme that does not match the reaction:

a. b. c. d. e.

sucrase—splits sucrose into glucose and fructose lipase—breaks ester linkages phosphatase—removes phosphate groups urease—converts urea to ammonia and CO2 kinase—breaks peptide bonds

144. If one continues to increase the temperature in an enzyme-catalyzed reaction, the rate of the reaction: a. b. c. d. e.

does not change. increases and then levels off. decreases and then levels off. increases and then decreases rapidly. decreases and then increases rapidly.

145. You conduct an experiment in which you add increasing amounts of substrate to an enzyme solution and then measure the resulting reaction rate. You graph your results, plotting the rate of the reaction on the Y-axis versus substrate concentration on the X-axis. What do you conclude from your graph? a. b. c. d. e. f. 146.

The reaction rate is directly proportional to substrate concentration. The reaction rate is independent of substrate concentration. The reaction rate increases but then decreases, forming a bell-shaped curve. The reaction rate decreases with increasing substrate concentration. The reaction rate increases with increasing substrate concentration up to a point, above which the rate remains constant.

Ibuprofen, a common pain killer, is an inhibitor of a. prostaglandin endoperoxide synthase b. Human Chorionic hormone c. Macrophage synthase d . Erythrocyte dismutase

Answer key

1. E 2. B 3. D 4. A 5. A 6. E 7. E 8. D 9. E 10.E 11.B 12.A 13.A 14.E 15.E 16.B 17.E 18.D 19.D 20.E 21.B 22.B

23.B 24.C 25.D 26.C 27.B 28.A 29.C 30.c 31.d 32.b 33.e 34.b 35.c 36.b 37.c 38.d 39.d 40.a 41.b 42.d 43.a 44.a 45.d 46.b

47.c 48.c 49 .a 50.d 51.c 52.c 53.b 54.c 55.c 56.d 57.d 58.d 59.a 60.d 61.d 62.d 63.a 64.b 65.c 66.a 67.b 68.b 69.c 70.a

71.a 72.b 73.a 74.c 75.c 76.a 77.a 78.a 79.b 80.d 81.d 82.b 83.a 84.c 85.a 86.a 87.c 88.a 89.b 90.d 91.d 92.c 93.b 94.a

95.c 96.c 97.b 98.a 99.d 100.c 101.c 102.a 103.d 104.e 105.b 106.c 107.c 108.a 109.a 110.b 111.d 112.e 113.e 114.a 115.d 116.d 117.c 118.a

119.b 120.c 121.a 122.c 123.a 124.d 125.c 126.c 127.b 128.a 129.c 130.e 131.e 132.d 133.b 134.c 135.a 136.b 137.b 138.a 139.d 140.a 141.a 142.e

143.e 144.d 145.f 146.a

Structure of Atoms, Molecule and Chemical Bonds

1. Which of the following represents the most reduced form of carbon? a. R-CH3 b. R-COOH c. R-CHO d. R-CH2OH e. CO2 2. If red blood cells cultured in an isotonic medium are placed in distilled water, they will most likely a. remain unchanged b. shrivel c. swell and lyse d. divide e. become dormant 3. Which of the following species contains three bond pairs and one lone pair around the central atom? [CBSE AIPMT 2012] a. b. PCl3 c. H2O d. BF3 4. Which one of the following pairs is isostructural (i.e. having the same shape and hybridization)? [CBSE AIPMT 2012] a. [NF3 and BF3] b. c. [BCl3 and BrCl3]4 d. [NH3 and ] 5. In allene (C3H4), the type(s) of hybridisation of the carbon atoms is (are) [IIT JEE 2012] a. sp and sp3 b. sp and sp2 c. Only sp2 d. sp2 and sp3. 6. The sp3d2 hybridization of central atom of a molecule would lead to [West Bengal JEE 2011] a. square planar geometry b. Tetrahedral geometry c. Trigonal bipyramidal geometry d. Octahedral geometry

7. Hybridization of C2 and C3 of H3C – CH = C = CH – CH3 are [West Bengal JEE 2011] a. Sp, Sp3 b. Sp2, Sp c. Sp2, Sp2 d. Sp, Sp 8. Which one of the following is paramagnetic? [West Bengal JEE 2011] a. N2 b. NO c. CO d. O3 9. Among the following the maximum covalent character is shown by the compound [AIEEE 2011] a. MgCl2 b. FeCl2 c. SnCl2 d. AlCl3 10. The hybridization of orbitals of N atom in NO3–, NO2+ and NH4+are respectively [AIEEE 2011] a. sp2, sp3, sp b. sp, sp2, sp3 c. sp2, sp, sp3 d. sp, sp3, sp2 11. The number of types of bonds between two carbon atoms in calcium carbide is [AIEEE 2011] a. Two sigma, two pi b. One sigma, two pi c. One sigma, one pi d. Two sigma, one pi 12. The structure of IF7 is [AIEEE 2011] a. Pentagonal bipyramid b. Square pyramid c. Trigonal bipyramid d. Octahedral 13. Which of the following has maximum number of lone pairs associated with Xe? [AIEEE 2011] a. XeF2 b. XeO3 c. XeF4 d. XeF6

14. Malleability and ductility of metals can be accounted due to[Karnataka CET 2008] a. the presence of electrostatic force b. the crystalline structure in metal c. the capacity of layers of metal ions to slide over the other d. the interaction of electrons with metal ions in the lattice 15. The maximum number of hydrogen bonds that a molecule of water can have is [Karnataka CET 2008] a. 1 b. 2 c. 3 d. 4 16. During the formation of a chemical bond [Karnataka CET 2007] a. energy decreases b. energy increases c. energy of the system does not change d. electron-electron repulsion becomes more than the nucleus-electron attraction 17. The state of hybridization of the central atom and the number of lone pairs over the central atom in POCl3 are [WB JEE 2012] a. sp, 0 b. sp2, 0 c. sp3, 0 d. dsp2, 1 18. NiCl2{P(C2H5)2(C6H5)}2 exhibits temperature dependent magnetic behaviour (paramagnetic/diamagnetic). The coordination geometries of Ni2+ in the paramagnetic and diamagnetic states are respectively [IIT JEE 2012] a. tetrahedral and tetrahedral b. square planar and square planar c. tetrahedral and square planar d. square planar and tetrahedral 19. When O2 is converted into O2+ [Karnataka CET 2011] a. both paramagnetic character and bond order increase b. bond order decreases c. paramagnetic character increases d. paramagnetic character decreases and the bond order increases 20. Which one of the following conversions involves change in both hybridization and shape? [Karnataka CET 2010] a. NH3  NH4+ b. CH4 C2H6 c. H2O  H3O+

d.

BF3  BF4–

21. Assuming that Hund's rule is violated, the bond order and magnetic nature of the diatomic molecule B2 is [IIT JEE 2010] a. 1 and diamagnetic b. 0 and diamagnetic c. 1 and paramagnetic d. 0 and paramagnetic 22. Based on VSEPR theory, the number of 90 degree F — Br — F angles in BrF5 is [IIT JEE 2010] a. 0 b. 2 c. 4 d. 8 23. The species having pyramidal shape is [IIT JEE 2010] a. SO3 b. BrF3 c. SiO32– d. OSF2 24. The value of n in the molecular formula BenAl2Si6O18 is [IIT JEE 2010] a. 3 b. 5 c. 7 d. 9 25. Using MO theory predict which of the following species has the shortest bond length? [AIEEE 2009] a. O2+ b. O2– c. O22– d. O22+ 26. The number of nodal planes present in 2008] a. 1 b. 2 c. 0 d. 3

* s antibonding orbitals is [Karnataka CET

27. Which metal has a greater tendency to form metal oxide?[Karnataka CET 2008] a. Cr b. Fe c. Al

d. Ca 28. Which one of the following constitutes a group of the isoelectronic species? [AIEEE 2008] a. b. c. d. 29. Which one of the following pairs of species have the same bond order? [AIEEE 2008] a. CN− and NO+ b. CN− and CN+ c. O2− and CN− d. NO+ and CN+ 30. For a stable molecule the value of bond order must be[Karnataka CET 2007] a. negative b. positive c. zero d. there is no relationship between stability and bond order 31. In acetylene molecule, between the carbon atoms there are[Karnataka CET 2007] a. three sigma bonds b. two sigma and one pi bonds c. one sigma and two pi bonds d. three pi bonds 32. You are carrying out a research on biological membranes. Your data suggests that Membranes composed of low-molecular weight and non-covalently bonded lipids are thermodynamically stable. What would you predict the possible reason for the observation? a. Hydrogen bonds b. Polarity of lipids c. Polarity of water molecules. d. Ionization coefficient of amino acids in lipid bilayer. 33. Which of the following species contains three bond pairs and one lone pair around the central atom? [CBSE AIPMT 2012] a. NH2b. PCl3 c. H2O d. BF3

34. Which one of the following pairs is isostructural (i.e. having the same shape and hybridization)? [CBSE AIPMT 2012] a. [NF3 and BF3] b. [BF4- and NH4+] c. [BCl3 and BrCl3]4 d. [NH3 and NO-] 35. In allene (C3H4), the type(s) of hybridization of the carbon atoms is (are) [IIT JEE 2012] a. sp and sp3 b. sp and sp2 c. Only sp2 d. sp2 and sp3 36. The sp3d2 hybridization of central atom of a molecule would lead to [West Bengal JEE 2011] a. square planar geometry b. Tetrahedral geometry c. Trigonal bipyramidal geometry d. Octahedral geometry 37. Hybridization of C2 and C3 of H3C – CH = C = CH – CH3 are [West Bengal JEE 2011] a. Sp, Sp3 b. Sp2, Sp c. Sp2, Sp2 d. Sp, Sp 38. Which one of the following is paramagnetic? [West Bengal JEE 2011] a. N2 b. NO c. CO d. O3 39. Among the following the maximum covalent character is shown by the compound [AIEEE 2011] a. MgCl2 b. FeCl2 c. SnCl2 d. AlCl3

40. The hybridization of orbitals of N atom in NO3–, NO2+ and NH4+are respectively [AIEEE 2011] a. sp2, sp3, sp b. sp, sp2, sp3 c. sp2, sp, sp3 d. sp, sp3, sp2 41. The number of types of bonds between two carbon atoms in calcium carbide is [AIEEE 2011] a. Two sigma, two pi b. One sigma, two pi c. One sigma, one pi d. Two sigma, one pi 42. The structure of IF7 is [AIEEE 2011] a. Pentagonal bipyramid b. Square pyramid c. Trigonal bipyramid d. Octahedral 43. Which of the following has maximum number of lone pairs associated with Xe? [AIEEE 2011] a. XeF2 b. XeO3 c. XeF4 d. XeF6 44. Peroxide ion ______. a) is diamagnetic. b) has five completely filled antibonding molecular orbitals. c) is isoelectronic with neon. d) has bond order one. Which one of these is correct? [Karnataka CET 2010] a. a), b) and d) b. d) and c) c. a) and d) d. a), b) and c) 45. Malleability and ductility of metals can be accounted due to[Karnataka CET 2008] a. the presence of electrostatic force

b. the crystalline structure in metal c. the capacity of layers of metal ions to slide over the other d. the interaction of electrons with metal ions in the lattice 46. The maximum number of hydrogen bonds that a molecule of water can have is [Karnataka CET 2008] a. 1 b. 2 c. 3 d. 4 47. The state of hybridization of the central atom and the number of lone pairs over the central atom in POCl3 are [WB JEE 2012] a. sp, 0 b. sp2, 0 c. sp3, 0 d. dsp2, 1 48. NiCl2{P(C2H5)2(C6H5)}2 exhibits temperature dependent magnetic behaviour (paramagnetic/diamagnetic). The coordination geometries of Ni2+ in the paramagnetic and diamagnetic states are respectively [IIT JEE 2012] a. tetrahedral and tetrahedral b. square planar and square planar c. tetrahedral and square planar d. square planar and tetrahedral 49. When O2 is converted into O2+ [Karnataka CET 2011] a. both paramagnetic character and bond order increase b. bond order decreases c. paramagnetic character increases d. paramagnetic character decreases and the bond order increases 50. Which one of the following conversions involve change in both hybridization and shape? [Karnataka CET 2010] a. NH3 → NH4+ b. CH4 → C2H6 c. H2O → H3O+ d. (D) BF3 → BF4–

51. Assuming that Hund's rule is violated, the bond order and magnetic nature of the diatomic molecule B2 is [IIT JEE 2010] a. 1 and diamagnetic b. 0 and diamagnetic c. 1 and paramagnetic d. 0 and paramagnetic 52. Based on VSEPR theory, the number of 90 degree F — Br — F angles in BrF5 is [IIT JEE 2010] a. 0 b. 2 c. 4 d. 8 53. The species having pyramidal shape is [IIT JEE 2010] a. SO3 b. BrF3 c. SiO32– d. OSF2 54. The value of n in the molecular formula BenAl2Si6O18 is [IIT JEE 2010] a. 3 b. 5 c. 7 d. 9 55. Using MO theory predict which of the following species has the shortest bond length? [AIEEE 2009] a. O2+ b. O2– c. O22– d. O22+ 56. The number of nodal planes present in 2008] a. 1 b. 2 c. 0 d. 3

* s antibonding orbitals is [Karnataka CET

57. Which metal has a greater tendency to form metal oxide?[Karnataka CET 2008]

A. B. C. D.

Cr Fe Al Ca

58. Which one of the following constitutes a group of the isoelectronic species? [AIEEE 2008] a. (A) b. (B) c. (C) d. (D) 59. Which one of the following pairs of species have the same bond order? [AIEEE 2008] a. CN− and NO+ b. CN− and CN+ c. O2− and CN− d. NO+ and CN+ 60. For a stable molecule the value of bond order must be[Karnataka CET 2007] a. negative b. positive c. zero d. there is no relationship between stability and bond order 61. In acetylene molecule, between the carbon atoms there are[Karnataka CET 2007] a. three sigma bonds b. two sigma and one pi bonds c. one sigma and two pi bonds d. three pi bonds 62. A.

a. Acetone contains this functional group. b. Dopamine contains this functional group.

B c. Formaldehyde contains this functional group. C

d. Ethanol contains this functional group. D a. b. c. d.

A-3; B-2; C-2; D-4 A-4; B-2; C-3; D-1 A-4; B-3; C-2; D-1 A-2; B-3; C-4; D-1

64. Which of the following is the most direct result of the heating up of pond water during the summer? a. The water’s ability to hold oxygen decreases. b. The water’s ability to act as a buffer changes. c. The viscosity of the water increases. d. Hydrogen bonding at the surface of the water increases. e. Less light penetrates the surface of the water. 65. Which bond cannot be observed between enzyme and substrate a. b. c. d.

Ionic Covalent bond Hydrogen bond Peptide bond Ionic bond

66. Which of the following cannot be considered as weak interaction a. b. c. d.

Hydrogen bonds Vanderwaals forces Peptide bonds Ionic interactions

67. Which of the following bonds will be most difficult to break? a. C-O b. C-C c. C-N d. C-S 68. How many hydrogen bonds can a water molecule potentially take in liquid form? a. one b. two c. three d. four 69. Why H-bonding between water molecules should result in a high heat capacity? a. Energy supplied to water is initially utilized for H-bond breakage.

b. All the energy supplied to water is utilized to raise the internal energy c. Water has no polar molecules d. Conduction of heat in water is radial, so it needs high energy to raise the temperature. 70. Which of the following is true about the newly discovered GOD particle, Higgs bosons a. It explains the presence of GOD, the creator of universe b. It explains the Einstein theory of relativity c. It explains the speed of light d. It explains how particles got their mass. 71. How many electrons are there in 27Al+3 ? a. 27 b. 24 c. 30 d. 32 72. You have an ice weighing 1 kg. You add 10 kg, 15 kg and 20 kg of salt to it. In which case the ice would melt faster? a. 10 kg b. 20kg c. 15kg d. Same rate in all cases. 73. In electric current conduction, what is the source of electrons? a. Electrons in the chemicals of Battery b. Switch c. Electrons in coils of Bulb d. Electrons in Copper wire 74. The actual speed of electrons is 2 m/day! Then why does electric current is so speed? a. Because the electrons are already present in the copper wire b. Because circuits are small c. Because electrons are supplied from the power station, they are of superior quality. d. Electricity is not speed, we feel it speed. 75. carbon tetrachloride has no net dipole moment because of

a. b. c. d.

its planar structure its regular tetrahedral structure similar sizes of carbon and chlorine atoms similar electron affinities of carbon and chlorine

76. Which one among the following does not have the hydrogen bond? a. Phenol b. liquid NH3 c. water d. liquid HCl 77. On hybridization of one s and one p orbital we get: a. Two mutually perpendicular orbital b. Two orbital at 180° c. Four orbital directed tetrahedrally d. Three orbital in a plane

ANSWER KEYS:

1. A 2. C 3. B 4. B 5. B 6. D 7. B 8. B 9. D 10.

C

11.

B

12.

A

13.

A

14.

C

15.

D

16.

A

17.

C

18.

C

19.

D

20.

D

21.

A

22.

A

23.

D

24.

A

25.

D

26.

A

27.

D

28.

B

29.

A

30.

B

31.

C

32.

C

33.

D

34.

A

35.

B

36.

D

37.

B

38.

B

39.

D

40.

C

41.

B

42.

A

43.

A

44.

C

45.

C

46.

D

47.

C

48.

C

49.

D

50.

D

51.

A

52.

A

53.

D

54.

A

55.

D

56.

A

57.

D

58.

B

59.

A

60.

B

61.

C

62.

C

63.

A

64.

C

65.

C

66.

D

67.

D

68.

A

69.

D

70.

B

71.

D

72.

D

73.

A

74.

B

75.

D

76.

B

THERMODYNAMICS AND PRINCIPLES OF BIOPHYSICAL CHEMISTRY 1. Protein molecules carry ionized groups, H-bonding group, and non-polar group on their surfaces. Which type of non-covalent interaction do you suppose is likely to be most important in causing protein molecules in dilute aqueous solution to associate with each other? a. Hydrophobic interactions b. Hydrogen bonding c. Covalent interactions d. Ionic bonds 2. e. f. g. h.

Why H-bonding between water molecules should result in a high heat capacity? Energy supplied to water is initially utilized for H-bond breakage. All the energy supplied to water is utilized to raise the internal energy Water has no polar molecules Conduction of heat in water is radial, so it needs high energy to raise the temperature.

3. a. b. c. d.

Why the steam would burns more than the boiling water? The steam has higher temperature than the water In the steam, water molecules are in gaseous state, can penetrate well in skin Steam has higher latent heat of vaporization Steam expands, so can easily spread around.

4. You are carrying out a research on biological membranes. Your data suggests that Membranes composed of low-molecular weight and non-covalently bonded lipids are thermodynamically stable. What would you predict the possible reason for the observation? a. Hydrogen bonds b. Polarity of lipids c. Polarity of water molecules. d. Ionization coefficient of amino acids in lipid bilayer. 5. How many different polypeptides of 61 residues can be made from the 20 naturally occurring amino acids? a. 61^3 b. Infinite c. 20^3 d. 2 0^61 6. pH is the measure of Hydrogen ions and pKa explains the equilibrium constant in logarithmic scale. If ph is greater than the pKa of an amino acid, then a. More than half of amino acid will be ionized. b. Less than half of amino acid will be ionized. c. No amino acids are ionized d. All amino acids are completely ionized 7. In proteins under normal physiological condition (pH =7) which of the following amino acids will be partially ionized? a. Arginie b. Lysine c. Histidine

d. Serine 8. Cell is a complex thermodynamically balanced entity. Thermodynamically entropy of the system is always increasing, i.e. system becomes increasingly disturbed. Cell/organism becomes more and more organized constantly exchanging the energy and matter from surrounding. Which of the following statements explains the thermodynamics of cell? a. Cell is does not follow laws of thermodynamics, as it is a living being b. Cells create disturbance in the surrounding, not inside the cell c. Cells are open system so do not follow entropy law d. Thermodynamics law has certain exceptions.

a. b. c. d. e.

9. Which of the following accurately represents the relationship between the terms anabolism, catabolism, and metabolism? anabolism = catabolism metabolism = catabolism catabolism = anabolism + metabolism anabolism = catabolism + metabolism metabolism = catabolism + anabolism 10. Catabolic reactions involve the: a. breakdown of large organic molecules to simple building blocks. b. breakdown of life sustaining processes within cells. c. building up of complex organic molecules from simple building blocks. d. anabolic production of complex molecules. e. expenditure of energy.

a. b. c. d.

11. Pathways that have an overall energy requirement are referred to as: catabolic reactions. anabolic reactions. energy-releasing reactions. reactions that will proceed spontaneously. 12. Every type of chemical bond contains a certain amount of energy. The total bond energy, which is essentially equivalent to the total potential energy of the system, is a quantity known as: a. entropy. b. kinetic energy. c. thermodynamic energy. d. enthalpy. e. free energy.

a. b. c. d. e.

13. The equation, G = H - TS, predicts that: as entropy increases, the amount of free energy decreases. as enthalpy increases, the amount of free energy increases. as enthalpy decreases, the amount of entropy also decreases. metabolism decreases proportionately to anabolism. metabolism increases proportionately to catabolism.

a. b.

14. An exergonic reaction is considered to be: spontaneous. potentially spontaneous.

c. d. e.

endergonic. nonspontaneous. energy requiring.

a. b. c. d.

15. A reaction with a negative value of ΔG is referred to as an entropy exergonic enthalpy activation

a. b. c. d. e.

16. An endergonic reaction can proceed only if it absorbs: more free energy than is released by a coupled exergonic reaction. less free energy than was released by a coupled endergonic reaction. less free energy than is released by a coupled exergonic reaction. the same amount of free energy that is absorbed by the enzymatic breakdown of proteins. energy from ADP, forming ATP.

a. b. c. d. e.

17. In a reaction in which the rate of the reverse reaction is equal to the rate of the forward reaction, a state of is attained. total entropy enthalpy thermodynamics dynamic equilibrium product reversibility

a. b. c. d.

18. Which of the following statements about the accompanying figure is true? The reactants have more free energy than the products. The products have more free energy than the reactants. The figure represents a spontaneous reaction. The figure represents an endergonic reaction.

a. b. c. d. e.

19. Which of the following conclusions can be accurately derived from the associated figure? ΔS is positive. ΔH equals zero. ΔG is positive. ΔG is negative. ΔT is negative.

reaction.

a. b. c. d. e.

20. Energy stored within the molecules of ATP is in the form of kinetic heat potential nuclear light

a. b. c. d.

21. Consider the following two chemical equations: A) glucose + fructose → sucrose + H2O, ΔG = +27kJ/mole (or +6.5 kcal/mole) B) glucose + fructose + ATP → sucrose + ADP + Pi, ΔG = -5kJ/mole (or -1.2 kcal/mole) The free energy change difference between the chemical equations (A) and (B) above is accomplished by: a decrease in activation energy. combining two endergonic reactions. combining an endergonic and an exergonic reaction. combining two exergonic reactions.

a. b. c. d.

22. Which of the following statements concerning ATP is FALSE? It is a nucleotide. It is called the energy currency of the cell. It contains phosphate groups joined in a series. It contains phosphate groups joined by unstable bonds..

a. b. c. d. e.

23. Select the phosphorylation reaction: glucose + fructose  sucrose + H2O glucose + ATP glucose-P + ADP glucose-P + fructose  sucrose + Pi glucose + glucose  maltose None of these are phosphorylation reactions.

a. b. c. d. e.

24. The maintenance of a high ATP to ADP ratio within cells ensures that: the hydrolysis of ADP to ATP will be strongly exergonic. the hydrolysis of ATP to ADP will be strongly exergonic. the hydrolysis of ATP to ADP will be strongly endergonic. the hydrolysis of ADP to ATP will be an energy releasing reaction. the conversion of ADP to ATP will proceed spontaneously. 25. The

a. b. c. d. e.

transfer

of

electrons

from

one

compound

energy.

to

another

Transfer. heat oxygen enzymatic phosphorus energy 26.

Is a process where energy (as electrons) is released, whereas

is

equivalent

to

is a process where energy (as electrons) is accepted. Reduction; oxidation Enthalpy; entropy Entropy; enthalpy Oxidation; reduction Anabolism; catabolism

a. b. c. d. e. e. f. g. h.

27. Which of the following is true about the newly discovered GOD particle, Higgs bosons It explains the presence of GOD, the creator of universe It explains the Einstein theory of relativity It explains the speed of light It explains how particles got their mass.

28. e. f. g. h.

How many electrons are there in 27Al+3 ? 27 24 30 32

29. In winter seasons in northern hemisphere, we use road salt to make away through the ice on road. What strategy can you suggest that we can use to melt the ice on road? a. Addition of Cacl2 b. Addition of liquid water c. Addition of NaOH d. Wait till the winter gets over and ice melts automatically 30. a. b. c. d.

Addition of salt to the liquid water will Increase the freezing temperature Decrease the melting point Increases the melting point Increase the latent heat of melting

31. You have an ice weighing 1 kg. You add 10 kg, 15 kg and 20 kg of salt to it. In which case the ice would melt faster? e. 10 kg f. 20kg g. 15kg h. Same rate in all cases. 32. e. f. g. h.

In electric current conduction, what is the source of electrons? Electrons in the chemicals of Battery Switch Electrons in coils of Bulb Electrons in Copper wire

33. e. f. g. h.

The actual speed of electrons is 2 m/day! Then why does electric current is so speed? Because the electrons are already present in the copper wire Because circuits are small Because electrons are supplied from the power station, they are of superior quality. Electricity is not speed, we feel it speed.

34. The phosphate buffers are used in research. Na and k are highly soluble in phosphate buffers. So any ratio of Na and k ions can be selected. But a. It is impossible to prepare the phosphate buffer with high buffering capacity and low ionic strength b. It is easy to prepare the phosphate buffer with high buffering capacity and low ionic strength c. It is impossible to prepare the phosphate buffer with high buffering capacity and low ionic strength d. It is easy to prepare the phosphate buffer with low ionic strength and high buffering capacity 35. e. f. g. h.

Carbon tetrachloride has no net dipole moment because of its planar structure its regular tetrahedral structure similar sizes of carbon and chlorine atoms similar electron affinities of carbon and chlorine

36. e. f. g. h.

Which one among the following does not have the hydrogen bond? Phenol liquid NH3 water liquid HCl

37. d. e. f. g.

On hybridization of one s and one p orbital we get: Two mutually perpendicular orbital Two orbital at 180° Four orbital directed tetrahedrally Three orbital in a plane

38. Acid and bases are the important chemicals, which maintain the amount of H+ ions and OH-ions in any solution. Strong acids, dissociate completely in water and weak acids dissociate partially. Which of the following is the primary reason for the different acid strength? a. Ionic state b. Ionization constant c. Molecular structure d. Chemical formula 39. Reverse osmosis is widely used to purify water, mixed with solutes. It is widely used in salt water purifiers. Which of the following explains best the process of reverse osmosis? a. Removal of solvent form solution b. Removal of solute from solvent c. Addition of solute to the solvent d. Addition of solvent to the solution. 40. In a reversible reaction between A and B, k1 is the rate constant for the formation of B from A, and k–1 is the rate constant for the formation of A from B. What is the rate equation if both reactions are first order with respect to the reactant? a. Rate of disappearance of A = k1[A] – k–1[B] b. Rate of disappearance of A = k1[A] + k–1[B] c. Rate of disappearance of A = k–1 [A] – k1[B] d. Rate of disappearance of A = k–1 [A] + k1[B]

41. The half-life of 222Rn is 3.8 days. A sample of 222Rn is allowed to decay for 15.2 days and after this time, 5.6 mmol remained. What was the initial quantity of 222Rn? a. 0.090 mol b. 0.045 mol c. 45 mmol d. 0.90 mol 42. For the reaction: A + B → C + D the reaction rate halves when the concentration of A halves (B is in excess), and when A is in excess, the reaction rate is unaffected by changes in the concentration of B. Which equation is consistent with these observations? a. Rate of reaction = k[A][B] b. Rate of reaction = k[A] c. Rate of reaction = k[A]2[B] d. Rate of reaction = k[A]2 43. Consider the following series of elementary steps which together give a proposed mechanism for an overall reaction: 2NO → N2O2 rate constant = k1 N2O2 → 2NO rate constant = k–1 N2O2 + O2 → 2NO2 rate constant = k2 Which statement is incorrect? a. The K2 step consumes an intermediate species. b. The overall reaction is: 2NO + O2 → 2NO2 c. Applying the steady state approximation sets [N2O2] = 0 d. The k1 step is bimolecular. 44. ATP is the major energy source in living cells. Assuming a cellular concentration of 0.5 mM. Calculate how many molecules of ATP would be present in our hypothetical (and spherical) eukaryotic cell. (Avogadro’s number, the number of molecules in 1 mol of a non ionized substance, is 6.02x 10 23, volume of the cell is 6.5 x 10-8 mL) a. 3.9 x 10-10 b. 3.9 x 10+10 c. 1.75 x 10-10 d. 1.75 x 10+10 45. In the reactions given below, identify the species undergoing oxidation: (i) H2S (g) + Cl2 (g)  2 HCl (g) + S (s) (ii) 3Fe3O4 (s) + 8 Al (s)  9 Fe (s) + 4Al2O3 (s) (iii) 2 Na (s) + H2 (g)  2 NaH (s) a. H2S, Al, H2 b. H2S, Fe3O4, Na c. NaH , Al2O3, , H2 d. Al, Cl2, H2 46. One way to appreciate the structure of water is by comparing it to H2S. Like oxygen, sulfur has six outer-shell electrons and forms single bonds with two hydrogen atoms. But because sulfur is a larger atom, it is less electronegative than oxygen. At room temperature, H2S is a gas, not a liquid. What is the reason for this anomality, even though it has similar molecular structure as water? a. Reduced hydrogen bonding capacity b. Reduced Steric hindrance in water molecules, making them stable

c. Gravitational force is less on H2S molecules d. Water has Oxygen atom which is getting reduced. 47. Sweating is a physiological process involved in cooling the body. The sweat is secreted by sweat glands and helps in reduction of body heat. How do you think sweat actually reduces body surface temperature? a. Sweat has temperature reducing hormones which signals hypothalamus to reduce body temperature. b. Sweat can form alkyl bonds which removes the heated chemicals c. Water in the sweat evaporates, absorbing the heat from surrounding d. Water in the sweat absorbs heat and gives out to environment. 48. Atoms have protons and neutrons in nucleus, and electrons in the outer space. Atomic mass comprises of the total no. of protons and neutrons. In a hypothetical atom, 81Gh, no. of neutrons is 31.4% more than the no. of protons. Find out the no. of electrons in the uncharged electron. a. 46 b. 10 c. 32 d. 35 49. If the diameter of a carbon atom is 0.15 nm, calculate the number of carbon atoms which can be placed side by side in a straight line across length of scale of length 20 cm long. a. 2.33 x 109 b. 1 x 1019 c. 1.33 x 109 d. 1.33 x 1029 50. Matter exists in 3 forms on earth. Solids, Liquid and Gases. Each has different physical characteristics and different chemical reactions. Solids are more stable and has specific structure compared to other forms. Why in room temperature, Solid remains as solid and will not spontaneously converted into other state? a. Entropy needed to reduce, which needs external energy b. Atoms of solid are stable at room temperature c. Atoms have a lone pair of electron, which makes the solids stable in room temperature d. Molecular bonds are stable and conversion to liquid needs external energy source. 51. Silicon is in the same group of the periodic table as carbon and, like carbon, can form up to four single bonds. Many science fiction stories have been based on the premise of silicon-based life. Is this realistic? What characteristics of silicon make it less well adapted than carbon as the central organizing element for life? a. Yes it is realistic and life can consider silicon as major macro molecule b. No, It’s all rubbish story, silicon was not present when life started c. Yes, Silicon can be a possibility but presence of Oxygen in atmosphere, makes it impossible. d. Yes, it is possible, but carbon was selected by a chance factor, as explained by Charles Darwin, theory of evolution. 52. A. B. C.

Which of the following quantum number is not possible? n = 0, l = 0, ml = 0, ms = + _ n = 1, l = 0, ml = 0, ms = – _ n = 1, l = 1, ml = 0, ms = + _

D. a. b. c. d.

n = 2, l = 1, ml = 0, ms = – _ Only A Both A and C Only C Both A and B

53. A polypeptide chain has 300 amino acids, each weighing 110 amu (μ). What is the weight of the polypeptide in terms of Kilogram? a. 5.18 x 10-23 kg b. 518 x 10-22kg c. 33000 x 10-27Kg d. 33 x 10-24 Kg 54. a. b. c. d.

An electron is present in 3d orbital. How many possible “m” values it can have? 2 5 8 1

55. The gecko is a reptile with an amazing ability to climb smooth surfaces, including the glass. The attachment is so firm, that even a full grown adult man can hold onto its tail and yet the gekho does not loosen its grip. Interestingly, the gekho can easily lift its hand for upward movement inferring that the firm attachment is only temporary. What may be a reason for such firm attachment? a. The gravitational pull exerted by the earth on gekho is balanced by the pull of the wall. b. The gekho secrets some acids which dissolve the wall/ glass making it firm adhesive material. c. The atoms of the gekho feet and atoms of glass form temporary but strong inter atomic interactions. d. The atoms of gekho feet and glass form stronger covalent bonds. 56. To prepare a phosphate buffer, you weighed Sodium di-hydrogen phosphate and Disodium hydrogen phosphate in equal amount, in 1L of water. What would be the pH of the buffer, if pKa of phosphoric acid id 4.76. a. 7.46 b. 3.78 c. 4.76 d. 5.28 57. Electromotive force(e.m.f) is a measure of charge of solution. When we dip a electrodes in solutions, they pick up some electric charges from the solution. The charge development depends on its pH content. In a saturated calomel electrode pH = e.m.f - 0.246/ 0.059 . When the electrodes are dipped in a solution the voltmeter reads 0.652. What is the pH if the solution? a. 6.9 b. 6.0 c. 9.6 d. 0.69 58. Which of the following aqueous solutions has the lowest pH: 0.1 M HCl; 0.1 M acetic acid (pKa 4.86); 0.1 M formic acid (pKa 3.75)? a. 0.1 M HCl b. 0.1 M acetic acid c. 0.1 M formic acid

d. Homogenous mixture of all acids 59. Calculate the ration of concentrations of HCO-3 to H2CO3 at blood pH? Take the pKa of H2CO3 to be 6.4 a. 1 b. 20 c. 10 d. 15 60. a. b. c. d.

Why addition of EDTA is recommended while working with DNA? EDTA dissolves DNA EDTA inhibits nucleases , as it is toxic for nucleases EDTA chelates Mg+2 which is a cofactor of nucleases EDTA intercalates the DNA, changing its structure

61. In a hospital laboratory, a 10.0 mL sample of gastric juice, obtained several hours after a meal, was titrated with 0.1 M NaOH to neutrality; 7.2 mL of NaOH was required. The patient’s stomach contained no ingested food or drinks, thus assume that no buffers were present. What was the pH of the gastric juice? a. 2.1 b. 3.1 c. 1.1 d. 0.0 62. A. B. C. D. a. b. c. d.

Which is the conjugate base in each of the pairs below? RCOOH, RCOO_ RNH2, RNH3 _ H2PO4 _ , H3PO4 H2CO3, HCO3 A. RCOO_ B. RNH2 C. H2PO4 _ D. HCO3 A. RCOOH B. RN2H2 C. H2PO4 D. H2CO3 A. RCOO- B. RNH3 C. H3PO4 D. H2CO3 RCOO- B. RNH3 C. H3PO4 D. HCO3

63. Which of the following chemicals ionize more in low ph conditions?

a. b. c. d.

Pyridine ion B-naphthol Both same Cannot be determined

64. Aspirin is a weak acid with a pKa of 3.5. It is absorbed into the blood through the cells lining the stomach and the small intestine. Absorption requires passage through the plasma membrane, the

a. b. c. d.

rate of which is determined by the polarity of the molecule: charged and highly polar molecules pass slowly, whereas neutral hydrophobic ones pass rapidly.it becomes neutral at pH 2.5. The pH of the stomach contents is about 1.5, and the pH of the contents of the small intestine is about 6. Is more aspirin absorbed into the bloodstream from the stomach or from the small intestine? Stomach Small intestine Absorption happens in both cases as both are the part of digestive system In kidney

65. a. b. c. d.

Which one among the CH3, NH3 and NF3 has higher dipole moment? Both same NF3 NH3 CH3

66. a. b. c.

Which of the following statement is NOT true? The cannonical forms have no real existence. The electrons collapse into the nucleus, if driven out of all its energy. The molecule does not exist for a certain fraction of time in one cannonical form and for other fractions of time in other canonical forms. d. There is no such equilibrium between the cannonical forms as we have between tautomeric forms (keto and enol) in tautomerism. 67. We have 2 lactic acid buffers, one with 10 ml of 0.1 M lactate and 10 ml of 0.1N lactic acid. Another buffer has 10 ml of 0.025M lactate and 0.025 M lactic acid. Which of them will show higher buffer capacity on addition of 1 ml of 0.1M HCl solution? a. Buffer 2 because it has low acid/ base ratio b. Buffer 2 because it has low acid content c. Buffer 1 because its morality of lactic acid buffer is equal to HCL added d. Buffer 1 because its morality is not equal 68. For the reaction A + B  C, the rate constant at 215 oC is 5.0 x 10-3 and the rate constant at 452o C is 1.2 x 10-1.What is the rate constant at 100 o C a. 2.50 x 10-4 s-1 b. 1.5 x 10-4 s-1 c. 0.5 x 10-4 s-1 d. None of the above 69. Calculate the pH of a 0.01M solution of acetic acid and its fractional ionisation given that its Ka = 1.75 x 10-5 a. 3.38 b. 4.38 c. 6.5 d. 8.2 70. The plot of concentration of product formed in a hypothetical chemical reaction versus time follows an exponential curve. What will be the shape of the plot of the net forward rate of the reaction versus time? a. Exponential b. Linear

c. Parabolic d. Hyperbolic 71. a. b. c. d.

What is the effective H3O+ concentration in a 10 mM phosphate buffer at pH 4.0? 10-4 x 10-14 M 10-4 + 10-7 M 10-4 M None of the above

72. a. b. c. d.

Why the steam would burns more than the boiling water? The steam has higher temperature than the water In the steam, water molecules are in gaseous state, can penetrate well in skin Steam has higher latent heat of vaporization Steam expands, so can easily spread around.

73. In winter seasons in northern hemisphere, we use road salt to make away through the ice on road. What strategy can you suggest that we can use to melt the ice on road? a. Addition of CaCl2. b. Addition of liquid water. c. Addition of NaOH. d. Wait till the winter gets over and ice melts automatically. 74. a. b. c. d.

Addition of salt to the liquid water will Increase the freezing temperature. Decrease the melting point. Increases the melting point. Increase the latent heat of melting.

75. Matter exists in 3 forms on earth. Solids, Liquid and Gases. Each has different physical characteristics and different chemical reactions. Solids are more stable and has specific structure compared to other forms. Why in room temperature, Solid remains as solid and will not spontaneously converted into other state? a. Entropy needed to reduce, which needs external energy b. Atoms of solid are stable at room temperature c. Atoms have a lone pair of electron, which makes the solids stable in room temperature d. Molecular bonds are stable and conversion to liquid needs external energy source. 76. Silicon is in the same group of the periodic table as carbon and, like carbon, can form up to four single bonds. Many science fiction stories have been based on the premise of silicon-based life. Is this realistic? What characteristics of silicon make it less well adapted than carbon as the central organizing element for life? a. Yes it is realistic and life can consider silicon as major macro molecule b. No, It’s all rubbish story, silicon was not present when life started c. Yes, Silicon can be a possibility but presence of Oxygen in atmosphere, makes it impossible. d. Yes, it is possible, but carbon was selected by a chance factor, as explained by Charles Darwin, theory of evolution. 77. Reverse osmosis is widely used to purify water, mixed with solutes. It is widely used in salt water purifiers. Which of the following explains best the process of reverse osmosis? a. Removal of solvent form solution

b. Removal of solute from solvent c. Addition of solute to the solvent d. Addition of solvent to the solution. 78. a. b. c. d.

How much of 8X buffer you need, to prepare a solution of 640ml of 0.5X concentration? 20ml 40ml 100ml 50ml

79. pH is the measure of Hydrogen ions and pKa explains the equilibrium constant in logarithmic scale. If ph is greater than the pKa of an amino acid, then a. More than half of amino acid will be ionized. b. Less than half of amino acid will be ionized. c. No amino acids are ionized d. All amino acids are completely ionized 80. The phosphate buffers are used in research. Na and k are highly soluble in phosphate buffers. So any ratio of Na and k ions can be selected. But a. It is impossible to prepare the phosphate buffer with high buffering capacity and low ionic strength b. It is easy to prepare the phosphate buffer with high buffering capacity and low ionic strength c. It is impossible to prepare the phosphate buffer with high buffering capacity and low ionic strength d. It is easy to prepare the phosphate buffer with low ionic strength and high buffering capacity 81. Acid and bases are the important chemicals, which maintain the amount of H+ ions and OH-ions in any solution. Strong acids, dissociate completely in water and weak acids dissociate partially. Which of the following is the primary reason for the different acid strength? a. Ionic state b. Ionization constant c. Molecular structure d. Chemical formula 82. A reaction; A + B  C + D; Keq = 2 x 10-5, the entropy of the reaction has not changed, and the reaction is just a isomerization step. What is the enthalpy/ bond energy of the isomerization step? ( R = 8.314 J/mol.K ; T = 25C ) a. 24 KJ/mol b. 50Kj/mol c. 24 J /mol d. 0.24 kj/mol. 83. Cell is a complex thermodynamically balanced entity. Thermodynamically entropy of the system is always increasing, i.e. system becomes increasingly disturbed. Cell/organism becomes more and more organized constantly exchanging the energy and matter from surrounding. Which of the following statements explains the thermodynamics of cell? a. Cell is does not follow laws of thermodynamics, as it is a living being b. Cells create disturbance in the surrounding, not inside the cell c. Cells are open system so do not follow entropy law d. Thermodynamics law has certain exceptions.

84. In a reversible reaction between A and B, k1 is the rate constant for the formation of B from A, and k–1 is the rate constant for the formation of A from B. What is the rate equation if both reactions are first order with respect to the reactant? a. Rate of disappearance of A = k1[A] – k–1[B] b. Rate of disappearance of A = k1[A] + k–1[B] c. Rate of disappearance of A = k–1 [A] – k1[B] d. Rate of disappearance of A = k–1 [A] + k1[B] 85. The half-life of 222Rn is 3.8 days. A sample of 222Rn is allowed to decay for 15.2 days and after this time, 5.6 mmol remained. What was the initial quantity of 222Rn? a. 0.090 mol b. 0.045 mol c. 45 mmol d. 0.90 mol 86. For the reaction: A + B → C + D the reaction rate halves when the concentration of A halves (B is in excess), and when A is in excess, the reaction rate is unaffected by changes in the concentration of B. Which equation is consistent with these observations? a. Rate of reaction = k[A][B] b. Rate of reaction = k[A] c. Rate of reaction = k[A]2[B] d. Rate of reaction = k[A]2 87. Consider the following series of elementary steps which together give a proposed mechanism for an overall reaction: 2NO → N2O2 rate constant = k1 N2O2 → 2NO rate constant = k–1 N2O2 + O2 → 2NO2 rate constant = k2 Which statement is incorrect? a. The K2 step consumes an intermediate species. b. The overall reaction is: 2NO + O2 → 2NO2 c. Applying the steady state approximation sets [N2O2] = 0 d. The k1 step is bimolecular. 88. In the reactions given below, identify the species undergoing oxidation: (i) H2S (g) + Cl2 (g)  2 HCl (g) + S (s) (ii) 3Fe3O4 (s) + 8 Al (s)  9 Fe (s) + 4Al2O3 (s) (iii) 2 Na (s) + H2 (g)  2 NaH (s) a. H2S, Al, H2 b. H2S, Fe3O4, Na c. NaH , Al2O3 , H2 d. Al, Cl2, H2 89. A. B. C. a. b.

How does the hydrophobic effect influence the structures of large molecules? Nonpolar molecules are not easily solubilized in water and aggregate Polar groups are oriented on the surface, interacting with the water Nonpolar molecules can mask the polar characteristics of the hydrophilic molecules A and B A and C

90. Most components of energy conversion systems evolved very early; thus, the most fundamental aspects of energy metabolism tend to be: a. Quite different among a diverse group of organisms. b. Very different among plants and animals. c. The same among the autotrophs but different among heterotrophs. d. The same among prokaryotes but different among eukaryotes. e. Very similar in a wide range of different organisms. 91. An organism can exchange matter and energy with its surroundings. Thus, any change in an organism’s energy content must be balanced by a corresponding change in the energy content of the surroundings. As such, an organism is referred to as: a. a closed system. b. an open system. c. a dynamic system. d. a thermally reactive system. e. a potential system. 92. Only 20% to 30% of the energy stored in the chemical bonds of gasoline molecules is transformed into mechanical energy; the other 70% to 80% is dissipated as waste heat. Which statement explains this phenomenon? a. The first law of thermodynamics. b. The second law of thermodynamics. c. When energy is converted from one form to another, some of the energy is converted into heat. d. Both the first and second laws of thermodynamics. e. Both the second law of thermodynamics, as well as the observation that the conversion of energy involves some energy being converted to heat. 93. a. b. c. d. e.

In order for a cell to maintain a high degree of order, it must: constantly release energy. constantly produce energy. constantly destroy energy. constantly use energy. constantly increase energy.

94. Which of the following accurately represents the relationship between the terms anabolism, catabolism, and metabolism? a. anabolism = catabolism b. metabolism = catabolism c. catabolism = anabolism + metabolism d. anabolism = catabolism + metabolism

e. metabolism = catabolism + anabolism 95. a. b. c. d. e.

Which of the following statements concerning ATP is FALSE? It is a nucleotide. It is called the energy currency of the cell. It contains phosphate groups joined in a series. It stores energy for long periods. It contains phosphate groups joined by unstable bonds.

96. a. b. c. d. e.

Select the phosphorylation reaction: glucose + fructose  sucrose + H2O glucose + ATP  glucose-P + ADP glucose-P + fructose  sucrose + Pi glucose + glucose  maltose None of these are phosphorylation reactions.

97. a. b. c. d. e.

The maintenance of a high ATP to ADP ratio within cells ensures that: The hydrolysis of ADP to ATP will be strongly exergonic. The hydrolysis of ATP to ADP will be strongly exergonic. The hydrolysis of ATP to ADP will be strongly endergonic. The hydrolysis of ADP to ATP will be an energy releasing reaction. The conversion of ADP to ATP will proceed spontaneously.

98. The transfer of electrons from one compound to another is equivalent to transfer. a. heat b. oxygen c. enzymatic d. phosphorus e. energy 99. ---- is a process where energy (as electrons) is released, whereas-----is a process where energy (as electrons) is accepted. a. Reduction; oxidation b. Enthalpy; entropy c. Entropy; enthalpy d. Oxidation; reduction e. Anabolism; catabolism 100. XH2 + NAD+ → X + NADH + H+. In the preceding equation, NAD+ is said to be: a. an enzyme. b. Storing two hydrogen atoms. c. reduced. d. oxidized. e. a catalyst. 101. Select the reduced molecule: a. NAD+ b. FAD

c. NADH d. He. NADP+ 102. Select the hydrogen acceptor molecule that stores electrons in the process of photosynthesis: a. nicotinamide adenine dinucleotide (NAD+) b. nicotinamide adenine dinucleotide phosphate (NADP+) c. flavin adenine dinucleotide (FAD) d. adenine triphosphate (ATP) e. adenine diphosphate (ADP) 103. FAD and cytochromes are classified as: a. hydrogen or electron acceptors. b. phosphate oxidizers. c. phosphate reducers. d. proteins that donate hydrogens or electrons. e. redox intermediate catalysts. 104. Enzymes are important biological catalysts because they: a. supply the energy to initiate a biochemical reaction. b. increase the free energy of a biochemical reaction. c. lower the entropy of a biochemical reaction. d. decrease the enthalpy of a biochemical reaction. e. lower the activation energy of a biochemical reaction. 105. Which of the following statements concerning activation energy is FALSE? a. Exergonic reactions have an energy of activation. b. Endergonic reactions have an energy of activation. c. Enzymes lower a reaction’s activation energy. d. Catalysts raise a reaction’s activation energy. e. Activation energy is the energy required to break existing bonds. 106. Which of the following statements concerning activation energy is FALSE? a. Exergonic reactions have an energy of activation. b. Endergonic reactions have an energy of activation. c. Enzymes lower a reaction’s activation energy. d. Catalysts raise a reaction’s activation energy. e. Activation energy is the energy required to break existing bonds. Use the figure to answer the corresponding questions, 102 & 103

7-6

107. The line on the graph labeled B represents the: a. Activation energy with an enzyme. b. Activation energy without an enzyme. c. Free energy of the reactants. d. change in entropy. e. change in enthalpy. 108. The line on the graph labeled C represents the: a. Activation energy with an enzyme. b. Activation energy without an enzyme. c. Change in free energy. d. Change in entropy. e. Change in enthalpy. 109. The nucleoside adenosine exists in a protonated form with a pKa of 3.8. The percentage of the protonated form at pH 4.8 is closest to a. 1 b. 9 c. 50 d. 91 e. 99 110. The free energy ΔG of a dissolved solute a. Increases with solute concentration. b. Decreases with solute concentration c. Is independent of solute concentration. d. Depends only on temperature. 111. Equilibrium constant (K) of non-covalent interaction between two non-bonded atoms of two different groups was measured at 27 degree Celsius. It was observed that K = 100M -1. The strength of this noncovalent interaction in terms of Gibbs free energy change is a. 2746 kcal/mole b. -2746 cal/mole c. 247 kcal/mole d. -247 cal/mole

111. A student conducted an experiment where CO2 and N2 were bubbled through water in beakers A and B respectively. He recorded the pH in each of the solutions every 5 min. What is the most valid conclusion the student could draw from these results: pH reading

a. b. c. d.

Time (min)

A

B

5

7.5

7.5

10

7.2

7.3

15

7.0

7.5

20

6.8

7.4

The change in pH was too small to be significant Bubbling CO2 through water makes it more acidic Bubbling N2 through water makes it more acidic Both CO2 and N2 make water more acidic

112. What volume of 20% sucrose would you use to make 2mL of 5% sucrose solution a. 0.0005 mL b. 0.05 mL c. 0.5 mL d. 0.005 mL 113. Cell is a complex thermodynamically balanced entity. Thermodynamically entropy of the system is always increasing, i.e. system becomes increasingly disturbed. Cell/organism becomes more and more organized constantly exchanging the energy and matter from surrounding. Which of the following statements explains the thermodynamics of cell? a. Cell is does not follow laws of thermodynamics, as it is a living being b. Cells create disturbance in the surrounding, not inside the cell c. Cells are open system so do not follow entropy law d. Thermodynamics law has certain exceptions. 114. In an intact cell, free energy change associated with an enzyme catalyzed reaction is frequently different from the standard free energy change of the same reaction because in the intact cell the a. activation energy is different b. reaction is always near equilibrium c. enzyme may be regulated allosterically d. reactants are not at 1M concentration e. reaction may be catalyzed by more than one enzyme 115. Which of the following can act as nucleophile in metabolic reactions? I. Nitrogen of an amino group II. Oxygen of hydroxyl group III. Carbon of carbonyl group a. I only b. II only

c. III only d. I and II only e. I, II and III 116. Under what thermodynamic condition reaction would be spontaneous? a. delta H > 0 and delta s > 0 b. delta H < o and delta S > 0 c. delta H < 0 and delta s < 0 d. delta H > T delta s 117. Efficiency of ATP synthesis is 40% and enthalpy of reaction ADP + Pi giving ATP is eight an individual consumes 2000 Kcal, then the net ATP production from it would be a. 40 b. 100 c. 250 d. 1000

KCal. If

118. When Keq = 1, standard free energy change is equal to a. -1 b. 0 c. + 1 d. 10 119. The ultimate source of energy for almost all living organisms is: a. heat. b. glucose. c. carbohydrates d. the sun. 120. Kilojoules are: a. units of heat energy. b. units of matter. c. units of work. d. units of kinetics. e. units of mechanical change. 121. Only 20% to 30% of the energy stored in the chemical bonds of gasoline molecules is transformed into mechanical energy; the other 70% to 80% is dissipated as waste heat. Which statement explains this phenomenon? a. The first law of thermodynamics. b. The second law of thermodynamics. c. When energy is converted from one form to another, some of the energy is converted into heat. d. Both the first and second laws of thermodynamics. e. Both the second law of thermodynamics, as well as the observation that the conversion of energy involves some energy being converted to heat. 122. If the ∆G'° of the reaction A → B is –40 kJ/mol, under standard conditions the reaction: a. is at equilibrium. b. will never reach equilibrium. c. will not occur spontaneously.

d. will proceed at a rapid rate. e. will proceed spontaneously from left to right. 123. For the reaction A → B, ∆G'° = –60 kJ/mol. The reaction is started with 10 mmol of A; no B is initially present. After 24 hours, analysis reveals the presence of 2 mmol of B, 8 mmol of A. Which is the most likely explanation? a. b. c. d.

A and B have reached equilibrium concentrations. An enzyme has shifted the equilibrium toward A. B formation is kinetically slow; equilibrium has not been reached by 24 hours. Formation of B is thermodynamically unfavorable.

124. When a mixture of 3-phosphoglycerate and 2-phosphoglycerate is incubated at 25 °C with phosphoglycerate mutase until equilibrium is reached, the final mixture contains six times as much 2-phosphoglycerate as 3-phosphoglycerate. Which one of the following statements is most nearly correct, when applied to the reaction as written? (R = 8.315 J/mol·K; T = 298 K) 3-Phosphoglycerate → 2-phosphoglycerate a. ∆G'° is –4.44 kJ/mol. b. ∆G'° is zero. c. ∆G'°is +12.7 kJ/mol. d. ∆G'°is incalculably large and positive. 125. When a mixture of glucose 6-phosphate and fructose 6-phosphate is incubated with the enzyme phosphohexose isomerase (which catalyzes the interconversion of these two compounds) until equilibrium is reached, the final mixture contains twice as much glucose 6-phosphate as fructose 6phosphate. Which one of the following statements is best applied to this reaction outlined below?(R = 8.315 J/mol•K; T = 298 K) Glucose 6-phosphate → fructose 6-phosphate a. ∆G'° is incalculably large and negative. b. ∆G'° is –1.72 kJ/mol. c. ∆G'° is zero. d. ∆G'° is +1.72 kJ/mol. 126. Hydrolysis of 1 M glucose 6-phosphate catalyzed by glucose 6-phosphatase is 99% complete at equilibrium (i.e., only 1% of the substrate remains). Which of the following statements is most nearly correct? (R = 8.315 J/mol•K; T = 298 K) a. ∆G'° = –11 kJ/mol b. ∆G'° = –5 kJ/mol c. ∆G'° = 0 kJ/mol d. ∆G'° = +11 kJ/mol 127. The reaction A + B  C has a ∆G'° of –20 kJ/mol at 25° C. Starting under standard conditions, one can predict that: a. at equilibrium, the concentration of B will exceed the concentration of A. b. at equilibrium, the concentration of C will be less than the concentration of A. c. at equilibrium, the concentration of C will be much greater than the concentration of A or B. d. C will rapidly break down to A + B. 128. Which of the following compounds has the largest negative value for the standard free-energy

change? (∆G'°) upon hydrolysis? a. Acetic anhydride b. Glucose 6-phosphate c. Glutamine d. Glycerol 3-phosphate 129. For the following reaction, ∆G'° = +29.7 kJ/mol. L-Malate + NAD+  oxaloacetate + NADH + H+ The reaction as written: a. can never occur in a cell. b. can occur in a cell only if it is coupled to another reaction for which ∆G'° is positive. c. can occur only in a cell in which NADH is converted to NAD+ by electron transport. d. cannot occur because of its large activation energy. e. may occur in cells at some concentrations of substrate and product. 130. For the reaction A → B, the Keq' is 104. If a reaction mixture originally contains 1 mmol of A and no B, which one of the following must be true? a. At equilibrium, there will be far more B than A. b. The rate of the reaction is very slow. c. The reaction requires coupling to an exergonic reaction in order to proceed. d. The reaction will proceed toward B at a very high rate. 131. In glycolysis, fructose 1,6-bisphosphate is converted to two products with a standard free-energy change (∆G'°) of 23.8 kJ/mol. Under what conditions encountered in a normal cell will the freeenergy change (∆G) be negative, enabling the reaction to proceed spontaneously to the right? a. Under standard conditions, enough energy is released to drive the reaction to the right. b. The reaction will not go to the right spontaneously under any conditions because the ∆G'° is positive. c. The reaction will proceed spontaneously to the right if there is a high concentration of products relative to the concentration of fructose 1,6-bisphosphate. d. The reaction will proceed spontaneously to the right if there is a high concentration of fructose e. 1,6-bisphosphate relative to the concentration of products. 132. During glycolysis, glucose 1-phosphate is converted to fructose 6-phosphate in two successive reactions: Glucose 1-phosphate --> glucose 6-phosphate ∆G'° = –7.1 kJ/mol Glucose 6-phosphate  fructose 6-phosphate ∆G'° = +1.7 kJ/mol ∆G'° for the overall reaction is: a. –8.8 kJ/mol. b. –7.1 kJ/mol. c. –5.4 kJ/mol. d. +5.4 kJ/mol. 133. The standard free-energy changes for the reactions below are given. Phosphocreatine creatine + Pi ∆G'° = –43.0 kJ/mol ATP ADP + Pi ∆G'° = –30.5 kJ/mol What is the overall ∆G'° for the following reaction? Phosphocreatine + ADP creatine + ATP a. –73.5 kJ/mol b. –12.5 kJ/mol

c. +12.5 kJ/mol d. +73.5 kJ/mol e. ∆G'° cannot be calculated without Keq'. 134. The ∆G'° values for the two reactions shown below are given. Oxaloacetate + acetyl-CoA + H2O citrate + CoASH ∆G'° = –32.2 kJ/mol citrate synthase Oxaloacetate + acetate  citrate ∆G'° = –1.9 kJ/mol citrate lyase What is the ∆G'° for the hydrolysis of acetyl-CoA? Acetyl-CoA + H O  acetate + CoASH + H+ a. –34.1 kJ/mol b. –32.2 kJ/mol c. –30.3 kJ/mol d. +61.9 kJ/mol e. +34.1 kJ/mol 135. All of the following contribute to the large, negative, free-energy change upon hydrolysis of “highenergy” compounds except: a. electrostatic repulsion in the reactant. b. low activation energy of forward reaction. c. stabilization of products by extra resonance forms. d. stabilization of products by ionization. 136. The hydrolysis of ATP has a large negative ∆G'°; nevertheless it is stable in solution due to: a. entropy stabilization. b. ionization of the phosphates. c. resonance stabilization. d. the hydrolysis reaction being endergonic. e. the hydrolysis reaction having a large activation energy. 137. The hydrolysis of phosphoenolpyruvate proceeds with a ∆G'° of about –62 kJ/mol. The greatest contributing factors to this reaction are the destabilization of the reactants by electostatic repulsion and stabilization of the product pyruvate by: a. electrostatic attraction. b. ionization. c. polarization. d. resonance. e. tautomerization. 138. Which one of the following compounds does not have a large negative free energy of hydrolysis? a. 1,3-bis phosphoglycerate b. 3-phosphoglycerate c. ADP d. Phosphoenolpyruvate 139. The immediate precursors of DNA and RNA synthesis in the cell all contain: a. 3' triphosphates. b. 5' triphosphates. c. adenine. d. deoxyribose.

140. Muscle contraction involves the conversion of: a. chemical energy to kinetic energy. b. chemical energy to potential energy. c. kinetic energy to chemical energy. d. potential energy to chemical energy. 141. Biological oxidation-reduction reactions always involve: a. direct participation of oxygen. b. formation of water. c. mitochondria. d. transfer of electron(s). 142. The standard reduction potentials (E'°) for the following half reactions are given. Fumarate + 2H+ + 2e–  succinate E'° = +0.031 V FAD + 2H+ + 2e– FADH E'° = –0.219 V If you mixed succinate, fumarate, FAD, and FADH2 together, all at l M concentrations and in the presence of succinate dehydrogenase, which of the following would happen initially? a. Fumarate and succinate would become oxidized; FAD and FADH2 would become reduced. b. Fumarate would become reduced, FADH2 would become oxidized. c. No reaction would occur because all reactants and products are already at their standard concentrations. d. Succinate would become oxidized, FAD would become reduced. e. Succinate would become oxidized, FADH2 would be unchanged because it is a cofactor. 143. The structure of NAD does not include: a. a flavin nucleotide. b. a pyrophosphate bond. c. an adenine nucleotide. d. nicotinamide. e. two ribose residues. 144. The standard free energy change (∆G'°) for ATP hydrolysis is –30.5 kJ/mol. ATP, ADP, and Pi are mixed together at initial concentrations of 1 M of each, then left alone until the reaction: ADP + Pi → ATP has come to equilibrium. For each species (i.e., ATP, ADP, and Pi) indicate whether the concentration will be equal to 1 M, less than 1 M, or greater than 1 M. a. ATP < 1 M; ADP > 1 M; Pi > 1 M b. ATP < 1 M; ADP < 1 M; Pi < 1 M c. ATP >1 M; ADP > 1 M; Pi > 1 M d. ATP < 1 M; ADP > 1 M; Pi < 1 M 145. If a 0.1 M solution of glucose 1-phosphate is incubated with a catalytic amount of phosphoglucomutase, the glucose 1-phosphate is transformed to glucose 6-phosphate until equilibrium is reached. At equilibrium, the concentration of glucose 1-phosphate is 4.5 x 10–3 M and that of glucose 6-phosphate is 8.6 x 10–2 M. What is the value of ∆G'° for this reaction?(in the direction of glucose 6-phosphate formation). (R = 8.315 J/mol•K; T = 298 K) a. –8.3 kJ/mol b. +7.3 kJ/mol c. –7.3 kJ/mol d. +8.3 kJ/mol

146. Which of the following statement is wrong? 1) In a reaction under standard conditions, only the reactants are fixed at 1 M. 2) When ∆G'° is positive, Keq' > 1. 3) ∆G and ∆G'° mean the same thing. 4) When ∆G'° = 1.0 kJ/mol, Keq' = 1. a. Only 1 b. Only 2 c. Only 3 d. All of them 147. Why is the actual free energy (∆G) of hydrolysis of ATP in the cell different from the standard free energy (∆G'°)? a. The concentrations of the reactant (ATP) and the products (ADP and Pi) are not all equal to 1M. The pH is not exactly equal to 7.0 (the standard pH) and the temperature is usually higher than 25C b. The cell has several other ingradient which influence the ATP level c. The cells have more than 1 mitochondria d. Gibbs value for each mitochondria is different. 148. The free energy of hydrolysis of phosphoenolpyruvate is –61.9 kJ/mol. Rationalize this large, negative value for ∆G'° in chemical terms. a. The product of hydrolysis is keto pyruvate, which quickly tautomerizes to pyruvate. This keto- enol tautomerization stabilizes the products of hydrolysis relative to phosphoenolpyruvate, which cannot tautomerize. b. The product of hydrolysis is enol pyruvate, which quickly tautomerizes to pyruvate. This keto- enol tautomerization stabilizes the products of hydrolysis relative to phosphoenolpyruvate, which cannot tautomerize. c. The product of hydrolysis is enol pyruvate, which does not quickly tautomerizes to pyruvate. This keto- enol tautomerization stabilizes the products of hydrolysis relative to phosphoenolpyruvate, which cannot tautomerize. d. The product of hydrolysis is enol pyruvate, which quickly tautomerizes to pyruvate. This keto- enol tautomerization does not stabilizes the products of hydrolysis relative to phosphoenolpyruvate, which cannot tautomerize. 149. Match the following , List I shows forms of energy conversion and list Ii shows examples for each. List I P p. Chemical to osmotic energy Qq. membrane electromagnetic to electrical energy R r. Chemical to electromagnetic energy S.s. Chemical to mechanical energy a. P-II; Q-III; R-I; S-IV b. P-I; Q-III; R-I; S-IV c. P-II; Q-III; R-IV; S-I d. P-II; Q-I; R-II; S-III

List II I. I. ATP-driven muscle contraction IIII. ATP-driven active transport across a Il III. light-induced electron flow in chloroplasts I IV. ATP-dependent photon emission in fireflies

150. Metabolism of which of the following molecules results in the greatest net usable energy per gram? a. A triglyceride

b. A tripeptide c. An alpha-linked disaccharide d. A beta-linked disaccharide 151. Which of the following compounds has the largest negative value for the standard free-energy change (∆G'°) upon hydrolysis? a. Acetic anhydride b. Glucose 6-phosphate c. Glutamine d. Glycerol 3-phosphate e. Lactose 152. For the following reaction, ∆G'° = +29.7 kJ/mol. L-Malate + NAD+ → oxaloacetate + NADH + H+ The reaction as written: a. can never occur in a cell. b. can occur in a cell only if it is coupled to another reaction for which ∆G'° is positive. c. can occur only in a cell in which NADH is converted to NAD+ by electron transport. d. cannot occur because of its large activation energy. e. may occur in cells at some concentrations of substrate and product. 153. In glycolysis, fructose 1,6-bisphosphate is converted to two products with a standard free-energy change (∆G'°) of 23.8 kJ/mol. Under what conditions encountered in a normal cell will the freeenergy change (∆G) be negative, enabling the reaction to proceed spontaneously to the right? a. Under standard conditions, enough energy is released to drive the reaction to the right. b. The reaction will not go to the right spontaneously under any conditions because the ∆G'° is positive. c. The reaction will proceed spontaneously to the right if there is a high concentration of products relative to the concentration of fructose 1,6-bisphosphate. d. The reaction will proceed spontaneously to the right if there is a high concentration of fructose 1,6bisphosphate relative to the concentration of products. 154. All of the following contribute to the large, negative, free-energy change upon hydrolysis of “highenergy” compoundsexcept: a. electrostatic repulsion in the reactant. b. low activation energy of forward reaction. c. stabilization of products by extra resonance forms. d. stabilization of products by ionization. 155. The hydrolysis of ATP has a large negative ∆G'°; nevertheless it is stable in solution due to: a. entropy stabilization. b. ionization of the phosphates. c. resonance stabilization. d. the hydrolysis reaction being endergonic. e. the hydrolysis reaction having a large activation energy. 156. Which one of the following compounds does not have a large negative free energy of hydrolysis? a. 1,3-bis phosphoglycerate b. 3-phosphoglycerate c. ADP d. Phosphoenolpyruvate e. Thioesters (e.g. acetyl-CoA)

ANSWER KEYS: 77.A 78.A 79.C 80.C 81.D 82.A 83.C 84.B 85.E 86.C 87.B 88.D 89.B 90.A 91.B 92.A 93.D 94.B 95.A 96.A 97.C 98.D 99.B 100.B 101.E 102.D 103.D 104.B 105.A 106.B 107.D 108.D 109.A 110.A 111.B 112.D 113.B 114.C 115.A 116.A 117.A 118.B 119.C 120.D 121.A 122.A 123.C 124.D

125.C 126.D 127.C 128.B 129.A 130.B 131.C 132.C 133.A 134.A 135.C 136.C 137.C 138.A 139.A 140.A 141.C 142.B 143.C 144.A 145.A 146.A 147.C 148.C 149.A 150.B 151.D 152.C 153.A 154.B 155.A 156.A 157.C 158.A 159.B 160.A 161.A 162.B 163.C 164.A 165.A 166.E 167.B 168.A 169.A 170.E 171.E 172.B 173.B 174.A

175.D 176.C 177.C 178.C 179.A 180.E 181.D 182.B 183.C 184.B 185.D 186.B 187.B 188.C 189.B 190. 191.B 192.A 193.B 194.B 195.D 196.A 197.E 198.E 199.C 200.A 201.D 202.A 203.C 204.A 205.E 206.A 207.A 208.C 209.B 210.C 211.B 212.E 213.E 214.B 215.B 216.A 217.D 218.B 219.A 220.B 221.C 222.D 223.A 224.B

225.C 226.A 227.A 228.E 229.A 230.B 231.E 232.B

Related Documents

Unit-1
January 2021 7
Unit 1
February 2021 3
Unit 1 (1).docx
February 2021 3
Unit 1 Management
January 2021 1

More Documents from "Saket Verma"